0% found this document useful (0 votes)
17 views237 pages

9417

Uploaded by

fatimahurain251
Copyright
© © All Rights Reserved
We take content rights seriously. If you suspect this is your content, claim it here.
Available Formats
Download as DOCX, PDF, TXT or read online on Scribd
0% found this document useful (0 votes)
17 views237 pages

9417

Uploaded by

fatimahurain251
Copyright
© © All Rights Reserved
We take content rights seriously. If you suspect this is your content, claim it here.
Available Formats
Download as DOCX, PDF, TXT or read online on Scribd
You are on page 1/ 237

UNDERSTANDING

MATHEMATICS AND STATISTICS

Units 1-9 Code-9417

DEPARTMENT OF MATHEMATICS &


DEPARTMENT OF STATISTICS
ALLAMA IQBAL OPEN UNIVERSITY, ISLAMABAD

i
(All rights reserved with the publisher)

Year of Printing .................................................................................................2020

Quantity ............................................................................................2000

Price .........................................................................................................

Printer .........................................................................................................

Publisher .....................................Allama Iqbal Open University, Islamabad

ii
COURSE DEVELOPMENT TEAM
DEAN FACULTY OF SCIENCES: P ROF .D R .S YED ZAFAR ILYAS

DEVELOPMENT COORDINATOR: DR.NASIR REHMAN

WRITERS: 1.DR. NASIR REHMAN


2.D R .I RSHAD AHMAD ARSHAD
3. DR.MUHAMMAD ZAKRIA
4.D R . ZAHID IQBAL
5.D R .Y AMEEN DANISH

6.M S . FOUZIA REHMAN

7.M S.M UBASHARA HAFEEZ

REVIEWERS: 1.D R .B ABER AHMAD


2.D R . IRSHAD AHMAD IRSHAD
3.D R .M UHAMMAD ZAKRIYA

EDITOR: MR.FAZAL KARIM DR.

COURSE COORDINATOR: NASIR REHMAN

COMPOSED BY: MS.FOUZIA REHMAN

iii
COURSE CONTENTS
Page No.
Preface ............................................................................................................... v

Message from the Dean ..................................................................................................... vi

Acknowledgments ............................................................................................................ vii

Course Introduction ......................................................................................................... viii

Unit-1: Arithmetics ............................................................................................ 1 Unit-

2 Linear and Quadratic Equations .......................................................... 37 Unit-3

Matrices and Determinants .................................................................. 56 Unit-4

Sequences, Series and Mathematical Induction................................... 84 Unit-5 Limit

and Differentiation ................................................................... 107 Unit-6 Introduction

to Statistics ................................................................... 126 Unit-7 Presentation of

Data........................................................................... 139 Unit-8 Measure of Central

Tendency............................................................ 154 Unit-9 Measure of

Dispersion ....................................................................... 189

iv
PREFACE
Mathematics as a subject is a key and door to all other fields of knowledge. The history of
mathematics, therefore, is as old as the history of humanity. The branches of
mathematical sciences like mathematics and statistics are essential to study all fields of
social, natural and other sciences. It is rightly stated by an eminent philosopher Galileo that
God has created this universe mathematically. The beauty of mathematics is that it goes like
a music tune if you move forward logically. In other words, it creates logical thinking into
minds. Thus, to improve our living standards and to make the best use of natural resources
available, understanding of the basic concepts of mathematics and statistics are
necessary.

The present book is a step in this direction. The text of the book is simple and has been
developed to provide the essential knowledge of basic mathematics and statistics. A lot of
effort has been put in and many fruitful discussions are given to develop this new
performance-based course. I hope that this book will appeal to the intuition of students and
provide a great deal of satisfaction and visual reinforcement. This modern approach based
on illustrations and discussions given in the book, I am sure, has enhanced the readability
and comprehensibility of the topics for its readers and it is indeed an important and
encouraging aspect of this book as a course-reading.

I wish to express my sincere congratulations to all those who have contributed either
directly or indirectly in the development of this book in such a short span of time.

Prof. Dr. Zia Ul-Qayyum


(Vice Chancellor)

v
Message from the Dean
Mathematics and Statistics are the branches of science which develop a relationship and fill
the gaps between the other subjects of natural sciences. Naturally Mathematics and Statistics
look a bit abstract but when we use it in other sciences, their applications come across us at
once. This course is much needed for our students to inculcate the first-level concepts. The
basic mathematics and statistics knowledge is beautifully covered in this book.

I wish to express my sincere thanks, appreciation and congratulations to all those persons
who have contributed in the development of this book.

(Prof. Dr. Syed Zafar Ilyas)


Dean F/O Sciences

vi
ACKNOWLEDGFMENTS
I am thankful to Almighty Allah for giving me the opportunity to develop this course. I
would like to express my gratitude to the many people who saw me throughout the
preparation of this book and to all those who provided support, talked things over read,
wrote, offered comments and assisted in the editing, proof reading and design.

I would like to thank Dr. Waseem ul Haq, Dr. Babar Ahmed, Dr. Irshad Ahmad Arshad, Dr.
Irfan Mustafa, Mr. M. Aatif, Mr. M. Faisal Iqbal, Dr. M Nazam, Dr. Saqib Hussain, Dr.
Sultan Hussain, Dr. Liaquat Ali Khan, Mr. A. Waseem Siddiqui, Dr. M. Hanif, Dr. Zahid
Iqbal, Dr. M Yameen Danishand Dr. Muhammad Zakria for their constant support and
encouragement. I am especially grateful to Dr. Babar for his extremely detailed
reviews and suggestions of the manuscript many times till the finalization of this book.
Many extraordinary talented are responsible for helping to create this text.

I would like to thank Mr. Fazal Karim and Mr. Sajid for composing and editing this course
on priority basis. Particular thanks go to each of my colleagues at AIOU. Our everyday
conversations regarding Mathematics instruction have been of the utmost importance to
the development of this text and to my teaching career.

I am highly grateful to Prof. Dr. Syed Zafar Ilyas, Dean Faculty of Sciences and Prof. Dr.
Zia ul Qayyum, Vice-Chancellor, AIOU for giving creative ideas of writing
challengeable mathematical concepts in a simple way and provided all facilities to
complete this course in the shortest possible time.

Suggestions and criticism to improve the text will be highly appreciated. It can be
forwarded at [email protected]

(Dr. Nasir Rehman)


Associate Professor/Chairman
Department of Mathematics
Course Coordinator

vii
COURSE INTRODUCTION
This course is basically designed to develop and inculcate the basic concepts of
Mathematics and Statistics in our students at undergraduate level. An effort has been made
to make these contents easy and understandable. It is the need of the hour to understand the
applications of Mathematics and Statistics in studying different subjects of Social Sciences
and Islamic Studies.

Objectives of the Course


The course aims to teach students, the basic concepts of Mathematics and Statistics and
also create value for their use in practical situations.

The objectives of the course are:


• to develop an understanding and desire of Mathematics and Statistics in students
• to present the material in a way which is helpful in motivating the students to study
these subject sat a higher level.

How the course is organized?


The text is divided into nine units in one volume. An attempt has been made to present the
material in an informal way. Only those topics of Mathematics and Statistics are covered
which are thought to be useful for everyone. e.g. we started studying Mathematics
when we were in class one and we continue its studies it till class 10th. In spite of this most
of us do not have full command on every day arithmetics.

Unit-1 Unit 1 covers some basic concepts of arithmetic like real number system,
ratio and proportion algebra and sets. These are the basic pillars of
mathematics to understand the further advanced mathematics topics. You
might have noticed that the land for cultivation is usually divided into
different geometrical shapes, such as square, rectangle, and triangle.
Similarly, these shapes can also be seen in construction of buildings.
Impractical situation, sometime you are interested to find the area of such
shapes.

Unit-2 Unit 2 is designed to provide you necessary tools needed in further study of
Mathematics. Various types of equations and their solutions are introduced in
this unit.

Unit-3 Unit 3 is about the introduction of matrices and their uses in the solution of
different practical problems. Solutions of linear equations are also given at the
end of this unit.

Unit-4 Unit 4 explains sequences, series and mathematical induction with the help
of many examples. Also the uses of sequences in other branches of
Mathematics like Physics and Chemistry has been discussed.

viii
Unit-5 In Unit 5 we define and explain the concepts of limit and derivatives which
are very basic and important to understand the other difficult mathematical
concepts later on.

Unit-6 Unit 6 explains the definition, nature, importance and limitations of the
Statistics.

Unit-7 Unit 7 explains that how large data is summarized and presented by using
different Statistical tools.

Unit-8 Unit 8 covers the calculations and properties of measures of central tendency,
including mean, mode, median etc.

Unit-9 Unit 9 is about different measures of dispersion and their application to data
in different fields.

ix
UNIT- 1

ARITHMETICS

Written By: Dr. Nasir Rehman


Reviewed By: Dr. Baber Ahmed

1
CONTENTS
Page No
Introduction ............................................................................................................... 3
Objectives ............................................................................................................... 3
1.1 Real Number System................................................................................................. 4

1.2 Direct and Inverse Proportions: ................................................................................ 5


1.3 Basic Algebra ........................................................................................................... 10
1.3.1 Algebraic Terms ............................................................................................ 10
1.3.2 Algebraic Sentences ...................................................................................... 10
1.3.3 Algebraic Equations ...................................................................................... 11

1.4. Algebraic Operations ............................................................................................... 11


1.4.1 Addition ......................................................................................................... 11
1.4.2 Subtraction .................................................................................................... 11
1.4.3 Multiplication ................................................................................................ 12
1.4.4 Division ......................................................................................................... 13
1.4.5 Three Basic Algebraic Identities ................................................................... 14

1.5 Profit, Loss and Percentages ................................................................................... 16


1.6 Sets ............................................................................................................. 19
1.6.1 Set Description: .............................................................................................. 19
i. Descriptive Method .................................................................................... 19
ii. Tabular Method ......................................................................................... 19
iii. Set-Builder Notation ................................................................................ 19
1.6.2 Set Types ...................................................................................................... 20
1.6.3 Basic Operations on Sets ............................................................................... 20
1.6.4 Venn Diagram ............................................................................................... 21

1.7 Distance Formula ....................................................................................................... 24


1.8 Basic Geometric Shapes ............................................................................................ 26
1.8.1 Triangle ......................................................................................................... 26
1.8.2 Quadrilateral .................................................................................................. 31
1.8.3 Circle ............................................................................................................. 32
1.8.4 Cylinder: ........................................................................................................ 33
1.8.5 Circular Cone: ............................................................................................... 34
1.8.6 Sphere: ........................................................................................................... 35

2
INTRODUCTION
The history of Arithmetics goes back with the history of the humanity. As soon as the
human started thinking, the Arithmetic started right there. It is the language that human
needed all the times. That is why Mathematics is called the mother of all sciences. The
mathematics that we use in our daily life is called basic Arithmetics. It includes real
numbers, basic operations of addition, subtraction, multiplication and division, profit and
loss, direct and inverse proportion, sets, algebra, geometry, area and volume, graphs and
many more.

OBJECTIVES
At the end of unit (1), the students will be able to:

1. understand the nature of different real numbers.

2. find direct and inversion proportions.

3. perform basic algebraic operations of addition, subtraction, multiplication and


division for the algebraic sentences.

4. calculate profit, loss and percentages.

5. use sets and Venn diagrams to solve different problems.

6. find the distance between two points.

7. calculate the surface area of different shapes.

8. measure the capacity and volume of different objects

3
1.1 Real Number System.
Numbers play an important role in our practical life. Every time we are dealing with the
numbers whether we have been counting something, collecting the data of the ages of
human beings, measuring the heights of certain trees, recording the temperature of a certain
city at different times. People have been using different symbols in the past for these
observations and with the passage of time, these symbols were transformed and modified
so that today we use the set {1,2,3,4,….} as a counting set of basic instrument
in Mathematics and call it the set of natural numbers. Some important number
systems are given below:

N: Natural numbers 1, 2, 3, 4, 5, …
Z: Integers,0,±1,±2,±3 ,±4,…
W: Whole numbers, 0, 1, 2, 3, 4, …
Q: Rational numbers, ½, 3, 0
I: Irrational numbers, √2, 2 , 1 ± √5
√3

Real Numbers can be classified into two groups which are known as rational
numbers and irrational numbers. Rational numbers are those which can be
written in the form p/q where q ≠ 0 and p and q are integers, while the irrational
numbers are those which cannot be written in the form p/q(p and q both integers).

-∞ -3 -2 -1 -0.2 0 ½ 1√2 2 3…. +∞

Real numbers satisfy the following properties with respect to addition and multiplication

Let a, b, and c be three real numbers i.e. , , ∈ ℝ. Then

1. a + b = b + a for all a, b ℝ
Real number system is commutative with respect to the basic operation of addition.
2. (a + b) + c = a + (b + c) for all a, b IR
Real number system is associative with the respect to the basic operation of
addition.
3. There exists an element 0 IR such that 0 + a = a + 0 = a for all real numbers a IR.
This is called zero element or additive identity.

4. For all real numbers a IR, there exists –a R such that we have
a + (-a) = (-a) + a = 0. This is called negative element or additive inverse of a real
number.
5. a. b= b. a for all a, b IR.
The real number system is commutative with respect to multiplication also.

4
6. (a. b). c = a. (b. c) for all a, b IR The real number system is associative
with respect to multiplication also.

7. There exists an element 1 IR such that 1.a = a.1 = a for all real numbers a IR, this is
called unit element or multiplicative identity.

8. For all non-zero real numbers a IR, there exists an element 1/a IR such that we have
1 1 . This is called reciprocal element or the multiplicative inverse of a
. = . =1
real number.

9. In this property we have both (the operations of addition and multiplication)


operations together.
. ( + ) = ( . ) + ( . ) or ( + ). = ( . ) + ( . ) for all real numbers a, b, c
IR. This is called distributive property of multiplication over addition.

Self-Assessment Questions
Q. 1 Represent the following real numbers on a real line:
i) 1.25 ii) 1 1 iii) √3 iv) 1 + √2 v) −0.5 vi) −4 2
vii) 1 + 1 viii) 0 ix) +∞ x) 1
√2 2−√3

Q. 2 Identify the property of real numbers used in each question


i) 2+3=3+2 ii) 0.5 + 0 = 0 + 0.5 iii) 3 + (−3) = 0
iv) 2(3 + 4) = 2(3) + 2(4) v) 10 × 1
=1
10

1.2 Direct and Inverse Proportions:


Often the things are changing (increasing or decreasing) relative to other things. These
changes are basically of two types. In the first case both the objects have similar change and
in the second case the objects have opposite behavior. In this unit we will see these
important properties of objects which are known as direct proportions and inverse
proportions.

Consider the following example where we borrow a book from the Central Library of AIOU
and due to certain reasons we cannot return the book within the specified time so the library
charges a fine for overdoing at a rate Rs.10 per day. Here when one object (no. of days) will
increase the other object (fine) will also increase and vice versa. So it is a question of direct
proportion. Let us denote these quantities using certain variables for example.
a) the number of days can be represented by variable x.
b) and the fine is represented by variable y.

5
then we say that the fine (y) is directly proportional to the number of days (x).
The following table shows the fine for the overdue book.

Overdue days x 1 2 3 4 5 10 15 20
Fine y 10 20 30 40 50 100 150 200

From the above table we observe that if the numbers of days increase, the fine also increases
with the constant rate. Now to calculate this rate we can use the information of
abovementioned table i.e. dividing the fine by no. of days (y/x). We can see that this number
(rate):
10 20 30 40 50 100 150 200 =
= = = = = = = 10
1 2 3 4 5 10 15 20
comes out to be a constant.

So, we can say that:


1 = 2 = 3 = 4 = − − − − −−= 20 1
2 3 4 20

We have the same rate throughout the example.


Therefore, we have the relation 1 = 2 , the question will be of direct proportion or we
1 2
can say that when y is directly proportional to x then we have 1 = 2 .
1 2
We can rearrange this formula and write in the following form 2 = 2 .
1 1

Example 1. Find if 15:20 = 6:

Solution. We are given 15:20 = 6:

Using the direct proportion rule 15 = 6


20
We have
15 = 120 => =8
Example 2. Find the cost of 15 kg of apples if the cost of 4kg of the apples is Rs.240.

Solution. As the weight of the apples increases, the total cost also increases so it is a
question of direct proportion and we use the rule for direct proportion i.e.
1 = 2 to solve this example.
1 2
Weight 4 15
Price 240
We have 240 =
4 15

6
i.e. = 15×240
4

= 900
Therefore, the price of 15kg of apples will be Rs. 900.

Self-Assessment Questions.
Q.1 Find the value of if,
i) :27 = 13:6 ii) 5: = 24:60 iii) 5:8 = :32
iv) 3:5 = 6: v) 4: = 10:25

Q.2 Find the cost of hiring 12 technicians for a task if the cost of hiring the five
technicians is Rs. 200.

Constant of Proportion
We represent the constant rate in the case of direct proportion by and use it. Solve the
problems of direct proportion in the following form. If is directly proportional to then we
write.

By introducing the constant , we have
=
⇒ = where is constant number which remains the same throughout the
question under consideration.

Example: If is directly proportional to and = 20 where = 4 , find where


= 10. Also find where = 24
Solution: Since is directly proportional to , so
∝ ⇒ =
where is constant.
Putting the value of and in the above equation we find the value of the constant i.e.
20 = ∗ 4 ⇒ = 5

Therefore, the above equation takes the form


=5

Now we can use this equality to solve our both parts.

i) = 10, =? = 5 ∗ 10 = 50

ii) = 24, =? 24 = 5 ∗

⇒ = 24
= 4.8
5

7
Self- Assessment Questions.
Q. 1 Given that y is directly proportional to x. Complete the following table using direct
proportionality.
x 6 8 9 10
y 4 9 10
Inverse Proportion
Consider a construction company which wants to build a bridge. If it hires 40 labors, the
work will be completed in 60 days. If it hires labors more than 40, the work will be
completed obviously earlier. If the labors are 80, the work will be completed in 30 days.
However, if the company has shortage of labors the completion of bridge will be delayed.
So, we observe that if the number of labors increases, the number of days to complete the
bridge will decrease proportionally and if the number of labors decreases, the number of
days to complete the job will increase proportionally. This is called inverse proportion. This
information is represented in the table below.

No. of Labors x 40 60 80 100 120


No. of Days y 60 40 30 24 20

From the above table we observe that if the number of labors increase, the number of
days’ decreases with the same rate i.e. if the labors are doubled the speed of work will be
doubled and work will be completed in half days. It is clear from the table that the product
of the number of labors and the number of days remains constant i.e.

= 40 × 60 = 60 × 40 = 80 × 30 = ⋯
⇒ = 2400

So we can write
1 1 = 2 2 = 2400
• 1 = 2
2 1

Example: Find the time to complete the journey at a speed of 60km/hour. If it takes 4
hours to complete the same journey at a speed of 45km/ hour.

Solution: As the speed increases, the total time of the journey will decrease so it is a
question of inverse proportion and we use the rule for inverse proportion
i.e. 1 = 2 to solve this example.
2 1

Speed km/h 45 60
Time hours 4
We have

4 60
= 45

8
45 × 4
⇒ 60 × = 45 × 4 ⇒ = =3
60

Therefore, it will take 3 hours to complete the journey at a speed of 60km/h.

Self- Assessment Questions.


Q.1 Six equal pipes fill an empty tank in 90 mints. How long will it take to fill the same
tank of water if we use 4 pipes?

Q.2 If 12 persons complete a task in 12 days. How long will it take to 4 persons to
complete the same task?

Q.3 A car travels a distance at average a speed of 40km/hour in 5 hours. How long will
it take to travel the same distance if the speed of a car is increased by 10km/h?

Constant of Proportion

Now we change the inverse proportionality by an equality by introducing a constant and


use it to solve the problems of inverse proportions if is inversely proportional to then we
write.
1

By introducing the constant we have =
⇒ =
Where the constant k remains the same throughout the question under consideration.

Example. If the time ( ) to cover a distance is inversely proportional to the speed and
= 12 where = 100, find when = 8 and also find when = 80.

Solution. Since time is inversely proportional to the speed,


So,
1
∝ ⇒ =
where k is a constant.
Putting the values of and in the above equation we find the value of i.e.
12 = ⇒ = 12 ∗ 100 = 1200
100

Therefore, the above equation takes the form

1200
=

9
Now we can use this equation to solve both the parts.

i) = 8, 8 = 1200 ⇒ = 1200 = 150 =?


8

ii) = 80, =? = 1200 ⇒ = 15


80

Self-assessment Questions:
Q. 1 If is inversely proportional to and = 6where = 8, find an equation relating
also using that equation find where = 12?

Q. 2 Given that is inversely proportional to . Complete the following table using the
inverse proportionality.

x 1 2 4 6
y 6 12 2

Q. 3 Suppose that the time to complete task is inversely proportional to the number of
workers employed for that. If 6 workers do that job in 4 hours, then how many workers are
required if we are forced to compete the same task in ½ hour.

1.3 Basic Algebra


Algebra is one of the oldest branch of mathematics. Muslim scientists have a lot of
contribution in the development of this subject. Even the word ‘algebra’ was originated by
the Muslim scientist in the form of “Al-Jabr w’al-Muqabala”.

In algebra we have basically two types of terms. These are known as constants and
variables. Constants have fix values obviously and are denoted by usually first alphabets a,
b, c, etc. While the variables can adopt different values according to the situation of the
problem and they are denoted by the last alphabets, x, y, etc.
1.3.1 Algebraic Terms
When we combine constants and variables we get algebraic terms for example
2 ,−4 , 2 3 , 4

1.3.2 Algebraic Sentences


When we combine constants and variables by basic mathematical operations we get
algebraic sentences for example 2 , + 5, 2 − 3 ,−2 + 1.
In 2 : x is called variable and 2 is called coefficient
In + 5: is called variable and 5 is constant.
In 2 − 3 : is called variable, 2 is called exponent and −3 is coefficient.

10
1.3.3 Algebraic Equations
The algebraic sentences that involve the symbol of equality (=) are called algebraic
equations for example + 5 = 0, 2 + 3 = 2, −2 + 1 = 5, = 0

Self-Assessment Questions
Q. 1 Find variables, constants, coefficients, exponents from the following:

i) 2+3 ii) 2 − 2 = 5 iii) 4 2 + 40


iv) 2=0 v) x+y=2

1.4 Algebraic Operations


Now we learn about the four basic operations of addition, subtraction, multiplication and
division for algebraic sentences and equations.
1.4.1 Addition
Two or more algebraic terms can be added only if they have same variables and same
exponents. While doing addition their coefficient is added for example 2 and 3 can be added
became variable is same and both the terms have same exponent 1. So we get 2 + 3
= 5 . But 2 and 3 cannot be added because the variables are different although the exponent
is same so we write it as 2 + 3 = 2 + 3

Example 1: Add 2 3 + 3 2 + 5 − 1 and 3 − 2 −2 +5

Solution: 2 3 + 3 2 + 5 − 1
+ 3− 2−2+5 33 + 22
+3+4

1.4.2 Subtraction

Two algebraic terms can be subtracted only if they have the same variables and same
exponents (like the case of addition, while doing the subtraction the coefficient of the
second term is subtracted from the coefficient of the first term. For example, 2 and 3 can be
subtracted because both have same variable x and the same exponent 1. So we get 2 − 3 =
−1 = − . But 2 and 3 cannot be subtracted because the variables are different although the
exponent is same so we write it as 2 − 3 = 2 − 3
−1 3 + 1 2 + 3 +6
Example 2: Subtract 2 3 + 3 2 − 5 − 1 from

Solution: 3 + 42 − 2 + 5
− ± 23 ± 32 ∓ 5 ∓ 1

11
3 + 42 − 2 + 5
Self-Assessment Questions
Q. 1 Add the following algebraic sentences
i) + 2 + 1, 2 + 2 2 + 2, 3 2 + 3 + 3
ii) + , 2 +3
iii) 2 2 + 5, 2 + + 1
iv) 2 + 2 + 1, 2 2 + 3, 2 +

Q. 2 Subtract the second algebraic sentence from the first algebraic sentence
i) 2 2 + − 5, 2 + 5 + 2
ii) 2 +3 −3 , − +
iii) 2 + + 1, 2 2 + 5
iv) 4 − 3 + 2 + 5, 4 + 3 − + 5

1.4.3 Multiplication
Unlike the addition and subtraction, any two algebraic terms can be multiplied, while doing
multiplication correspondence coefficients are multiplied and the exponents of the same
variables are added for example.
2 ∗ 3 = 2 ∗ 3 1+1 = 6 2
2 ∗3 =2∗3 =6

Example 1: Multiply 2 2 + 3 + 1 by +5

Solution:

Example 2: Multiply 2 + + 2 + 1 by 2 + 5

12
Solution:

1.4.4 Division

Any two algebraic terms can be divided provided that the term in the denominator is not
2 2 2 2
zero i.e. We can divide but we cannot divide , while doing division, corresponding
0
coefficients are divided and the exponents of the same variables are subtracted for
example.
2 2 2 2
2 ÷3 = 1−1 = 0 = (1) =
3 3 3 3
2 ÷3 = 2 =2
3 3

Example 3: Divide 2 2 + 3 − 35 by +5

Solution:

Therefore,
(2 2 + 3 − 35) ÷ ( + 5) = 2 − 7

Self-Assessment Questions
Q. 1 Multiply the following algebraic sentence:
i) + 5, − 7
ii) 2 + + 1, −1
iii) 2 − 1, 2 + 1
iv) + , 2 + +5
13
v) 2 − 2 + 1, +1

14
Q. 2 Divide the first algebraic sentence by the second algebraic sentence:

i) 2 + 2 + 1, + 1
ii) 2 3 + 3 2 + 4 + 5, − 2
iii) 2 − 4 + 4, − 2
iv) 4 − 1, + 1
v) 2 2 + 2 + 1, + 1

1.4.5 Three Basic Algebraic Identities


Three basic algebraic identities are important to solve the problems of algebraic
manipulations. These identities include perfect squares of the sum of two algebraic terms
and difference of two algebraic terms and the difference of two squares. Consider the case
where we want to simplify (2012)2 − (2011)2. It is lengthy job to solve it but if we know
these identities then we can apply one of these. i.e.
Using the difference of two squares we will get
(2012)2 − (2011)2 = (2012 − 2011)(2012 + 2011)
= (1)(4023)
= 4023

There basic algebraic identities are following.

i) ( + )2 = ( + )( + )
= 2+ + + 2
∴ ( + )2 = 2 + 2 + 2

ii) ( − )2 = ( − )( − )
= 2− − + 2
∴ ( − )2 = 2 − 2 + 2

iii) ( − )( + ) = 2 + − − 2
∴ ( − )( + ) = 2 − 2
These are called basic identities

Example 1: Use identities (i),(ii)and (iii), to find

i) (3 + 5 )2 ii) (2 − 3 )2 iii) ( − 2 )( + 2 )

Solution:
i) Using identity (i) we have

(3 + 5 )2 = (3 )2 + 2(3 )(5 ) + (5 )2 = 9
ii) Using identity (ii) we have
(2 − 3 )2 = (2 )2 − 2(2 )(3 ) + (3 )2 = 4

15
2 + 30 + 25 2 2 − 12 + 9 2
iii) Using identity (iii) we have

( − 2 )( + 2 ) = ( )2 − (2 )2 = 2 −4 2

Example 2: Solve the following using algebraic identities:


i) (299)2
ii) 49×51
iii) (1001)2

Solution: Note that without calculator it is very hard to evaluate the above expressions. So
we use algebraic identities to make our solution simpler

i) ( 299)2 = (300 − 1)2


= (300)2 − 2(300)(1) + (1)2
= 90000 − 600 + 1
= 89401

ii) 49 × 51 = (50 − 1)(50 + 1)


= (50)2 − (1)2
= 2500 − 1
= 2499

iii) (1001)2 = (1000 + 1)2


= (1000)2 + 2(1000)(1) + (1)2
= 1000000 + 2000 + 1
= 1002001

Self-Assessment Questions
Q. 1 Use algebraic identities to solve the following:
i. (2 − 1)2
ii. ( + 3)2
iii. (2 − 5 )2
iv. ( − 3 )( + )
v. 1 2
( + )
vi. −1 1
( + )

Q. 2 Using algebraic identities to evaluate the following expressions:


i. (703)2
ii. (495)2
iii. (1501)2
iv. (892)2

16
v. (29)2 + 58 + 1

17
1.5 Profit, Loss and Percentages
Every one faces the questions of profit and loss in his daily life. Whether he is a
customer, shopkeeper, and manufacturer of some product, businessman, banker and many
more. They also want to know the percentage of their profit or loss. Profit and loss are
relative terms and these terms depend upon the actual price or cost price of a good. For
example, if a person sells some product at a price higher than the cost price then he makes a
profit. On the otherhand, if he is forced for some reasons to sell his product at a price less
than the cost price then he faces a loss. Therefore, we can write

Profit = Selling price – Cost price


Loss = Cost price – Selling price

Consider the example of a vegetable vendor who buys mangoes from fruit market at a rate
of Rs. 60 per kilogram and sells at a rate of Rs. 80 per kilogram. Then he makes a profit of
Rs. 20 per kilogram. On the second day he sells the remaining mangoes at a rate of Rs. 45
per kilogram. Then he faces a loss of Rs. 15 per kilogram. This information is represented
by using the above mentioned formula.

Profit = Selling price – cost price


= 80 − 60 = 20
Loss = Cost price – Selling price
= 60 − 45 = 15

These considerations give us profit and loss in a business but mostly we are interested to
find the percentage of profit and loss. To calculate these percentages, we use the
following formulae:
=% = 100

=% = 100

For the above example of vegetable vendor his percentage profit and loss can be
calculated as:
20
% = ∗ 100
60
= 33.33 %
15
% = ∗ 25
100
= 25 %

Example 1: The price of wheat flour is Rs. 30 per kilogram. If the government
announces an increase of 20%, what will be the new price of wheat flour?
18
Solution: Actual Price of wheat flour: = 30
Percentage increase: = 20%

New Price: =?

Increase = 20 % of actual price


= 20 ∗ 30
100
= 6

Therefore, new price will be 30 + 6


⟹ New price will be Rs. 36 per kilogram

Example 2: A shopkeeper gains a profit of 25% ,if he sells a school bag for Rs. 400. Find
the cost price of that bag.

Solution: Selling Price: = 400


Profit = 25%
Cost Price =?
Let the cost price of the bag = x
= 25% ℎ
25
= ∗ =
100 4

19
We know that;
Profit = selling price – cost price
= 400 −
4
+ = 400
4
5
= 400 ⟹ 5 = 1600
4
⟹ = 320

Therefore, the cost price of the bag will be Rs. 320.

Example 3. The profit on a television set is 20% of the cost price. Suppose the
shopkeeper gets an amount of Rs. 1500 as a profit. Find the cost price and
selling price of that television set.

Solution: = 1500
= 20%

=?
=?
We know that;
% profit = ∗ 100
1500
20 = ∗ 100
⟹ = 1500 ∗ 100 20
= 7500
Hence cost price = Rs. 7500
And
selling price = cost price + profit
= 7500 + 1500
= Rs. 9000
Self-assessment Questions
Q.1 Mr. Inam has monthly income of Rs. 12,000. He spends 25% out of his monthly
income on house rent, 10% on oil for bike, 20% on tuition fee of his children and 15%
on food items. Calculate the amount spent on all these things and also find his net
saving of the month.
Q.2 Mr. Akram sells his car at a loss of 6%. Calculate the selling price of the car if he
bought that car Rs.330,000.
Q.3. Mr. Ali bought a bike for Rs. 50,000. If he sells that bike at half price, calculate his

20
profit or loss percentage.
Q.4 Mr. Zain buys 200 kg of tomatoes from the vegetable market at a rate of Rs. 15 per
kilogram. If 10% of the tomatoes are found spoiled. Find the selling price per
kilogram so that he may get a profit of 20%.

1.6 Sets
This word is very commonly used in our daily life for example a set of glasses, a set of
books, sofa set, water set etc. The word “set” gives us a certain idea. In mathematics we
define this word as “a well-definedcollection of distinct objects”. When we say well defined
we mean that we have a collection of objects and looking at this collection we can easily say
that which object is included in the set and which one is not included. The second word
distinct means that the objects can be clearly and easily differentiated from each other in
set.These objects are called members or element of a set. Usually the name of a set is
represented by capital letters like A, B, C etc. and the members of a set are represented by
the small alphabets like a, b,c etc.

1.6.1 Set Description: Set description is very important to understand the nature of a set.
Three basic methods are used to describe a set. These methods include;

i. Descriptive Method: In this method a set is described by using words, that is we


write the given set in the form of a sentence for example the set of all double digit
positiveeven numbers.

ii. Tabular Method: This is mostly commonly used method. Here we enlist all the
members of the set and put a bracket at the start and at the end of the list. For
example, the set of all double digit positive even numbers in tabular method will be
represented by
= {10,12,14,16,…,98}

iii. Set-Builder Notation: This type of set description is very useful in the case
where the members of the set are too many. We take an arbitrary element of the set
and describe the set by giving an explanation of this member. For example, the
above set A will be represented by
= { :10 ≤ ≤ 98 ∧ }
Here symbol : means such that
and symbol ∧ means and

Self-Assessment Questions

Q. 1. Represent the following sets in the other forms of set description


i) The set of first ten natural numbers.
ii) The set of months of a year.
iii) The set of all primes between 2 and 22.

Now consider the tabular method where for example the set A is given as
= {10,12,14,16,….,98}

21
Clearly 12 is a member of this set A and we write it as: 12 ℎ (12 ) and 11 ∉ ℎ (11 )

1.6.2 Set Types


Basically sets are of two types, finite sets and infinite sets depending upon the number of
elements.

If a set has finite members, it is called finite set. Here in case of sets we start our counting
from zero instead of one. So a set with zero element is an important set and is known as
empty set and is represented by Φ ( ℎ ) or { }.On the other hand, the sets with infinite
number of elements are called infinite sets and are commonly represented in tabular form by
using … (means so on).

We can break a set into parts to get the smaller sets which are known as subsets. These are
also sets and have the same type of representation as that of sets. These subsets are of two
types, proper subsets and improper subsets, proper subsets are those sets which have strictly
lower membership than the father set for example if we have father set = {1,2,3,4,….,10}
then some proper subsets will be = { }, = {1}, = {1,2}, = {8,9,10}etc. improper subset of a
set is that set itself. So every set is an improper subset of itself. If we collect all the subsets
of a given set, we get the power set.

Example 1.Find the power set of = {2,4,6}

Solution: All the subsets of A are {}, {2}, {4}, {6}, {2, 4}, {2, 6}, {4, 6}, {2, 4, 6}
Therefore, the power set of A (denoted by P (A)) will be
( ) = {{ },{2},{4},{6},{2,4},{2,6},{4,6},{2,4,6}}

The strength of the power set for a given set can be calculated by using the following
formula.

Number of members of the ( ) = 2 where is the number of member of the set A We


have = 3 so its power set wil have 23 = 2 ∗ 2 ∗ 2 = 8 members as given above.

Self-Assessment Questions
Q. 1. Find the power set of the following sets:

i) = {1,2}
ii) ={ , , , }
iii) = {15}
1.6.3 Basic Operations on Sets
The basic operation of addition in set is different from that we have seen in the case of real
numbers. This operation is called union of two or more sets and it is defined as:

22
For two sets A and B, their union is another set which contains all the members of A and B,
written as ∪ .For example, if = {1,2,3,4} = {2,3 ,4,5}, then ∪ = {1,2,3,4,5}

The second operation is called intersection of two or more than two sets and it is defined as:

For two sets A and B, their intersection is another set which contains the common
members of A and B is written as ∩ . For example, if = {1,2,3,4} and = {2,3,4,5},
then ∩ = {2,3,4}

Some Important Definitions of Sets

a) Universal Set: It is the biggest from all the sets under consideration.
b) Empty Set: It is the set without any member.
c) Set Difference: If we subtract all the members of the set B from the set A we get the
difference of the sets A and B denoted by A – B.

For example, if = {1,2,3,4} = {2,3,4,5}, then − = {1}


d) Set Complement: If we subtract a set A form the universal set we get its
complement and it is denoted by .For example, if
= {1,2,3,4,….,10} = {1,3,5,7,8,9,10}
Then ( ) = −
= {1,2,3,4,….,10} − {1,3,5,7,8,9,10}
= {2,4,6}
1.6.4 Venn Diagram:
The term of Venn diagram was given by a mathematician John Venn and it is an
important way or method to describe and solve the different problems on sets. Venn diagram
is a special kind of diagram where we represent the given sets in the form of shapes and
diagrams and then looking at the desired regions of that diagram we get our answers. In
Venn diagram universal set is represented by the outer most shape that is the shape for
universal set is the circumference of all the shapes for other smaller sets. All other sets are
drawn accordingly.
Example 1: For two sets = {1,2,3,4} = {2,3,4,5} let we have =
{1,2,3,…,10}then represent ∪ ∩ using Venn diagram.

23
Solution:
U A B

6 2
7

1 3 5 8
4 9 10

⟹ ∪ = {1,2,3,4,5}
∩ = {2,3,4}

Example 2: In a class of 50 students suppose 23 students like Urdu, 20 students like Math,
21 students like English, 5 students like Urdu and English both, 6 students like English and
Math both and 5 students like Urdu and Math both. Find the number of students who like all
the three subjects Urdu, English and Math.

Solution: We rewrite all the given information

Total No. of students = 50

No. of students who like Urdu = 23

No. of students who like Math = 20

No. of students who like English = 21

No. of students who like both Urdu and English = 5 No.

of students who like both English and Math = 6 No. of

students who like both Urdu and Math = 5 No. of

students who like Urdu, English and Math = ? Let no. of

students liking Urdu, English and Math = x Then the

Venn diagram will be

24
Now we fill this diagram. Firstly, for Urdu we have to fill 4 parts. The part common to all
has entry x. Since 5 students like Urdu and English both so the second part has entry 5 – x.
Similarly, 5 students like Urdu and Math so 3 rd part has entry 5 – x. Also since 23 students
like Urdu so the 4thparthas entry

23 – ((5 – x) + x + (5 – x)) =23 – 5 + x – x – 5 + x


= 13 + x

6 students like English and Mathematics so 5th entry will be 6 – x,


21 students like English so 6th entry will be

21 – ((5 – x) + x + (6 – x)) = 21 – 5 + x – x – 6 + x
= 10 + x

20 students like Mathematics so 7th entry will be

20 – ((5 – x) + x + (6 – x)) = 20 – 5 + x – x – 6 + x
=9+x

Now since there are 50 students in the class so all the entries must sum up to 50. That is

13 + x + 10 + x + 9 + x + (5 – x) + (6 – x) + (5 – x) + x = 50

48 + x = 50

x = 50 – 48

x=2

Hence there are two students in the class who like all the three subjects Urdu,
Mathematics and English.

25
Self-AssessmentQuestions
Q.1 Suppose that we have the following sets.

= {1,2,3,4,5,…..,20}
={ ∶ 2}
={ ∶ 3}

Use Venn diagram to find ∪ , ∩ ,( ∪ ) ( ∩ )

Q.2 In a class, there are three types of students. Firstly, there are 10 students who like
cricket, secondly 25 students who like hockey, thirdly 6 students who like cricket and
hockey both. 1 student who does not like any game. Find the strength of this class (find the
universal set).

1.7 Distance Formula


This formula tells us the shortest distance between two points. Sometimes we also say that it
gives us the length of the line segment joining these two points. First of all, we represent a
point 1 ( 1 , 1)on the coordinate plane.

4 1 1( 1 , 1) 3

1 11

-x -4 -3 -2 -1 0 1 2 3

-1

-2

-3 -y

Similarly, if we have another point 2 ( 2 , 2 ) on the coordinate plane. Then the distance
between 1 ( 1 , 1 ) and 2 ( 2 , 2 ) is given by

26
= = ( − )2 + ( − 1)2
1 2 2 1 2

This is called the distance formula.

Example 1: Find the distance between the points (1, 4) and (4, 2)

Solution: Let 1 = (1, 4) and 2(4, 2)

We apply distance formula


= ( 2 − )2 + ( 2 − 1)2
1
= (4 − 1)2 + (2 − 4)2
= (3)2 + (−2)2 = √9 + 4 = √13
Therefore, the distance between (1, 4) and (4, 2) is √13 units.

Example 2: The distance between two points the1 = (4, ) and 2(1, 5) is 5 units. Find
values of .

Solution: 1 = (4, ) , 2 = (1 ,5), = =5


We know that the distance formula is given by

= ( − )2 + ( 2 − 1)2
2 1

5= (1 − 4)2 + (5 − )2

25 = (−3)2 + (5 − )2

25 = 9 + 25 + 2 − 10

2 − 10 + 9 = 0

2 −9 −5+9=0

( − 9) − 1( − 9) = 0 (

− 9)( − 1) = 0

−9 = 0 −1=0

=9 =1

Example 3: Prove that the Points 1 = (2, 5), 2 = (5, 2)and 3 = (−1, 2)are the
vertices of an isosles triangle.

27
Solution: We know that the isosceles triangle is a triangle whose two sides are equal. So
here we check that only two distances are equal.
First of all, we find the distance between 1and 2 and call it 1 :

1 = (5 − 2)2 + (2 − 5)2

1 = (3)2 + (−3)2 = √9 + 9 = √18

Then we find the distance between 1and 3and call it 2

= (−1 − 2)2 + (2 − 5)2


2

2 = (−3)2 + (−3)2

2 = √9 + 9 = √18

Since 1 = 2

⇒ The triangle 1 2 3 is an isosceles triangle.

Self-Assessment Questions:

Q. 1. Find the distance between the points 1 and 2with vertices.


i) (4, 6), (5, 5) ii) (4, 0), (6, 0) iii) (-1, 2), (1, 6)

Q. 2. Find the values of the variables m such that the distance between the points (2,5)
and ( ,−3) is 10 units.

1.8 Basic Geometric Shapes


Basic geometric shapes include triangles, quadrilaterals, circles, cones, cylinders, sphere,
cubes etc.

1.8.1 Triangle
This is the closed geometrical shape which has three corners and consists of three straight
lines, this is a 2-dimensional shape because we can draw it on the coordinate plane
(Cartesian plane). By looking at the three angels of a triangle we divide it into 6 types.
These types are given by:

a. Equilateral Triangle
It is a triangle whose all the three sides are equal for example:
C

3cm 3cm

26
A B
3cm
b. Isosceles Triangle
It is a triangle whose two sides are equal and the length of the third side is different for
example:
C

5cm 5cm A

2cm B

c. Scalene Triangle
It is a triangle whose all the three sides have different lengths for example;
C

4.5cm 5cm

A 2cm B

d. Acute Angle Triangle:


It is a triangle whose all the three angles are acute. This is all the three angles are less
then 90 in measurement for example:

C 500

600 700
A B

e. Right Angle Triangle:


It is a triangle whose one angle is right. That is this angle is 900measurement for
example:
C

500

900 400
A B

30
f. Obtuse Angle Triangle:
It is a triangle whose one angle is obtuse. That is this angle is more then 90 0 in
measurement for example:
C

450

1000 350 Self-


Assessment Questions: A B

Q.1 Find the types of the following triangles

i) ii) iii) 600


50 0 6cm
4cm 600
400
900
4cm

2 cm

5cm vi) 2cm

iv) 3cm v)
15 0 30 0
90 0 2cm
A B
4cm

Perimeter and Area of a Triangle


Let us consider a triangle ABC whose sides are represented as:

Perimeter is the distance covered if we walk along the three sides of a triangle once. i.e.
Perimeter of ∆ = + + units(∆ )
Area of the triangle ABC is shown as shaded region in the following diagram.
It is calculated by the following rule.
Let us denote by S the following sum.
C
S= + +
=
2
b a

Then area of ∆ = ( − )( − )( − )
A c B
Example 1: Find the perimeter and area of the ∆

Solution:First of all, we calculate S


C
+ + 8 + 10 + 6
= =
2 2

10cm 8cm

Perimeter of ∆ = 8 + 6 + 10 = 24
A 6cm B

Area of ∆ = ( − )( − )( − )
= 12(12 − 8)(12 − 10)(12 − 6)
= 12(4)(2)(6) = √574 = ±24

Therefore, Area of ∆ = 24 2

(Note since area is a nonnegative quantity so we take only the positive value of the square
root)
Right angle triangles are of special importance in Mathematics. We have certain values for
finding out three sides and areas.
We consider a Right Angle Triangle ABC.
C

b a

A c B

Here angle A is right. That is, it is of 900in measurement.


We give names to the sides of a right angle triangle in the following way:

= = ℎ ℎ
= = ℎ ℎ
33
= =ℎ ℎ ℎ
Then area of the right angle triangle = 1 ( x )
2
= 1( )
2

If we know two sides of a right angle triangle, we can find the third side using the
Pythagoras Theorem. This theorem gives us an interesting relation among the three sides of
a right angle triangle. This theorem was given by the well-known Greek
Mathematician Pythagoras and today it is remembered by his name.
For the above right angle triangle ABC, Pythagoras Theorem is given by
2 = 2 + 2

(ℎ )2 = ( )2 + ( )2

Example 1: Consider a right angle triangle XYZ. Find the length z.

y=8cm x=10cm

X z Y

Solution: We know that Pythagoras Theorem is given by


(ℎ )2 = ( )2 + ( )2
⇒ ( )2 = ( )2 + ( )2
⇒ (10)2 = (8)2 + ( )2
⇒ 100 = 64 + 2
⇒ 2 = 36
⇒ =6
⇒ ℎ =6

Example 2: What length of ladder is required to climb a wall of 12 feet high? If we have
to put the ladder 3 feet away from the base of wall.

12 feet wall
ladder

3 feet
34
Solution: Base = 3 feet, Altitude = 12 feet, Length of ladder = hypotenuse=?

35
Applying the Pythagoras theorem

(ℎ )2 = ( )2 + ( )2
⇒ (ℎ )2 = (12)2 + (3)2
⇒ (ℎ )2 = 144 + 9 = 155
⇒ℎ = 12.5
⇒Length of ladder will be 12.5 feet

Self-Assessment Questions
Q.1 Find the area of right angle triangle whose base is 4cm and altitude is 7 cm.
Q.2 For right angle triangles find out the missing sides:

i) x 12 ii) 4 x iii) 7cm 4cm

9 3 x

Q.3 A triangle with the sides 7cm, 8cm, 9cm is a right angle triangle or not.
Q.4 A triangle with sides 6cm, 10cm, 8cm is a right angle triangle or not.
Q.5 The length of one side of a square is 2cm. Find the length of the diagonal of this
square.

1.8.2 Quadrilaterals
This is a closed geometrical shape which has four corners and consists of four straight lines.
It is a two dimensional shape because we can draw it on the coordinate plane. By looking at
the sides and angles we can divide the quadrilaterals into many groups. Some of these are
given below.

a. Square: It is a quadrilateral who’s all the four sides are equal and all the four angles
are right. For example: D 3cm C

3cm 3cm

A 3cm B
Consider a Square ABCD then
Perimeter of square = 4 x length of one side
Area of square =( ℎ )2
b. Rectangle: It is a quadrilateral who’s all the four angles are right but only the
opposite sides are equal in length. For example:
D 5cm C

2cm 2cm

A 5cm B
Consider a rectangle ABCD then
Perimeter of rectangle = 2(length + width)
Area of rectangle = Length x width
c. Rhombus: It is a tilted square. That is if we tilt a square ABCD towards right side or
left side we get rhombus. So in rhombus all the 4 sides are equal in length. Opposite angles
are equal but not right.

D 3cm C 3cm

3cm A 3cm B

d. Parallelogram: It is a tilted rectangle. That it if we tilt a rectangle ABCD towards


right side or left side we get a parallelogram. So in parallelogram only opposite sides are
equal and opposite angles are equal in measurement but not right.

D 5cm C 2cm

2cm A 5cm B

1.8.3 Circle
It is a closed geometrical shape. It is the set of all the points in plane whose distance from a
fixed point is constant. This fixed point is called center of the circle. This is also a two-
dimensional shape because we can draw it on the coordinate plane.

The distance from the center of the circle O tothe point A on the circle is called radius of the
circle and doubling the radius of a circle we get the diameter of a circle.

37
We can calculate the perimeter of a circle by starting our motion from point A and moving
along the circle so that by completing one round we reach at the point A again and this
perimeter in the case of circle is called circumference and is given by the following
rule:
= 2
Here r is the radius of the circle whose circumference to be calculated and is an
irrational number whose approximate value is usually taken as 3.14 or 22.
7

All the region inside the circle including its boundary is called area of the circle and is given
by
= 2

Example 1. Find the circumference and area of a circle whose radius is 5cm.

Solution: = =5

= 2 = 2 x 3.14 x 5
= 31.40
Area of circle = 2 = 3.14 x 5 x 5
= 78.5cm2

Example 2. Find the area of a circle whose circumference is 24 .

Solution: Circumference of circle = 24


We know that circumference of circle of radius =2
Then 2 = 24
⇒ = 12

Therefore, area of circle with radius = 12 will be


Area = 2 = (12)(12) = 144 2

Self-Assessment Questions

Q.1 Find the circumference and area of a circle whose diameter is 12cm.
Q.2 The radius of the tire of a car is 10 inches. Find the area of the tire. Also if the tire
makes 60 revolutions in a minute. Find the distance covered by it in 10 minutes.
Q.3 Find the radius of a circle if its area is 49 2

1.8.4 Cylinder:
It is a geometrical shape whose base is a circle and height is h. It is a 3-dimensional shape.
So we can draw it in space. For example, we draw the base of cylinder which is a circle
parallel to the − and then height of the cylinder is in − .

38
a. Surface Area of Cylinder: Surface area of the cylinder is calculated by dividing it
into two parts. First part is taken as the two circle parts (base and upper) and a second
part is the curved part having height h of the cylinder. Then the total surface of the
cylinder of radius r and height h is given by:

Surface Area = 1+ 2
= ( 2 + 2 ) + (2 ℎ)
=2 2 +2 ℎ
= 2 ( + ℎ)square units

b. Volume of Cylinder: The volume or capacity of a cylinder is calculated by


multiplying the base circular area by the height of the cylinder. That is given by:
= 2xh
= 2 ℎ cubic units

Example 1: Find the surface area and capacity of a water tank whose base is circular with
radius 3 feet and its height is 4 feet.
Solution: First we find the surface area of two parts.
For circular part area = 2 2
= 2 (3)2 = 18
For curved part area = 2 × ℎ
= 2 (3) × 4 = 24
Therefore, total surface area of water tank is 42 square feet.
Now capacity of water tank = 2 ℎ
= (3)2(4)
= 36
Hence the capacity of the water tank is 36 cubic feet.
1.8.5 Circular Cone:
It is a geometrical shape whose base is a circle and its top is a single point with height h. It
is also a 3-dimensional shape so we can draw it in space. For example, a cone of base radius
r and height h is shown below.

39
h l

o r r
r

x
y

Here r is the radius of the circular base of cone and h is the actual height of the cone that is
starting from its vertex O. Note that l is the curved height of the cone.

a. Surface Area of Cone: We calculate the surface area of a cone by dividing it into two
parts.

First part is taken as the circular base of the core and second part is the curved part having l.
Then the total surface area of the cone is the sum of the two areas.

Surface Area of cone = 1 + 2


= 2+
So Surface Area = ( + )square units

b. Volume of Cone: The volume or capacity of a cone of a circular base radius r and height
h is calculated by the following formula.

Volume of cone = 1 2 cubic units.



3

Example 1: Find the volume of a cone whose circular base is of radius 10cm and its
height is 15cm

Solution: Radius of cone = = 10


Actual height of cone = ℎ = 15
Volume = 1 (10)2(15)
3
= 500
Therefore, volume of the cone is 500 3

1.8.6 Sphere:
Sphere is a 3-dimensional circular shape with radius . We can draw it in space. For
example, a sphere of radius is shown below.

40
r rr
r

Surfaces are of sphere = 4 2square units.

Surface Area of sphere = 4 2

where r is the radius.


Volume of sphere = 4 3cubic units.
3

Example 1: Find the surface area and volume of a sphere of radius 5cm.

Solution: Radius of sphere = =5


Surface area = 4 2
= 4 (5)2 = 100

So surface of sphere = 100 2


Volume of sphere = 4 3
3
4 500
= (5)3 = 3
3
Therefore, volume of sphere = 500 3
3

Self-Assessment Questions:
Q. 1. The volume of a cylinder of height 10cm is 160 3. Find the surface area of this
cylinder.

Q. 2. A cone has base radius 6cm and the height of the cone is 8cm. Calculate the total
surface area and volume of the cone.

Q. 3. Calculate the leather required to manufacture a football of radius 6 inches.

UNIT-2

41
LINEAR AND
QUADRATIC EQUATIONS

Written By: Dr. Nasir Rehman


Reviewed By: Dr. Baber Ahmed

42
CONTENTS
Page No
Introduction . ................................................................................................................ 39
Objectives ................................................................................................................. 39

2.1. Linear Equations ......................................................................................................... .40

2.1.1 Linear Equations in One Variable ..................................................................... .40


2.1.2 Linear Equations in Two Variables .................................................................. .41
i) Substitution Method ................................................................................ .43
ii) Elimination Method ................................................................................. .43
iii) Graphical Method .................................................................................... .44
iv) Matrices Method ...................................................................................... .45

2.2 Quadratic Equations ....................................................................................................... .47


2.2.1 Factorization Method ........................................................................................ .47
2.2.2 Completing Square Method............................................................................... .48
2.2.3 Quadratic Formula............................................................................................. .50
2.2.4 Applications of Quadratic Equations ................................................................ .53

43
INTRODUCTION
In our business when we make some calculations, when we deliver different types of goods,
when we pay our workers etc., we have been using equations. Equations are actually all the
mathematical statements where we see the symbol “=”. Equations are of many types but in
this chapter we will explore only linear and quadratic equations. These equations involve
one or more variables. And solution of an equation or equations means that we have to find
the exact values of these variables. Equations play an important role to solve the different
problems of Business, Economics, Banking, Stock Market, Agriculture etc.

OBJECTIVES
At the end of unit (2), the students will be able to:

1) differentiate between linear and quadratic equations.

2) solve linear and quadratic equations using different methods.

3) apply these equations into practical problems.

4) model mathematical equations in different situations.

44
2.1 Linear Equations
Linear equations are the algebraic equations which involve variables and constants and the
variables involved have powerexactly equal to 1. 1 + 2 = 0, 2 1 + 1 = −3, 2 1 +
4 1 = 0 etc.
When it is understood that thepower of all the variables involved is 1 we omit it and write
the linear equation in the form + 2 = 0, 2 + = −3, 2 + 4 = 0 .

2.1.1 Linear Equations in One Variable


These are the linear equations which involve only one variable.

Example 1: + 2 = 0 is a linear equation in one variable. The solution of linear


equation means we have to find the value of the variable involved. For the above example
we have to find to solve it i.e. + 2 = 0, subtracting -2 on both sides gives + 2 − 2 =
−2 => = −2

So, = −2 is the solution of this linear equation. Now we represent this solution on the
coordinate plane.

On the coordinate plane, = −2 is represented by the line shown. This is a vertical line.
Actually all the linear equations in one variable represent vertical lines. But what
happens if we consider a linear equation is one variable . This we consider in the next
example.

Example 2: Let us suppose that we have the linear equation:


2 −5=0
And we want to solve it.
2 −5=0
subtracting -5 on both sides gives
2 −5+5=5
45
• 2 =5

46
2 5
=
2 2
• 5
=
2
So = 5 is the solution of this linear equation. Now we represent this solution on the
2
coordinate plane

On the coordinate plane = 5 is represented by the line shown. This is a horizontal line.
2
Actually all the linear equations in one variable y represent horizontal lines.

Self-Assessment Question
Q.1 Solve the following linear equations in one variable and also represent them on the
coordinate plane
i) 2 −1=0
ii) −3 =1
2
iii) 2 =0
iv) 5 + 10 = 0

2.1.2 Linear Equations in Two Variables


These are the linear equations which involve two variables. For example, − = 2 is a
linear equation in two variables . If we take = 2, = 0, again the given
linear equation is satisfied. In the same way we can have infinity many pairs of numbers
satisfying − = 2. So we conclude that unlike the linear equations in one variable which
have a unique solution, the linear equation in two variables does not have a unique solution.

47
Example 3: Represent the linear equation 2 − = 4 on the coordinate plane. Solution:
To draw this linear equation, we make a table of values and satisfying 2 − = 4 and
then draw it on the coordinate plane, since

2 − =42
= +4
1
= ( + 4) 2

x 0 1 2 −1 −2 3
y 2 2.5 3 1.5 1 3.5

From the above diagram it is clear that linear equations in two variables are also straight
lines making a certain inclination along the − . Further the slution set of these types
of linear equations consists of infinitely many pairs of real numbers.

Self-Assessment Questions
Q. 1. Draw the linear equation 3 − = 6
Q. 2. Draw the linear equation = 3 + 2

To get a unique solution for the case of linear equations in two variables we need to consider
two different linear equations in two variables. Such a pair of linear equations is known as
simultaneous linear equations. To solve the simultaneous linear equations four methods are
commonly used. These methods include:

1) Substitution Method
2) Elimination Method
3) Graphical Method
4) Matrices Formation Method
48
We explain all these methods by taking a particular example.

Example 4: Solve the simultaneous linear equations:


2 − =4
+2 =7
Solution:
i) Substitution Method
Consider the system of simultaneous equations.

2 − = 4 ……………………... (1)
+ 2 = 7 ……………………... (2)
We solve the first equation for or and substitute its value in the second equation.

From equation (1) we can write:


= 2 − 4 ………………………(3)

Substituting = 2 − 4 in equation (2) we get:


+ 2(2 − 4) = 7
+4 −8 =7
5 = 15 => = 3

Now substituting = 3 in equation (3) we have


= 2(3) − 4 => = 2

Therefore, the solution set is = 3, = 2.

ii) Elimination Method


Consider the system of simultaneous equations:
2 − = 4 ……………………... (1)
+ 2 = 7 ……………………... (2)
In this method we try to eliminate anyone variable or by making their coefficient same and
then by subtracting or adding to eliminate them.

Multiplying equation (1) by 2we get:


4 − 2 = 8 ……………………... (3)

Now adding equations (2) and (3) we have:


( + 2 ) + (4 − 2 ) = 7 + 8
⟹ 5 = 15 =>
=3

Substitute = 3 in equation (3) we get


4(3) − 2 = 8
−2 = 8 − 12

49
−2 = −4 => = 2
Therefore = 3, = 2 is the solution set.

iii) Graphical Method


Consider the system of linear equations:

2 − =4 ……………………... (1)
+2 =7 ……………………... (2)

Here we draw the graphs of both the linear equations on the same coordinate plane.

Consider the first linear equation 2 − =4

We make a table of values


equation.
x 0 1 2 3
y −4 −2 0 2
of satisfying above

2 − =4

⟹ =2 −4

Consider the second linear equation:

• + 2 = 7

Again we make a table of satisfying + 2 = 7 That

is +2 =7

⇒ 2 =7−

1
⇒ = (7 − ) 2

x 0 1 2 3
y 3.5 3 2.5 2

With the help of these two tables we draw the simultaneous linear equations on the
coordinate plane

50
The point of intersects of these lines is called the solution set of the given simultaneous
linear equations for example in our case it is = 3 = 2.

iv) Matrices Method


Consider the system of simultaneous equations:

2 − = 4 ……………………... (1)
+ 2 = 7 ……………………... (2)

Writing these equations in Matrix form:


2 −1 = 4 1
2 7

We call = 2 −1 = = 4
1 2 7
That is = => = −1

So we need to find −1

The determinant of the matrix is | | = 2 ∗ 2—1 ∗ 1 = 4 + 1 = 5 ≠ 0


Therefore −1 = 1
| |
1 2 −1
=
5 −1 2
Now = −1 = 1 2 −1 4 5
−1 2 7

51
1 2x4 + 1x7 1 15 3
= = =
5 −1 x 4 + 2x7 5 10 2

So the solution set of the given simultaneous linear equations is = 3 = 2.

Self-Assessment Questions
Q. 1. Solve the following system of linear equations in two variables.
i) 3+=2
− 2=3 Using Substitution Method.

ii) 4 − = 11 3 + 4 = −6 Using Elimination


Method.

iii) −2=2
2−=7 Using Graphical Method.

iv) 2 + 3 = −1
−2 =3 Using Matrices Method.
Example 5: Consider a two-digit number. The sum of its digits is 6. If the number is
reversed (changing the unit digit with the tens digit). The new number increases by 18. Find
the actual number.
Solution: We suppose that the unit digit of the number is and tens digit is then the number
will be written as + 10

If the digits are reversed the new number will be 10 + ,

According to the condition of the question:


10 + − ( + 10 ) = 18
⇒ 10 + − − 10 = 18
⇒ 9 − 9 = 18
⇒ − = 2………….. (1)

Also since the sum of two digits of the number is 6


that is + = 6……………(2) So
we have a system of simultaneous linear equations
− = 2 ……………. (1)
+ = 6 …………… (2)

We solve by Elimination method.

Adding both equations we get


( − ) + ( + ) = 2 + 6 =>
2=8

52
=> =4
Putting = 4 in (2)
4+=6
=> =2

Therefore, the actual number is 4 + 10(2) = 4 + 20 = 24

Self-Assessment Question
Q.1 Three sides of an equilateral triangle are given by2 + , − + 5, 3 + 2 −
3. Find the exact length of each side.

2.2 Quadratic Equations


An equation of the form 2 + + = 0 where , and are constants and ≠ 0, is called a quadratic
equation in variable . Because the case = 0 corresponds to linear equations in one variable.
Here the highest coefficient of the variable is 2 so these are also called second degree
equations. In practical situations many objects are not related with each other linearly so
there we need quadratic and higher order equations. For example, the motion of a freely
falling object can be described by a quadratic equation. Let us represent its height by ℎ and
time by . Then the height of that object at any time is given by:
ℎ( ) = 100 + 2 − 2

Solution of Quadratic Equation


By solution of the quadratic equation we mean finding the values of the variable
involved. There are three well known methods to solve the quadratic equations. These
methods include.

2.1 Solving Quadratic Equations by Factorization.


2.2 Solving Quadratic Equations by Completing Square.
2.3 Solving Quadratic Equations by Quadratic Formula.

2.2.1 Factorization Method


In this method we consider a quadratic equation 2 + + = 0, ≠ 0 and we factorize it into
two linear parts ( + ) and ( + ), ≠ 0 and ≠ 0 such that

( + )( + )= 2+ + =0
⇒ ( + )( + ) = 0
⇒ + =0 + =0
Example 1: Solve the following quadratic equation by Factorization method
5 2 − 8 = 0
Solution: 5 2 − 8 = 0 Taking
common from both the terms we get
(5 − 8) = 0

53
⟹ =0 5 −8=0
Therefore = 0 = 8 is the solution set.
5

54
Example 2: Solve the quadratic equation by Factorization.
5 2 − 8 − 13 = 0
Solution: 5 2 − 8 − 13 = 0
Here we break the middle term −8 into two parts in such a way that if we add both parts we
get −8 and if we multiply both parts we get the product of first and third term i.e.
(5 2)(−13) = −65 2 .

So the required parts will be (5 ) and (-13 ) since


(5 ) + (−13 ) = −8 And
(5 )x(−13 ) = −65 2 .

Therefore, we can write the given quadratic equation in the form


5 2 + 5 − 13 − 13 = 0
⇒ 5 ( + 1) − 13( + 1) = 0
⇒ ( + 1)(5 − 13) = 0
⇒ + 1 = 0 5 − 13 = 0
= −1 = 13 is the solution set. 5

Example 3: Solve the quadratic equation by Factorization.


( − 2)( − 3) = 2

Solution:( − 2)( − 3) = 2 First


we simplify
2 −2 −3 +6=22
−5 −4=0

Now we break the middle term (−5 ) into two parts (−4 ) and (−1 ) then
we have,
2−4 − +4 =0
⇒ ( − 4) − 1( − 4) = 0
⇒ ( − 4)( − 1) = 0
⇒ −4=0 −1=0

⇒ = 4, = 1 is the solution set.

Self-Assessment Questions
Q. 1. Solve the following quadratic equations using Factorization method.
i) 22 = 6 ii) 2 + − 6 = 0 iii) ( − 2)( + 3) = 6

2.2.2 Completing Square Method


When it is difficult to factorize a give quadratic equation we move forward to the
completing square method to solve the quadratic equation. We have the quadratic
equation 2 + + = 0, ≠ 0.

55
Dividing by a we get 2 + + =0

We can further modify it as ( + )2 = , where are some real numbers. We

consider some examples to justify and explain this method.

Example 1: Solve the quadratic equation using completing square method.


2 +4 +2=0

Solution: We write the given equation in the form :


( )2 + 2( )(2) + 2 = 0
Adding (2)2 on both sides to complete the square:
( )2 + 2( )(2) + (2)2 + 2 = (2)2
⇒ ( + 2)2 + 2 = 4
⇒ ( + 2)2 = 2
⇒ + 2 = ±√2
⇒ + 2 = √2 + 2 = −√2
⇒ = √2 − 2 = −√2 − 2 is the solution set.

Example 2: Solve the quadratic equation using completing squares method:


5 −4− 2 =0
Solution: 5 − 4 − 2 = 0
First we write it in standard from:
2−5 +4=0
5
⇒ ( )2 − 2( ) +4=0
2
2
So adding 5 on both sides we can complete the square
2
2 5 5 2 5 2
( ) − 2( ) + +4=
2 2 2
5 2 5 2
⇒ − = −4
2 25
2 9
⇒ − =
2 4
5 3
⇒ − =±
2 2

5 3 5 3
⇒ − = − =−
2 2 2 2

4 5 3 5

56
⇒ = + =− +
2 2 2 2

⇒ =4 = 1 is the solution set.

Example 3: Solve the quadratic equation using competing square method 2 +2 +


7 = 0 Solution: 2 +2 +7=0
( )2 + 2( )(1) + 7 = 0
So adding (1)2 on the both sides to complete the square
( )2 + 2( )(1) + (1)2 + 7 = (1)2

⇒ ( + 1)2 = 1 − 7
⇒ ( + 1)2 = −6

Now if we take square root on both sides, we get √−6 which is not a real number. So in this
case we say that the given quadratic equation has not real roots and thus no real solution
possible in this case.

Self-Assessment Questions
Q. 1. Solve the following equations using completing square method:
i) 2 −2 −8=0
ii) 2 + −6 =0
iii) y 2 + 20y − 100 = 0

Quadratic formula can be used to solve any quadratic equation. First of all, we formulate
this formula using the completing square method of quadratic equations:

2.2.3 Quadratic Formula


We know that the general quadratic equation in one variable is given as:
2+ + = 0, ≠0
Dividing both sides by we get:
2 + + =0
Now we complete the square by writing above equation in the form:

( )2 + 2( ) + =0
2
2
Adding on both sides we get
2
2 2(
)2 + 2( ) + + =
2 2 2
2 2
⇒ + =− +
57
2 4 2
2 −4 + 2
⇒ + =
2 4 2
2 2−4
⇒ + =
2 4 2

2 −4 )
⇒ + =± 2
2

2 −4 )
⇒ =− ±
2 2
− ±√ 2−4
⇒ =
2

Therefore, for the above quadratic equation we have the solution set
− ±√ 2−4
=
2
The nature of the solution set depends on the term 2 − 4 .

If 2 −4 < 0 i.e. It is negative we can’t take square root so we have no real solution set.

If 2 −4 = 0 then we have =− ±0 i.e. we have two equal values for . We call this
2
solution set as equal and read.

If 2 − 4 > 0 then √ 2 − 4 will be two distinct real numbers and so the solution set
will be two distinct real values of .

Example 1. Solve the quadratic equation using quadratic formula.


5 2 − 8 − 13 = 0

Solution: 5 2 − 8 − 13 = 0

Comparing it with the standard quadratic equation: 2


+ + = 0, we have
a= 5, = −8, = −13
Applying quadratic formula =− ±√ 2−4 we get
2

58
−(−8) ± (−8)2 − 4(5)(−13)
=
2(5)
8 ± √64 + 260
⇒ =
10
8 ± √324
⇒ =
10
8 ± 18
⇒ =
10
8 + 18 8 − 18
⇒ = =
10 10
⇒ = 13 = −1 is the solution set. 5

Example 2: Solve the quadratic equation using quadratic formula:


4 2 − 12 + 9 = 0

Solution: 4 2 − 12 + 9 = 0
Comparing it with standard quadratic equations 2 + + = 0, we get:
= 4, = −12, = 9
Now applying quadratic formula =− ±√ 2−4 , to get:
2

−(−12) ± (−12)2 − 4(4)(9)


=
2(4)
12 ± √144 − 144
⇒ =
8
12 ± 0
⇒ =
8
⇒ = 12+0 and = 12−0
8 8

⇒ =3 = 3is the solution set.


2 2

Example 3: Solve the quadratic equation using Quadratic formula.


2 + 2 + 15 = 0

59
Solution: 2 + 2 + 15 = 0
From here we have
= 1, = 2, = 15
We apply the quadratic formula =− ±√ 2−4 to get
2
−2 ± (2)2 − 4(1)(15)
=
2(1)
−2 ± √4 − 60
⇒ =
2
−2 ± √−56
⇒ =
2
−2 ± 2√−14
⇒ =
2
−1 ± √−14
⇒ =
2
Since √−14 is a real number. So there does not exist a real solution set for the given
quadratic equation.

60
Self-Assessment Question
Q. 1. Solve the following quadratic equations using the Quadratic formula.
i) 2 −5 +4=0
ii) 4 2 − 12 + 9 = 0
iii) 2 + 15 + 36 = 0
iv) 2 +5 +7=0

2.2.4 Applications of Quadratic Equations


Example 1: Consider a rectangle whose area is 20 2 and perimeter is18 . Find the length
and breadth of this rectangle.

Solution: Let the length and breadth of rectangle are and respectively.
We know that for a rectangle =
Perimeter of rectangle = 2( + )
Therefore, we have
2( + ) = 18 ……………………… (1)
= 20 …………………………….. (2)
From equation (1)
+ =9
= 9 − ………………… (3)
Putting the value of y in equation (2) we get:
(9 − ) = 20
⇒ 9 − 2 = 20
⇒ 2 − 9 + 20 = 0
⇒ 2 − 4 − 5 + 20 = 0
⇒ ( − 4) − 5( − 4) = 0
⇒ ( − 5)( − 4) = 0
⇒ −5 =0 −4 =0
⇒ =5 =4
So from equation (3) we have
=9−5=4

=9−4=5
Therefore, length of rectangle = = 5cm
And the breadth of rectangle = = 4cm

Example 2: The product of two positive consecutive odd numbers is given by 63. Find both
the numbers.
Solution: Let the first odd number =
61
Then next consecutive off number will be = + 2

According to the given condition the product is given by:


( + 2) = 63
⇒ 2 + 2 = 63
⇒ 2 + 2 − 63 = 0
⇒ 2 + 2( )(1) − 63 = 0
⇒ 2 + 2( )(1) + (1)2 − 63 = 0
⇒ ( + 1)2 = 63 + 1 = 64
⇒ + 1 = ±8
=8−1 = −8 − 1
⇒ =7 = −9
Since we have
to find positive
odd numbers we
take = 7

Therefore, first odd number is = 7 And

other odd number is = 7 + 2 = 9

Example 3: A ball is dropped from a height of 100 m. If its motion is described by the
following quadratic equation.
ℎ( ) = 100 + 2 − 2

Where h represents the height (meters) of the ball at any time t (seconds)then find the time
taken by the ball to hit the ground.

Solution: At any time t, the height of the ball is given by:


ℎ( ) = 100 + 2 − 2……………………….. (1)

If = 0 i.e. at the start


ℎ(0) = 100

We have to find the t such that ℎ( ) = 0 (height is zero when it touches the ground) therefore
equation (1) becomes
0 = 100 + 2 − 2

⇒ 2 − 2 − 100 = 0

62
Applying Quadratic formula, we have:
−(−2) ± (−2)2 − 4(1)(−100)
=
2(1)

2 ± √404 2 ± 2√101
⇒ = =
2 2

⇒ = 1 ± √101

⇒ = 1 + √101 = 1 − √101

Since time is positive so we

neglect negative value

= 1 + 10.05

So the time taken by the ball to reach at the ground is 11.05 seconds.

63
UNIT-3

MATRICES AND DETERMINANTS

Written By: Ms Fouzia Rehman


Reviewed By: Dr. Baber Ahmed

64
CONTENTS

Page No

Introduction ............................................................................................................ .58

Objectives ............................................................................................................ .58

3.1 Matrix ............................................................................................................. 59


3.1.1 Order of a Matrix ........................................................................................ .59
3.1.2 Kinds of Matrices ........................................................................................ .60

3.2 Addition and Subtraction of Matrices: ................................................................... .62

3.3 Multiplication of Matrices...................................................................................... .64

3.4 Determinant of a 2 × 2 Matrix................................................................................ .66


3.4.1 Singular and Non-Singular Matrices ............................................................ .66
3.4.2 Adjoint of a 2 × 2 Matrix ............................................................................. .66
3.4.3 Inverse of a 2 × 2 Matrix .............................................................................. .67
3.4.4 Inverse of a Matrix using Adjoint: ............................................................... .67

3.5 Solution of Simultaneous Linear Equations by Using Matrices: ........................... .68

3.6 Minor and Cofactor of an Element of a Matrix ...................................................... .72


3.6.1 Determinant of a Square Matrix of order 3: ................................................. .73
3.6.2 Adjoint of a Matrix of order 3: ..................................................................... .74
3.6.3 Inverse of a Matrix of order 3: ..................................................................... .74

3.7 System of Linear Equations in 3 Variables: ........................................................... .76


3.7.1 Homogenous Linear Equations: ................................................................... .76
3.7.2 Non-Homogenous Linear Equations: ........................................................... .78
a) Use of Matrices: ................................................................................. .79
b) Cramer’s Rule:.................................................................................... .80

65
INTRODUCTION
The idea of Matrices was first presented in nineteenth century by Arthur Cayley, a
famous Mathematician who first gave “Theory of Matrices” in 1857 and applied them in
linear transformations. Matrices and Determinant sare widely applied in the field of
Mathematics, Physics, Statistics, Electronics and several other disciplines.

OBJECTIVES
At the end of unit (4), the readers will be able to

1) define Matrix and its various kinds

2) apply basics operations of addition, subtraction and Multiplication on Matrices

3) define and evaluate Determinant of a Matrix.

4) calculate Inverse of a nonsingular Matrix.

5) solve a system of linear equations and related real life problems.

66
3.1 Matrix
A matrix is an array of numbers, enclosed by square brackets [ ], which are arranged into
rows and columns. Each matrix is presented in horizontal and vertical lines. The
horizontal lines are called Rows while vertical lines are known as Columns. Usually capital
letters A, B, C ….. etc. are used to denote matrices.

In MatrixA, C1, C2 and C3 are vertical lines and hence called columns.Thus, Matrix A has
three columns.

In Matrix B, R1, R2 and R3 are horizontal lines and hence called rows. Thus, Matrix B has
three rows.

3.1.1 Order of a Matrix


The order of a Matrix is described by the number of rows and columns in a matrix. If a
Matrix A has “m” rows and “n” columns then A is of order, “ m-by-n” or “m × n”

1 3 5 4
Example 1: Find the order of D = 3 1 2 6
4 5 0 2

Solution: The matrix D is of order 3-by-4 or 3 × 4, since it has three rows and four columns.

0 5 3
Example 2: Find the order of M =
2 9 6
Solution: The matrix M is or order 2 × 3, since it has two rows and three columns.

67
3.1.2 Kinds of Matrices

a) Row Matrix
A matrix is said to be a row matrix if it has only one row.
e.g. A =[3] B=[1 4 5] C =[−1 1]
b) Column Matrix
A matrix is said to be a column matrix if has only one column.

−1
1
e.g.A = [3] = C= 0
4
1
c) Rectangular Matrix
A matrix is said to be a rectangular matrix if the number of rows and number of columns are
not equal.

1 5
2 4 8
e.g. A = 3 4 B=
1 4 0
0 2

d) Square Matrix
A matrix is said to be a square matrix if the number of rows and number of columns are
equal.
1 4 5
1 0
e.g. A = B = 8 6 1 C = [3]
0 1
2 5 0

e) Null or Zero Matrix


A matrix is said to be a null or zero matrix if its all the elements is zero.

0 0 0
0 0
e.g. A = B= 0 0 0 C = [0 0 0]
0 0
0 0 0
f) Diagonal Matrix
A square matrix with at least one of its entries in the diagonal is non zero and all other
entries are zero is called diagonal matrix.

1 0 0 0 0 0 0 0 0
68
e.g. A= 0 5 0 B= 0 3 0 C= 0 0 0
0 0 7 0 0 2 0 0 9
g) Identity Matrix
A diagonal matrix having all the entries in the diagonal are one is called identity matrix.
1 0 0
1 0
e.g. A= 0 1 0 B=
0 1
0 0 1

h) Transpose of a Matrix
The transpose of a matrix is obtained by turning all the rows of a matrix into columns and
vice versa. If A is a matrix then its transpose is denoted by At.

1 4 5 1 8 2 2 1
2 4 8
e.g.i) If = 8 6 1 , then = 4 6 5 ii) if = , then = 4 4
1 4 0
3 5 0 5 1 0 8 0

Self-Assessment Questions
1
3 7 1
Q.1) Find the order of : i) F = [8] ii) N = iii) G = 2
4 4 0
3
Q.2) Find the transpose of each of the following matrices:

−1 0 0 6 −6
i) A = 6 9 −2 ii) B = 4 9 iii) C = [0 0 2]
−4 5 1 −2 0

−8 0 7
Q.3) If A = 5 2 −1 , then verify that (At)t = A
−4 0 3

3.2 Addition and Subtraction of Matrices:


3.2.1 Addition of Matrices
Two matrices A and B are suitable for addition, if they have the same order.

69
0 1 3 7 5 2
e.g. A = and B =
2 6 5 1 −5 0

The order of matrix A is 2 × 3 and order of matrix B is also 2 × 3. Therefore, matrix A and
B are conformable for addition. The addition is obtained by adding the elements of matrix A
to the corresponding elements of matrix B.

0 1 3 7 5 2 0+7 1+5 3+2 7 6 5


e.g. A + B = + = =
2 6 5 1 −5 0 2+1 6 + (−5) 5+0 3 1 5

Example: Two schools conducted their Summer English Camps in April. The table shows
the number of students who applied for the summer camps from each school. Find the total
number of students applied for the summer camp.

Solution:
No. of students applied from school – I in matrix form is : 25 28 20
23 26 22

No. of students applied from school – II in matrix form is: 26 24 23


21 28 25

Total number of students applied for the summer camp = ?


25 28 20 26 24 23 25 + 26
28 + 24 20 + 23 26
Total number of students = + =
23 26 22 21 28 25 23 + 21 + 28 22 + 25

51 52 43 47
=
44 54

3.2.2 Subtraction of Matrices


Two matrices A and B are conformable for subtraction, if they have same order.

70
7 5 −2 6 3 −1
e.g. A = 4 −8 0 and B = −2 −8 3
1 −1 9 5 4 1

Since both the matrices have the same order i.e. 3 × 3, therefore they are suitable for
subtraction and is written as A-B.
7 5 −2 6 −8 3 −1 7−6
5−3 −2 − (−1)
e.g. A – B = 4 0 - −2 −1 9 −8 3 = 4 − (−2) −8 − (−8) 0−3
1 5 4 1 1−5
1 − (−4) 9−1
1 2 −1

= 6 −4 0 −3
−5 8

Example: The number of books, journals and CDs available in Library A and Library B
are represented by two matrices. How many of each of these items is more in Library A?

3081 2948 3081 − 2948 133


Solution: Library A – Library B = 70 - 33 = 70 − 33 = 37
251 146 251 − 146 105

Multiplication of a Matrix by a Real number


A matrix A can be multiplied with any real number k. This can be done by multiplying each
element of matrix A with k and is written as kA.

−1 −2 0
71
Let A = 4 3 1 be a matrix of order 3 × 3 and k = -3 be a real number, then
3 −1 1

kA = (-3) A

−1 −2 0 (−3)(−1) (−3)(−2) (−3)(0) 3 6 0


= (-3) 4 3 1 = (−3)(4) (−3)(3) (−3)(1) = −12 −9 −3
3 −1 1 (−3)(3) (−3)(−1) (−3)(1) −9 3 −3

72
3.3 Multiplication of Matrices
Two matrices A and B are conformable for multiplication if the number of columns of the
first matrix and the number of rows of the second matrix are same.

−1 0 1 3 −3
e.g. If A = 2 5 3 and B = 1 2
2 −2 0 0 4

The order of matrix A is 3-by-3 and the order of matrix B is 3-by-2. Since the columns of
matrix A is equal to the rows of matrix B. Therefore multiplication AB is possible and the
resulting matrix must be of order 3-by-2 but BA is not possible in this case. If the number of
columns of the first matrix does not match the number of rows of the second matrix then the
product cannot be defined.

Example:
On two days, a store sold the following amounts of pencils, erasers and sharpeners.
Pencils Erasers Sharpeners
Monday 40 8 6
Tuesday 50 11 5

The price of each pencil, eraser and sharpner is $ 0.3, $ 0.2 and $3 respectively. How much
was made each day?

73
(40)(0.3) + (8)(0.2) + (6)(3) 12 + 1.6 + 18 31.6
== = (50)(0.3) + (11)(0.2) + (5)(3) 15
+ 2.2 + 15 32.2
$31.6 on first day and $32.2 on 2nd day.

Self Assessment Questions


1 2 3 4 −1 1
Q.1 If A = −1 −2 3 and B= 1 4 3 , then find
3 4 5 3 −3 −2

i) 4A – 3B ii) A + 2 (B – A) iii) 2At + 3Bt iv) AB

also verify that:


i) (A + B)t = At + Bt ii) A + (B + A) = 2A + B iii) A(B - C) = AB - AC
iv) (AB)t = Bt At
Q.2 Ms. Juliet and Mr. John are two salespeople for a new car agency that sells only two
models. August was the last month for this year’s models and next year’s models were
introduced in September. Gross dollar sales for each month are given in the following
matrices.

i) What were the combined dollar sales in August and September for each salesperson and
each model? (Hint: find A+B)

ii) What was the increase in dollar sale from August to September? ( Hint: find B – A)
Q.3 Attendance for the first three football games of the season is described in the table.
Adult tickets sold for $3.00, student tickets sold for $1.5. Use matrix multiplication to
find the revenue for each.
Adults Students
Game 1 340 180
Game 2 250 195
Game 3 300 220
74
3.4 Determinant of a 2 × 2 Matrix
The determinant of a matrix A of order 2-b-2 is denoted by det A or |A|.

2 5 2 5
e.g. If A = , then det A or |A| = = (2)(3) – (-1)(5) = 6 + 5 = 11
−1 3 −1 3

|A| = 11

If B = 4 7 , then det B or |B| = 4 7 = (4)(9) – (3)(7) = 36 – 21 = 15


3 9 3 9

|B| = 15

3.4.1 Singular and Non-Singular Matrices


A square matrix A is said to be called a singular matrix, if |A| = 0 otherwise it is called non
singular matrix.

e.g. if A = 2 5 , then det A or |A| = 2 5 = (2)(3) – (-1)(5) = 6 + 5 = 11


−1 3 −1 3

|A| = 11≠ 0, therefore matrix A is a non-singular matrix.

4 8 4 8
If B = , then det B or |B| = = (4)(4) – (2)(8) = 16 – 16 = 0
2 4 2 4

|B| = 0, therefore matrix B is a singular matrix.

3.4.2 Adjoint of a 2 × 2 Matrix


The adjoint of a matrix A is obtained by interchanging the first and fourth entries of the
matrix and changing the signs of the rest of the entries and is denoted as Adj A.

4 5 3 −5
e.g. A = , then Adj A =
1 3 −1 4

75
3.4.3 Inverse of a 2 × 2 Matrix

Two matrices A and B are said to be multiplicative inverse of each other if AB = BA = I.


The inverse of a matrix A is written as A-1. Therefore AA-1 = A-1A= I. The inverse of a
matrix is feasible only if matrix is non-singular.
3.4.4 Inverse of a Matrix using Adjoint:

For a non-singular matrix A, the inverse of A can be obtained using adjoint. i.e.

−1 =
| |
Example: Find A-1 if A = 5 1 and verify that AA-1 = A-1A = I ( where I is identity
4 2
matrix i.e. I = 1 0 )
0 1
Solution:
5 1
|A| =
2 4
= (5)(2) – (4)(1) => 10 – 4 = 6 ≠ 0

As |A| ≠ 0, therefore we can find A-1


−1 =
| |

2 −1
Adj A =
−4 5

2 −1 2 −1
A-1= −4 5 = 6 6
6 −4 5
6 6
Now

2 −1
5 1
AA-1 = 6 6
4 2 −4 5
6 6

76
(6) (2) + (1) (−4) (5) (−1) + (1) ( 5)
= 6 6 6 6
(4) ( 2) + (2) (−4) (4) (−1) + (2) (5) 6
6 6 6
( 5
= 10/6 ) + (−4/6) (−5/6) + ( )
6
(8/6) + (−8/6) (−4/6) + (10/6)

10/6 – 4/6 = −5/6 + 5/6 (10 – 4)/ 6 0


8/6 – 8/6 = (−4 + 10)/ 6
−4/6 + 10/6 0

6/6 0 1 0
= = =I
0 6/6 0 1

2 −1 2 −1 2 −1
5 1 ( ) (5) + ( ) (4) ( ) (1) + ( ) (2)
A-1A = 6 6 = 6 6 6 6
−4 5 −4 5 −4 5
4 2 ( ) (5) + ( ) (4) ( ) (1) + ( ) (2)
6 6 6 6 6 6

10 4 2 2
= 6 −6 6 − 6 = 6/6 0 = 1 0 −20
+ 20 −4 + 10 0 6/6 0 1
6 6 6 6
Hence proved AA-1 = A-1A = I

3.5 Solution of Simultaneous Linear Equations by Using Matrices:


Consider the system of linear equations:
11 1 + 12 2 = 1

77
21 1 + 22 2 = 2 This

system of equations can be written in matrix form i.e.

i.e.
AX=B (i)
A-1 exists when |A| ≠ 0, so (i) gives by multiplying A-1 on both sides
A-1(AX) = A-1B
Since matrix multiplication is associative, therefore,
(A-1A)X = A-1B
• I2X = A-1B (AA-1 = I2, Identity matrix)

• X = A-1B

22 − 12

12 = |1| − 21 11 12

22 1 − 12 2

= 1 22 1 − 12 2 = | |
| | − 21 1 11 2 − 21 1 + 11 2
| |

1 12 11 1

1 = 2 22 and 2 = 21 2| |
| |

Example: Solve the following system of equations:

Solution:

78
This system can be written in matrix form i.e.

2 −3 1 2
=
1 2 2 1
Or
AX=B (1)
Where A = 2 −3 , X = 1 and B = 2
1 2 2 1

|A| = 2 −3 = 4 – (-3) = 4 +3 = 7 ≠ 0
1 2
And adj A = 2 3 , therefore,
−1 2
−1 =
| |

2 3
3 7 7 2
A-1 = 1 2 = −1 2
7 −1 7 7
2 4+ 3 7
(1) becomes X = A-1 B i.e. = 7 7 = 7 = 1
1 −2+ 2 0 0
2 3 7 7
1 = 7 7
−1 2
7 7

• 1 = 1 and 2 =0

The matrix form of the given system is:


79
2 4 1 = 1
1 2 2 2

AX = B (1)

Where A = 2 4 , X= 1 , B= 1
1 2 2 2

|A| = 2 4 = 4 – 4 = 0, so inverse does not exist. Thus the system has no solution.
1 2
Example: Two cups of tea and three pieces of cake cost Rs. 80, while one cup of tea and
two pieces of cake cost Rs. 50. Find the cost of one cup of tea and one piece of cake.

Solution: Suppose the cost of one cup of tea = 1


And the cost of one piece of cake = 2

Then according to condition


2
+ 3 2 = Rs. 80 1
1
+ 2 2 = Rs.50
This system can be written in matrix form i.e.

2 3 1 80
=
1 2 2 50

AX = B (1)

Where A= 23, X= 1 , B = 80
1 2 2 50

|A| = 2 3 = 4 – 3 = 1 ≠ 0
1 2

Adj A = 2 −3
−1 2

−1 =

| |
A-1 = 2 −3
−1 2
(1) becomes X = A-1B i.e.
80
1 = 2 −3 80
2 −1 2 50

(2 × 80) + (−3 × 50)


= = 160 − 150 = 10 (−1 × 80) + (2 × 50) −80 +
100 20
1 = .10 and 2 = .20

Self Assessment Questions

Q.1 If A = 3 −1 and B = −2 3 , find A-1 , B-1 and then show that (AB)-1 = B-1A-1
2 1 −4 5

Q.2 Solve the following system of equations:


i) 4 1 + 2 2 = 3 ii) 3 1 − 8 2 = 10
51 + 2 2 =2 21 −4 2 =9

Q.3 Nine pencils and five erasers cost Rs. 60 and three pencils and ten erasers costs Rs.
45. Find the cost of one pencil and one eraser?

81
3.6 Minor and Cofactor of an Element of a Matrix
Minor of an Element: The minor of an element aij of a matrix A of order 3 is denoted by
Mij and is obtained by deleting the ith row and jth column of A.

11 12 13

For example: A = 21 22 23 then the minor M12 is the determinant of the


31 32 33
matrix obtained by deleting the first row and second column of the matrix A i.e.
11 12 13
21 23
21 22 23 M12 = . Similarly the minor M31 is the determinant of
31 33
31 32 33
the matrix obtained by deleting the third row and first column i.e.

11 12 13
12 13
21 22 23 , M31 =
22 23
31 32 33

Cofactor of an Element: The cofactor of an element aij of a matrix A is denoted by Aij and
is defined as
= (−1) + ×

For example: A12 = (-1)1+2 M12 = (-1)3 22 23 =- 22 23


32 33 32 33

2 −1 1
Example: Find all the cofactors if A = 3 2 0
0 4 −5
Solution:

A11 = (-1)1+1 2 0 = -10 - 0 = -10, A12 = (-1)1+2 3 0 = -1. (-15 - 0) = 15


4 −5 0 −5

A13 = (-1)1+3 3 2 = 1.(12 – 0) = 12, A21 = (-1)2+1 −1 1 = -1.(5 – 4) = -1


0 4 4 −5

82
2 1 2 −1
A22 = (-1)2+2 = 1. (-10 – 0) = -10, A23 = (-1)2+3 = -1.(8+0) = -8
0 −5 0 4
A31 = (-1)3+1 −1 1 = 1.(0 – 2) = -2, A32 = (-1)3+2 2 1 = -1.(0 – 3) = 3
2 0 3 0

2 −1
A33 = (-1)3+3 = 1.(4 – (-3)) = 1.(4 + 3) = 7
3 2

3.6.1 Determinant of a Square Matrix of order :

Let us consider a square matrix A of order 3 and we have chosen to make an expansion
along the first row, the determinant would be
|A| = a11A11 + a12A12 + a13A13
= a11M11 - a12M12 + a13M13

If we choose to carry out an expansion along the second column, the determinant would be

|A| = a12A12 + a22A22 + a32A32


= -a12M12 + a22M22- a32M32

1 −2 −3
Example:If A = 2 0 1 , find |A| ?
−4 6 8

Solution:
1 −2 −3
|A| = 2 0 1
−4 6 8
We expand it along the first row i.e.

0 1 2 1 2 0
|A| = 1. - (-2) + (-3)
6 8 −4 8 −4 6

= 1.(0 - 6) +2(16 – (-4)) -3 (12 – 0)

83
= -6 + 2(16 + 4) -3 (12)

= -6 + 40 -36 = 40 – 42 = -2 |

A| = -2

84
3.6.2 Adjoint of a Matrix of order 3:
The adjoint of a square matrix A is obtained by taking the transpose of the cofactor matrix
of a given original matrix.

11 12 13 11 12 13
Let A = 21
22 23 and its cofactor matrix is A = 21 22 23 , then
31
32 33 31 32 33

11 12 13 11 21 31

Adj A = 21 22 23 = 12 22 32

31 32 33 13 23 33

3.6.3 Inverse of a Matrix of order 3: If

A is a non singular matrix then,


−1 =
| |

2 1 0
Example: If A = −1 −5 2 , Find A-1 ?
0 1 3

Solution: We first find the cofactors of the elements of A.

A = (-1)1+1 −5 2 = -15 - 2 = -17, A = (-1)1+2 −1 2

11 1 3 12 0 3 = -1. (-3 - 0) = 3 A

= (-1)1+3 −1 −5 = 1.(-1 + 0) = -1, A = (-1)2+1 1 0 = -1.(3 – 0) = -3


13 0 1 21 1 3

A22 = (-1)2+2 2 0 = 1. (6 – 0) = 6, A = (-1)2+3 2 1 = -1.(2 - 0) = -2


0 3 23 0 1

= (-1)3+1 1 0 = 1.(2 + 0) = 2, A = (-1)3+2 2 0

85
A31 −5 2 32 −1 2 = -1.(4 +0) = -4 A

= (-1)3+3 2 1 = 1.(-10 – (-1)) = 1.(-10 + 1) = -9


33 −1 −5

11 The
cofactor matrix of A = 21 12 13 −17 3 −1

31 22 23 = −3 6 −2

32 33 2 −4 −9
−17 Adj A
= −3 3 −1 −17 −3 2
2 6 −2 = 3 6 −4 −1

−4 −9 −2 −9

|A| = a11A11 + a12A12 + a13A13 |

A| = 2(-17) + 1(3) + 0(-1)


= -34 + 3 +0 = -31

−1 =
| |
−17 −3 2
=1 3 6 −4 −31
−1 −2 −9

−17 −3 2 17 3 2
⎡−31 −31 −31⎤ ⎡ 31 ⎤ 361
⎢ 3 ⎥ ⎢ −431⎥⎥
−4 ⎥ = ⎢ 3 − −231 391 ⎥
= ⎢−31 6
−391⎥ ⎢−131 31 31 ⎦
⎢ −1 −−321 ⎣−31
−31⎦ ⎣ 31
−31

Self Assessment Questions:


−3 4 5 0 8 1
Q.1 If A = 0 −1 2 , B = −1 1 2 , then find

86
1 2 3 2 0 1

i) Adj (A) ii) Adj (B) iii) |A| iv) |B| v) |AAt| vi) |BtB|

1 2 3
Q.2. If C = −1 −3 0 , then find C-1 and verify that
2 2 1

i) (C-1)-1 = C ii) (C-1)t = (Ct)-1 iii) C-1C = I3 = CC-1


3.7 System of Linear Equations in 3 Variables:
Consider the system of linear equations:
1 + 1 + 1 = 12
+ 2 + 2 = 2
3 3 + 3 = 3

This is system of non homogenous linear equations in three variables, where x, y and z are
variables and k1, k2and k3 are constant terms and not all zero. If k 1= k2= k3=0, then it is said
to be a system of homogenous linear equation x, y and z.
A system is said to be consistent; if it has a unique solution or infinite many solutions and a
system is said to be inconsistent if it has no solution.

3.7.1 Homogenous Linear Equations:


11 1 + 12 2 + 13 3 =0
21 1 + 22 2 + 23 3 =0
31 1 + 32 2 + 33 3 =0

This system is always satisfied by 1 = 0, 1 = 0 3 = 0. Therefore, this system is


always consistent and the solution (0, 0, 0) of the given system is called trivial solution. Any
other solution of the above system is called non trivial solution.

The given system can be written as:


0
AX = O, where O = 0
0

For a non- trivial solution, at least one of x 1, x2, x3 is non zero and we have |A| = 0. If

|A| ≠ 0, in this case the system of homogenous equations possesses only the trivial

solution. Example: Solve the system:


+2 −2 =0 (i)

87
2 + +5 =0 (ii) 5
+4 +8 =0 (iii)

Solution: The given system can be written as:


1 2 −2 0
2 1 5 = 0
5 4 8 0
1 AX = 2 −2 0
O where A = 2 1 5 ,X= and O = 0
5 4 8 0

1 |A| = 2 2 −2
1 52 5 2 1 -2 -
5 2
1 5 = 1. 4
4 8 8 5 8 5 4

= 1(8 – 20) -2(16 – 25) -2(8 – 5)


= -12 -2(-9) -2(3) = -12 + 18 – 6 = 18 – 18 = 0
|A| = 0
Thus the system has non-trivial solution
Multiply (i) by 2 and then subtract from (ii)

• 3y = 9z
• y = 3z

Putting the value of y in (i)

+ 2(3 ) − 2 = 0 => +6 −2 =0
• + 4 = 0 => = −4

Put in eq. (iii)


5(−4 ) + 4(3 ) + 8 = 0 => -20z +12z +8z=0
• 12z – 12z = 0
Let z = t, where ∈
Then, x = -4t and y = 3t
The given system has infinitely many solutions.

88
Example: Solve the system:
2 − + =0 (i)
+2 +3 =0 (ii)
3 +2 +3 =0 (iii)
Solution: The given system can be written as:
2 −1 1 0
3 2 3 = 0
4 2 3 0

2 −1 1 0
AX = O where A = 1 2 3 ,X= and O = 0
3 2 3 0

2 −1 1
2 3 1 3 1 2
|A| = 1 2 3 = 2. –(-1) +1
2 3 3 3 3 2
4 2 3

= 2(6 – 6) +1(3 – 9) +1(2 – 6)


= 0-6-4 = -10 ≠ 0
|A| ≠ 0
Thus the system has trivial solution
Multiply (i) by 2 and then add with (ii)

• 5 = −5 => =− (iv)
Putting the value of y in (iii)
3(− ) + 2 − 3 = 0
• −3 + 2 − 3 = 0
• 2 = 0=> = 0
Put in eq. (ii)
(− ) + 2(0) + 3 = 0 => − + 0 + 3 = 0
• 2 = 0 => = 0
Put in eq. (iv)
=0
Thus the system has Trivial Solution.

89
3.7.2 Non-Homogenous Linear Equations:
Now we will solve the system of non-homogenous linear equations by using;

a) Matrices i.e. AX = B => X = A-1B.


b) Cramer’s rule

90
a) Use of Matrices:
Example: Use matrices to solve the system:
− 2 + = −1
3 + −2 =4
− =1

Solution:The given system can be written in matrix form as:


1 −2 1 −1 3
1 −2 = 40 1
−1 1

1 −2 1 −1
Let A = 3 1 −2 , X = and B = 4
0 1 −1 1

Then system becomes AX = B => X = A-1B


−1 =
| |

1 −2 1 |
A| = 3 1 −2
0 1 −1

= 1(-1 – (-2)) – (-2)(-3 + 0) + 1(3 – 0)

= 1(-1 + 2) +2(-3) + 3 = 1 – 6 + 3 = -2 ≠ 0

11 12 13

Cofactors of A = 21 22 23

31 32 33

A = (-1)1+1 1 −2 = -1 + 2 = 1, A12 = (-1)1+2 3 −2 = -1. (-3 + 0) = 3


11 1 −1 0 −1

A = (-1)1+3 3 1 = 1.(3 – 0) = 3, A = (-1)2+1 −2 1 = -1.(2 – 1) = -1


13 0 1 21 1 −1

A = (-1)2+2 1 1 = 1. (-1 – 0) = -1, A = (-1)2+3 1 −2 = -1.(1+0) = -1


22 0 −1 23 0 1

A31 = (-1)3+1 1 −12 = 1.(4 – 1) = 3, A32 3+2 13 −2 = -1.(-2 – 3) = 5

91
−2 = (-1) 1

A = (-1)3+3 1 −2 = 1.(1 – (-6)) = 1.(1 + 6) = 7


33 3 1
1 3 3
Cofactors of A = −1 −1 −1
3 5 7

1 −1 3
Adj A = (Cofactors of A)t = 3 −1 5
3 −1 7

−1 1 −3

−1 5 = ⎢⎢⎡−223 2 ⎥
3 2 ⎤

A-1 = −12 31 −−11 7 ⎣⎢−23 2122 2 ⎦


−5⎥

3 1 −7⎥

X = A-1 B

1 4 3 1+4−3
⎡ −1 1 −3⎤ −1 ⎡2 + 2 − 2⎤ ⎡ 2 ⎤ 2 1
⎢−23 21 −25⎥ = ⎢3 + 4 − 5⎥ = ⎢3+4−5⎥= 2 =

= ⎢2 2 2 ⎥ ⎢⎢2 2 2 ⎥ ⎢ 2 ⎥⎥ 22 1
⎥ 4 ⎥ ⎢
⎣⎢−3 1 −7⎦ 1 ⎣3 + 4 − 7⎦ ⎣3+4−7⎦ 0 0
2 2 2 2 2 2 2

x = 1, y = 1 and z = 0
92
b) Cramer’s Rule:
Consider the system of equations:
11 1 + 12 2 + 13 3 = 1
21 1 + 22 2 + 23 3 = 2

31 1 + 32 2 + 33 3 = 3

Then 1 , 2 and 3 can be obtained by

1 12 13 11 1 13 11 12 1
2 22 23 21 2 23 21 22 2

1 = 3 32 33 , 2 = 31 3 33 , 3 = 31 32 3
| | | | | |

This method of solving the system is known as Cramer’s Rule.

Example: Use Cramer’s rule to solve the system:


2 +2 + =3
3 −2 −2 =1
5 + −3 =2

Solution: The given system can be written in matrix form as:

2 2 1 3
3 −2 −2 = 1
5 1 −3 2

2 2 1 3
LetA = 3 −2 −2 , X = and B = 1
5 1 −3 2

2 2 1
|A| = 3 −2 −2 = 2(6 + 2) -2(-9 + 10) +1(3 + 10)
5 1 −3

= 2(8) -2(1) +1(13) = 16 – 2 + 13 = 27

3 2 1
1 −2 −2

93
= 2 1 −3
27

= 1 {3(6 + 2) − 2(−3 + 4) + 1(1 + 4)} 27

= 1 {24 − 2 + 5}
27

= 27 = 1
27

2 3 1
3 1 −2
= 5 2 −3
27

= 1 {2(−3 + 4) − 3(−9 + 10) + 1(6 − 5)} 27

= 1 {2 − 3 + 1} = 0
27
2 2 3
3 −2 1

= 5 1 2
27
= 1 {2(−4 − 1) − 2(6 − 5) + 3(3 + 10)} 27

= 1 {−10 − 2 + 39} 27

= 27 = 1
27

Example: The sum of three numbers is 26. The third number is twice the second and1 less
than three times the first. What are the three numbers?

Solution: Let the first number = x

The second number = y

And the third number = z


Then
x + y + z = 26

z= 2y => -2y + z = 0

94
z = 3x -1 = > -3x + z = -1
1 1 1 26
LetA = 0 −2 1 ,X= and B = 0
−3 0 1 −1
We solve it using Cramer’s Rule

1 1 1
|A| = 0 −2 1 = 1(-2) -1(3) +1(-6) = -2 -3 -6 =
−3 0 1
-11
26 1 1
0 −2 1
= −1 0 1
−11
= −1 (26(−2) − 1(1) + 1(−2) = −1(−55) = 5
11 11
1 26 1
0 0 1
= −3 −1 1 −11

= 1(1)−26(3)+1(0) = 7
−11

1 1 26
0 −2 0
= −3 0 −1 −11

= 1(2)−1(0)+26(−6) = 14
−11

So. = 5, y = 7 and z= 14 Self

Assessment Questions:

95
Q.1 Solve the system of homogenous equations: i)
x – 2y -1 =0 ii) x + 4y + 2z = 0

2x + y +5z = 0 2x + y -3z =0

2x –y + 4z = 0 3x + 2y – 4z = 0

Q.2 Solve the following systems using matrices as well as Cramer’s rule. i)

2x – y + z = 5 ii) x + y = 2 iii) 2x + y + 3z = 3 4x +

2y + 3z = 8 2x – z = 1 x + y – 2z = 0
3x – 4y – z = 3 2y – 3z = -1 -3x – y + 2z = -4
Q.3 The currents running through an electrical system are given by the following
system of equations. The three currents, I1, I2, and I3, are measured in amps. Solve
the system to find the currents in this circuit.

I1 + 2I2 - I3 = 0.425 3I1

- I2+2I3 = 2.225 5I1 + I2

+ 2I3 = 3.775
UNIT-4

SEQUENCES, SERIES AND


MATHEMATICAL INDUCTION
96
Written By: Ms Fouzia Rehman
Reviewed By: Dr. Baber Ahmed

97
CONTENTS
Page No
Introduction ............................................................................................................. 86
Objectives ............................................................................................................. 86

4.1 Sequence ............................................................................................................. 87


4.2 Series ............................................................................................................. 88
4.2.1 Formulas for Special Series:.......................................................................... 90

4.3 Arithmetic Progression/ Sequence .......................................................................... 91

4.4 Arithmetic Mean (A.M) .......................................................................................... 93

4.5 Arithmetic Series: ................................................................................................... 94

4.6 Geometric Progression/Sequence (G.P): ................................................................. 98

4.7 Geometric Mean (G.M):.......................................................................................... 99

4.8 Geometric Series ................................................................................................... 101


4.8.1 Sum of n Terms of a Geometric series: ....................................................... 101
4.8.2 The Infinite Geometric series: ..................................................................... 102

4.9 Mathematical Induction ........................................................................................ 103


4.9.1 Principle of Mathematical Induction ......................................................... 103
4.9.2 Generalized Principle of Mathematical Induction .................................... 105

98
INTRODUCTION
The world of sequences and series is entirely enchanting and interesting. We will find
number of patterns if we observe in our surrounding like the symmetry of a starfish, ripple
on a lake etc.

The method of induction was first developed by Francesco Mourolico (1494-1575) to prove
that 2 is equal to the sum of first n odd positive integers. He introduced many properties of
integers and used the technique of mathematical induction to prove some of these properties.

Mathematical sequences, series and the method of mathematical induction are used in
business, economics of finance, investment, geometry and algebra that help
mathematicians in solving simple as well as complex problems. Apart from various
branches of mathematics, they are also observed in chemistry, physics, computer science
and several other fields.

OBJECTIVES
At the end of unit (4), the readers will be able to:

1) differentiate between sequences and series


2) describe different kinds of sequences and series and apply them in real life
problems
3) define method of mathematical induction.
4) apply Principle of method of mathematical induction to prove various statements,
theorems etc.

99
4.1 Sequence
A sequence is a list of objects, numbers represented in definite order. Usually it follows
some sort of pattern. For example, Monday, Tuesday, Wednesday,…. Sunday is a
sequence that represents the days of a week. Each of the entry in a sequence is called term,
separated by commas. Mathematically, a sequence is a function whose domain is set of
natural numbers. Usually small letters a, b, c etc. are used to name the sequence. The symbol
{ } or 1 , 2 , … , is used to indicate the sequence where 1 , 2 are first and second term while
represents the nth term of the sequence. A sequence can be finite or infinite. A finite
sequence has a last term while an infinite sequence continues without stopping.

Example: Find the first five terms of the sequence:

= (−2) +2………………….. (i)

Solution:Put n = 1, 2, 3, 4 and 5 successively in (i)

31= (−2)3+2 = (−2)5 = −32, 24 == ((−−22))24++22 == ((−−22))46

== 1684
= (−2)1+2 = (−2)3 = −8,

5 = (−2)5+2 = (−2)7 = −128

Example:Find the sequence if − −1 = 2 + 1 5 = 35 . Solution:


Putting 2, 3, 4, 5 in − −1 = 2 + 1, we have
2 − 1 = 2(2) + 1 = 5 ………………….. (i) 3
− 2 = 2(3) + 1 = 7 …………………...(ii) 4 −
3 = 2(4) + 1 = 9…………………..(iii) 5 − 4
= 2(5) + 1 = 11…………………(iv)

As 5 = 35, eq. (iv) becomes

35 - 4 = 11 => 4 = 35 − 11 = 24

Putting 4 in (iii)

24 − 3 = 9 => 3 = 24 − 9 = 15

Putting 3 in (ii)

15 − 2 = 7 => 2 = 15 − 7 = 8

Putting 2 in (i) we get

100
8− 1 = 5 => 1 = 8 − 5 = 3 Thus

the sequence is 3, 8, 15, 24, 35, ……

Example: Describe the pattern, write the next term and write a rule for the nth term of
the sequence:
2 2 2 2
, , , ,…
3 9 27 81
Solution: The given sequence can be written as:

2 2 2 2 2
(3)1 , (3)2 , (3)3 (3)4 5 = (3)5
, ,… The next term is

A rule for the nth term is = 2

(3)

Self Assessment Questions:

Q.1 Write the first six terms of the following sequences:


a) = 2 b) (−2) ( 2 − 3 + 2) c) − = 5 − 2, =33
−2 −1 1

Q.2 Find the first three, 25th and 30th term of the sequence whose nth term is
(−1)2 +1( + 1).

Q.3 Describe the pattern, write the next three terms and write a rule for the nth term of
the sequences:
1 ,1 ,1 ,1
a) 1,4,9,16,…. b) 0,2,6,12,…. c) ,….
3 5 7 9

4.2 Series
The sum of the terms of a sequence is called series. It can be finite or infinite. For example

Finite Seq. Infinite Seq. 3, 6,


9, 12, 15 3, 6, 9, 12, 15…
101
Finite Series Infinite Series
3+6+9+12+15 3+6+9+12+15+…

A Greek letter Σ (Pronounced “Sigma”) is used to write the series. The series 3 + 6 + 9
+ 12 + 15 can be written as:

=
This is read as “ the sum from = 1 to 5 of 3 ” where 5 is the upper limit while 1 is the
lower limit, the first term of summation. The infinite series 3 + 6 + 9 + 12 + 15 +
⋯ can be written in summation notation as:

=
Where ∞ represents infinity symbol.

Example: Find the sum of the series:


5 6
+1
) ) ( 2 − 1)
=1 =1

5
+1 2 3 4 5 6 437
) = + + + + =
1 2 3 4 5 60
=1

) ( 2 − 1) =1(1 2 − 1) + 2(22 − 1) + 3(32 − 1) + 4(42 − 1) + 5(52 − 1) + 6(62 − 1)


=1

= 0 + 2(3) + 3(8) + 4(15) + 5(24) + 6(35)

= 0 + 6 + 24 + 60 + 120 + 210

= 420
Example: Write the given series with summation notation:

a)−9 − 17 − 25 − 33 − ⋯ b)2 +4+ 8 + 16


3 9 27 81

a)−9 − 17 − 25 − 33 − ⋯ This series can be written as:


(-8 -1) + (-16 -1) + (-24 -1) + (-32 -1)+ …
102
= (-8(1) -1 ) + (-8(2) -1) + (-8(3) -1) + (-8(4) -1) + ….

= (−8 − 1) =1

2 4 8 16
b) + + + This series can be written as:
3 9 27 81
1 2 3 4 4

((23))1 + ((32))2 + ((32))3 + ((32))4 = 23


=1
4.2.1 Formulas for Special Series:

i) Sum of the positive integers from 1 to n:

( + )
=
=

ii) Sum of squares of positive integers from 1 to n:

( + )( + )
=
=

iii) Sum of cubes of positive integers from 1 to n:

( + )
=
=

Example:Find the sum of the given series by using formula

i) ∑100 3 ii) ∑35 (2 − 1)2 iii) ∑10 3


=1 =1 =1

100 100
3 100(100 + 1)
) 3 =3 = = 15,150
=1 =1

35 35 35 35 35
) (2 − 1)2 = 4 2 −4 +1 = 4 2 − 4 + 1
103
=1 =1 =1 =1 =1

= 4∑35=1 2 − 4∑35=1 + ∑35 1 = 4 35(35+1)(2(35)+1) − 4 35(35+1) + 35


=1 6 2

4(35)(36)(71) 4(35)(36)
= − + 35 6
2

= 59640 –2520 + 35

= 57,155

10 3 10(10+1) 2 100(121)
iii) ∑ =1 = 2 = 4 = 3,025

Self Assessment Questions:


Q.1 Write the given series with summation notation also find their sum:

i) 1 +3+5+7+ 9 + 11 ii) 1 2 + 32 + 52 + 72 + 92
2 4 6 8 10 12

Q.2 Find the sum of the given series to n terms:

i) 23 + 43 + 63 + ⋯ ii) 12 − 22 + 32 − 42 + ⋯ + (2 − 1)2 − (2 )2

4.3 Arithmetic Progression / Sequence


An Arithmetic sequence/ Progression is a sequence in which the difference between any two
consecutive terms is same. This difference of an A.P is called common difference, denoted
by d. The general form of this sequence is:

1, 1 + , 1 +2 , 1 + 3 ,…, 1 + ( − 1) ,….

where a1 is the first term, d is common difference and 1 + ( − 1) is the nth or general
term of the sequence.

Example:Find the next two terms, 30th and 100th terms of the given sequence:

4, 9, 14, 19,….

Solution:

Here 1 = 4, 2 = 9, 3 = 14, 4 = 19, a2 – a1= 9 – 4 = 5, a3 – a2 = 14 – 9 = 5 so, common


difference d = 5

104
The general term of an A.P is;

= 1 + ( − 1) ……….. (i)

Put n = 5, 6, 30 and 100 in (i), we get

5 = 4 + (5 − 1)4 = 4 + (4)5 = 24

6 = 4 + (6 − 1)5 = 4 + (5)5 = 29

30 = 4 + (30 − 1)5 = 4 + (29)5 = 149 100

= 4 + (100 − 1)5 = 4 + (99)5 = 499


=9 = 112.
Example: Find the number of terms in A.P if 1 = 4,

Solution: Given that 1 = 4, =9 = 112. Using

= 1 + ( − 1) , we have

112 = 4 + ( − 1)9 =>112 − 4 = ( − 1)9

• 108 = − 1 => 12 =
− 1 => = 13
9

Example: The 8th and 23rd term of an A.P is 50 and 155 respectively. Find 17 .
Solution: Given that 8 = 50 and 23 = 155

The general term of an A.P is = 1 + ( − 1) ………….. (i)

Put n = 8 in (i)

8 = 1 + (8 − 1)

50 = 1 + 7 ……………. (ii)

Put n = 23 in (i)

23 = 1 + (23 − 1)

105
155 = 1 + 22 ………… (iii)

Subtract (ii) from (iii)

105 => d = 7 =
15

Putting the value of “d” in (ii)

50 = 1 + 7(7) => 50 − 49 = 1 => 1 =1

Put n=17, 1 =1 = 7 in (i)

17 = 1 + (16)7 => =

= 1 + ( − 1)7 => = 1 + 7 − 7 => = −

Example: Ali scored 42 points in his first quiz of mathematics, then he scored 48 points in
his second and 54 points in third quiz. If his marks in quizzes continued to improve at the
same rate, what will be his score on his 10thquiz?

Solution: The sequence is 42, 48, 54,…..

Where 1 = 42, 2 = 48 3 = 54 , common difference d = 6 and we have to find 10 . The

general term of an A.P is = 1 + ( − 1)

10 = 42 + (10 − 1)6 => 10 = 42 + (9)6 => 10 = 96

Self Assessment Questions:


Q.1 Find the 15th term of the sequence , 1 , 2 − , 3 − 2 , …
Q.2 If the 4th and 21st term of an A.P is 29 and 182 respectively. Find the 14th term and
the general formula for this sequence.
Q.3 The equation ( ) = −10 + 27 generates the arithmetic sequence for x = 0, 1, 2,
3, …. What is the 29th term in the sequence.

106
Q.4 Find the first six terms of the sequence 3 7 = 7 4 10 = 33.
Q.5 The sum of the interior angles of a triangle is 180o, of a quadrilateral and a
pentagon is 360o and 540o respectively. If this pattern continues, find the sum of the
interior angles of a dodecagon (12 sides).

4.4 Arithmetic Mean (A.M)


If , , are three consecutive terms of an . then A is called Arithmetic Mean (A.M) between a
and b. i.e. , , are in A.P and is their common difference then
– = – => 2 = +

+
=> =

In general if is the arithmetic mean between −1 +1 ,then

= − + +

107
Example: Find three A.Ms between √5 5√5
Solution:Let 1 , 2 3 be three arithmetic means between √5 5√5.

√5 , 1, 2, 3 ,5√5 are in A.P 1

= √5 , 5 = 5√5

Using = 1 + ( − 1) , we get

5 = 1 + (5 − 1)

4√5
5√5 = √5 + 4 => 5√5 − √5 = 4 => = => =√
4

1 = 1 + = √5 + √5 = 2√5

2 = 1 + = 2√5 + √5 = 3√5

3 = 2 + = 3√5 + √5 = 4√5

Therefore, 2√5 ,3√5 4√5 are three A.Ms between √5 5√5.

Self Assessment Questions:

Q.1 Find six A.Ms between 12 and -9.


Q.2 Find the value of x and then find general term if x+1, 4x+1 and 8x-1 are the
consecutive terms of an A.P.
Q.3 If 8 and 11 are two A.M between a and b. Find a and b.
Q.4 If is the A.M between y and -3, whereas is the A.M between and 48. Find the
value of and .

4.5 Arithmetic Series:


The sum of the terms of an arithmetic progression is called arithmetic series. The sum of the
arithmetic series is obtained using the following:
= ( + ) = [ +( − ) ]

Example:Find the arithmetic series if its sixth term is 25 and 5 = 16 ?


Solution:Given that 6 = 25 and 5 = 16
6 = +5

108
Or 25 = 11+ 5 ………….. (i)
5 = 5 [2 1 + 4 ] = (Given = )2
16 5 16

Or 5 [2 1 +4 ]= + 15
2 1

• 5[ 1 +2 ]= 1 + 15

• 5 1 + 10 = 1 + 15

• 5 1 − 1 = 15 − 10

• 4 1 =5

Put in (i)

25 = 1 +4 1 => 25 = 5 1 => 1 =5

Putting the value of 1 in (i)

25 = 5 + 5 => 20 = 5 => =4

Thus the series is 5 + 9 + 13 + 17 + 21 + 25 + …..

Example: Find the sum of all natural numbers from 1 to 400 which are divisible by 5.

Solution: The sequence of numbers divisible by 5 is:

5, 10, 15, 20, 25, 30, …….., 395 (which is in A.P)

Here 1 = 5, = 10 − 5 = 5, =?, = 395

First we find n

= 1 + ( − 1)

395 = 5 + (n-1)5 => 390 = (n-1)5 => 78 = n-1 =>n = 79


Now

= 2 [2 1 + ( − 1) ]

79
79 = [2(5) + (79 − 1)5]
2
109
• = 79 [10 + (78)5]
2

79
= (10 + 390)
2

79
= (400) => 79(200) 2

79 = 15,800

Example : If the sum of n terms of an A.P is 3 n + n2. Find the nth term.

Solution: We have =3 + 2

= 3( − 1) + ( − 1)2
−1

=3 −3+ 2 −2 +1

= −2 + + 2

ℎ = = − −1

=3 + 2 − (−2 + + 2)

=3 + 2 +2− − 2

=2+2

Example:A writer wrote 1000 words on the first day, 920 words on the 2nd day and 840
words on the 3rd day. How many words did the writer write in 10th day? Find the total
number of words did he write in a week?

110
Solution: The sequence is 1000, 920, 840 ….

Here 1 = 1000, 2 = 920 3 = 840 and common difference d = 920 – 1000 = - 80

10 = 1 + (10 − 1)(−80)

10 = 1000 + (9)(−80)

10 = 1000 − 720 = 280

7
= [2(1000) + (6)(−80)]
7 2

7
7 = [2000 − 480]
2

7
= [1520]
7 2

or 7 = 7(760) = 5320

Self Assessment Questions:


Q.1 The first and last term of an A.P is 9 and 81 respectively and the sum of all terms is
855. Find the common difference and the number of terms?

Q.2 Sum the series:

i) 15 + 23 + 31 + 39 + …..up to n terms

ii) 78 + 89 + 100 + 111 + …..up to 35th terms

Q.3 Find the sum of all natural numbers between 100 and 800 which are divisible

by 13. Q.4 Find the sum of all odd numbers between 500 and 1000.

Q.5 A theater has 40 rows of seats. There are 20 seats in first row, 25 in 2nd , and 30 in
the 3rd row. If this pattern continues, find the total number of seats in the theater.

Q.6 A tube well is bored 850 meters deep. The first meter costs $300 and the cost per
111
meter is increased by $60 for every next meter. Find the cost at 810th meter.

112
4.6 Geometric Progression/Sequence (G.P):
A sequence is said to be a geometric sequence/progression if the ratio between the
successive terms is same i.e. is same. The quotient is called common ratio and is
−1 −
usually denoted by r.
If the first term and common ratio of a sequence is 1 and r respectively then the
geometric progression G.P is
1, 1 , 1 2, 1 3 , …
Then the nth or general term of the sequence is:
= −

Example: Write down the first eight terms of G.P which has first term 3 and common
ratio r is 2. Solution: The general term of G.P is:
= 1 −1 , 1 = 3 =2
Put n = 2,3,….,8 successively in general term
= = (3)(2) = 6, = 2 = (3)(2)2 = (3)(4) = 12
2 1 3 1
= 3 = (3)(2)3 = (3)(8) = 24, = 4 = (3)(2)4 = (3)(16) = 48
4 = 5 = (3)(2)5 = (3)(32) = 96, = 1 6 = (3)(2)6 = (3)(64) = 192
1 5 1
6 1 7
= 7 = (3)(2)7 = (3)(128) = 384
1

Thus, the required G.P is: 3, 6, 12, 24, 48, 96, 192, 384.

Example: The 3rd term of a G.P is 16 and 7th term is 4096. Find the G.P.
Solution: Given that 3 = 16 7 = 4096
The general term of G.P is = 1 −1

3= 16 = 1 2 …………….. (i) 7
= 4096 = 1 6…………..(ii)
6 4096
1 =
2 16
1
4 = 256 => 4 = (4)4 => =4
Putting the value of r in (i)
1(4)2 = 16 => 1 = 1
2 = 1 => 2 = (1)(4) = 4
= 16
4 = (13)(4)3 = 64
5 = (1)(4)4 = 256
113
6 = (1)(4)5 = 1024
7 = 4096

Thus the required G.P is: 1, 4, 16, 64, 256, 1024, 4096, ……..

114
Example: The current population of a certain town is 300,000 and it increases at the
rate of 5% annually. What will be its population at the end of 6 years?

Solution: Suppose the current population = 1= 300,000

TThhee tinoctarleapsoepiunlapotiopuln aatttihon e eant tdheof efnird ofst y e2anrd


=year += 1(10.010550) ==5 = 1(1.05)(0.05) The increase in population at the
end of first year = 1(0.05)
1 1(1.05)

1(1.05) 100
The total population at the end of 2nd year =
1(1.05) + 1(1.05)(0.05) = 1(1.05)(1.05) = 1(1.05)2
The total population at the end of six years is the 7th term of G.P

, 1(1.05), 1 (1.05) 2 ,…
1

Where 1= 300,000, r = (1.05), = 7, 7 =?

= 1 −1

7 = (300,000)(1.05)6 = (300,000)(1.340095) = 402,029

Self Assessment Questions:


Q.1 The sum of three consecutive terms of G.P is 26 and their product is 216. Find those
consecutive terms.(Hint: take three consecutive terms as : 1 , , 2)
1 1
Q.2 If the 2nd and 4th term of a G.P is 43 and 9 respectively. Find the 10th and nth term.
Q.3 5, x, y is an A.P and x, y, 81 is a G.P. Find x and y.
Q.4 The population of Canada was 26.6 million in 1990. The population in 2025 is
estimated to be 38.4 million. If this calculation were based on geometric sequence,
what would the annual growth rate be?
Q.5 If the value of an automobile reduces 10% every year. What will be the price of an
automobile at the end of 5 years which is now purchased for Rs. 50,000?
Q.6 If , + 4 and + 6are three consecutive terms of a G.P. Find x and common
ratio of the sequence.

4.7 Geometric Mean (G.M):


If , , are in G.P then G is called geometric mean (G.M) between a and b.
= => 2 = => = ±√

n Geometric mean between two numbers:

115
If , 1 , 2 , 3 , … . , are in G.P, then 1 , 2, 3,…. are said to be n geometric
means between a and b.
As = 1 −1 here 1 = n = n+2 and +2 =
1
Thus we have = ( +2)−1 => = +1 => = +1
1

Thus = = +1
1
2
+1 2
= 2 =
33
= 3 = +1

⋮ ⋮ ⋮
+1
= =

Example: Find the geometric mean between 3 and 27.


Solution: Here a = 3 and b = 27

= ±√ = = ±√3 × 27 = ±√81 = ±9

Thus the geometric mean may b +9 or -9. We have two geometric sequences: 3,
9, 27 with r = 3 and 3, -9, 27 with r = -3.

Example:Insert three G.Ms between 2 and 32.


Solution:Let 1 , 2 , 3 be three geometric means between 2 and 32. Therefore 2,
1 , 2 , 3 ,32 are in G.P.

Here 1 =2, 5 = 32 =5

5 = 4 => 32 = 2 4 => 16 = 4 => 24 = 4 => =


1
1 = = (2)(2) = 4
2 = =1(2)(2)2 = (2)(4) = 8
1 2
3 = 1 3 = (2)(2)3 = (2)(8) = 16

The three G.Ms between 2 and 32 are 4, 8, 16.

Self Assessment Questions:


Q.1 Find two G.Ms between √2 and 2.
Q.2 Insert four G.Ms between 9 and 1 .
27
Q.3 If the A.M and G.M between two numbers are 34 and 16 respectively, find the

116
numbers.

117
4.8 Geometric Series:
The sum of the terms of the geometric sequence is known as geometric series.
If , , 2 , 3 , … . −1 is a geometric sequence, then = + + 2+ 3 + ….
+ −1 is a geometric series.

4.8.1 Sum of n Terms of a Geometric series:

The sum of n terms can be calculated by using:


= 1 (1− ) if |r|<1
1−
= 1( −1) if | r | >1
−1

Example: Find the sum of the G.P.: 1,


4, 16, 64,….up to the 9th term.

Solution: Here = 1, = 4 >1


=4
1 1

Now using = 1( −1) r>1

= 1(49−−11) => 9 = 1(262144−1) => = 1(262143)


9 4−1 4−1 9 3

9 = 87831

Example: Find the sum of the G.P. to n terms:


2, 1, 0.5, 0.25, 0.125,….
Solution: Here = 2, = 1 <1
= 0.5
1 2

Now using = 1 (1− ) r<1


1−

= 2(1−(121) ) => = 2(1−(12) ) => = 2(1−(12)


) 1− 2−1 1

2 2 2

=> =4 1− 1 is the required sum.


2

Example: An insect travels 90cm in first minute, 60cm in 2nd and 40 cm in 3rd minute.
118
If this pattern continues, find the total distance that the insect will travel in 10th
minute.
Solution The sequence is 90, 60, 40,…..
60 2
1 = 90, = 90 = 3 < 1, = 10
2 10
90 1−
The total distance travel in 10th minute = = 3 = 270(1 − 0.01734) = 270(1 − 0.01734)
10 2
1−
3

= 265.38

4.8.2 The Infinite Geometric series:

Consider the series + + 2 + 3 + ….+ −1 + ⋯ with common ratio r.


Case 1: If | r | >1
For the sum of n terms of the G.P we have = 1( −1)
−1
As n gets larger and larger rn also becomes larger and larger and the sum becomes
infinitely large. Thus the series does not converge and have no sum in this case.
Case 2:If | r | < 1
For the sum of n terms of the G.P we have = 1 (1− )
1−

rn becomes smaller and smaller and approaches to zero as n becomes infinitely large. In this
case the given series converges to the sum =

Example: Find the sum of the given infinite series, if exist:


i)2, 8 , 32 , 128 ii)1, 2, 4, 8,…….
,….
5 25 125

i) Solution: Here = 2, r = 8 ÷2= 8 × = , < 1the sum exists. 1


5 5

Now using the formula of an infinite geometric series.

= 1−1 =1−4 = 5−24 = 10


2

5 5

The sum of the series is 10.


ii) Solution: Here 1 = 1, = 2 = 2 >1, the sum does not exist. 1

119
Example: A ball is dropped from the height of 8feet. Each time it strikes the ground, it
rebounds two third of the distance fallen. Find the total distance travelled by the
ball before coming to rest.
Solution: This problem contains two infinite geometric series. One series includes ball
falling and other includes ball rebounding.
Falling: 1 = 8, = 23
<1
Rebounding: = 8 2 = 16 2<1
, =
1 3 3 3
Now using the formula of an infinite geometric series.
1 = 8

= 1− 1−2 = 24
3
16 16
=1= 3 = 3 = 16 1−
1−2 1
3 3
The total distance travelled by the ball is:
+ = 24 + 16 = 40

Self Assessment Questions:


Q.1 How many terms of the following G.P.: 2 ,− 1 , 1 , … has the sum 55?
3 3 2 72
Q.2 Find the sum of the G.P 3, 0.3, 0.09, 0.027,… to n terms.
Q.3 A line is divided into five parts making a geometric sequence. The length of the
shortest and the longest part is 2 and 162 respectively. Find the length of the whole
line.
Q.4 Find the sum of the infinite G.P 1 , 1 , 1 , 1 ,….

Q.5 A simple pendulum travels a dis4tan12ce 36(in10c8m) 20, 50 , 125 in consecutive seconds.
3 9

Find the total distance travelled by it before coming to rest.

4.9 Mathematical Induction


Mathematical induction is a technique, which is generally used to prove a theorem,
formula or a statement is true for all natural numbers.
Method of mathematical induction can be visualized by means of an analogy. Imagine the
properly arranged dominoes in a row. If we push the first domino the rest will fall
because if any domino falls, it knocks down its neighbor and the next domino will fall too
(fig. 1). The mathematical induction works in the similar manner.

120
Figure 1

4.9.1 Principle of Mathematical Induction

Let ( )be a statement involving positive integers n, to prove a statement this


technique consists of the following two steps:
i) Base Step:for = 1, show that (1) is true
ii) Inductive Step: for any positive integer = , show that the truth
of ( ) implies the ( + 1) is true.
If these two conditions satisfied then ( ) is true for all positive integers.

Procedure:
1. Putting n=1 in the statement, prove that the statement is true for initial value.
2. Assuming the truth of statement for any random positive integer k, then using that
assumption to prove that the statement is true for the next integer, + 1
To prove the 2nd step, one of the following two methods can be used:
• Start from one side of ( + 1) and derive its other side by using ( ).
• Perform algebraic operations on ( ) to establish ( + 1).

Example: Prove by mathematical induction that for all positive integer , we have

The dot (.) represents the


multiplication symbol.

Solution: For each positive integer , let ( ) be the given statement,


( ): 2.2 + 3.22 + 4.23 + ⋯ + ( + 1).2 = . 2 +1
Step 1. If = 1, (1) becomes
(1):2.2 = 1.21+1 = 2 2 , i.e., 4 = 4 which is true.
Hence condition (1) is satisfied.
Step 2. Let us consider that ( ) is true, where be any positive integer, i.e.,

121
2.2 + 3.22 + 4.23 + ⋯ + ( + 1).2 = . 2 +1…………….. ……………. (1) (
+ 1): 2.2 + 3.22 + 4.23 + ⋯ + ( + 1) + 1 . 2 +1 = ( + 1).2 ( +1)+1
2.2 + 3.22 + 4.23 + ⋯ + ( + 2).2 +1 = ( + 1).2 +2 …………………….(2)

We have, LHS of eq. (2):


= 2.2 + 3.22 + 4.23 + ⋯ + ( + 1)2 + ( + 2).2 +1

= . 2 +1 + ( + 2).2 +1 (using eq. 1)


= ( + + 2)2 +1 (taking 2 +1 common)
= (2 + 2)2 +1
= 2( + 1)2 +1
= ( + 1)2 +2 = .(2)

Thus ( + 1) is true. We have proved that ( ) => ( + 1). Therefore, by the


principle of mathematical induction, the given statement holds for every positive integer.

122
Example: Use mathematical induction to prove that 11 − 6 is divisible by 5 for every
positive integer .
Solution: Let P(n) be the given statement,
( ): 11 − 6 5
Step1. If = 1, (1) becomes
(1): 11 − 6 = 5 which is divisible by 5
Hence condition 1 is satisfied.
Step 2. Let us consider that ( ) is true, where is any positive integer i.e.,
11 − 6 5 that means 11 − 6 can be expressed as a multiple of 5. Hence 11 − 6 = 5 for some
integer m. So
11 = 5 + 6 …………….. (i)

Now we will show that ( + 1) is true i.e., 11 +1 − 6 can be expressed as a multiple of 5.


11 +1 − 6 = (11 × 11 ) − 6 (Using Laws of Powers)
= 11(5 + 6) − 6 (Using (i))
= 55 + 66 − 6
= 55 + 60 => 5(11 + 12)
Which is divisible by 5. Thus ( + 1) is true. We have proved that ( ) => ( + 1).
Therefore, by the principle of mathematical induction, the given statement holds for every
positive integer.

4.9.2 Generalized Principle of Mathematical Induction

Let ( ) be a statement for ≥ , where m be a fixed positive integer such that:

i) Base Step: ( ) is true and

ii) Inductive Step: Truth of ( ) implies ( + 1) is true for any positive integer ≥

Then ( ) is true for all positive integers ≥

Example: Prove by principle of mathematical induction that


5 +5≤ 2 ≥6
Solution: Let ( ) represents the given statement i.e. 5 + 5 ≤ 2 ≥ 6

Step 1. For = 6, (6) becomes

(6): 5(6) + 5 = 30 + 5 = 35 (6)2 = 36


i.e. 35 > 36 which is true. The first condition is satisfied.
Step 2. Let us consider that the statement ( ) is true, where ≥ 6 be any positive
integer i.e.
5 + 5 ≤ 2 ……………………………(i)
Now by using induction step, we will show that ( + 1) is true i.e.,
123
5( + 1) + 5 ≤ ( + 1)2 ……………………..(ii)
L.H.S, of inequality (ii) becomes:
5( + 1) + 5 = (5 + 5) + 5 ≤ 2 + 5 (using (i))
We need to show:
2 + 5 ≤ ( + 1)2 = 2 + 2 + 1
We note that for ≥ 2 this inequality holds. So, this is also valid for ≥ 6
When ≥ 6,
5( + 1) + 5 ≤ ( + 1)2
Therefore, by the principle of mathematical induction, the given statement holds for
every positive integer ≥ 6.

Example: Show that the inequality ! > 2 is true for all ≥ 4.


Solution: Let ( ) be the given statement i.e. ! > 2 ≥4

Step 1. For = 4, (4) becomes


(4):4! > 2 4 , i.e., 24 > 16 which is true.
Thus, the first condition is satisfied.
Step 2. Let the statement is true for any positive integer k, where ≥ 6 i.e.,
! > 2 ………………………………….. (i)
Now we will show that ( + 1) is true for ≥ 6 i.e.,
( + 1)! > 2 +1 ………………………… (ii)

L.H.S of (ii) becomes The exclamation mark “!” represents the


factorial. We denote the factorial of a
( + 1)! = ( + 1) ! > ( + 1)2 (using (i)) positive integer n by n! which means the
product of all numbers from 1 to n i.e.
! = 1 × 2 × 3 × 4 × … . . × . So, 4! is
defined as 4! = 4 × 3 × 2 × 1 = 24

We need to show that (

+ 1)2 > 2 +1

( + 1)2 > 2.2 = 2 +1 (for > 2,( + 1) > 2)


This inequality is also valid for ≥4

The inequality (ii) satisfies the condition (2).Thus, by the principle of mathematical
induction, the given statement holds for all positive integers ≥ 4

Self Assessment Questions:


Use principle of mathematical induction to prove the following:

Q.1 4 > 3 + 2 −1 ≥2

124
Q.2 13 + 33 + 53 + ⋯ + (2 − 1)3 = 2(2 2 − 1) for every positive integer n
Q.3 = 1 + ( − 1) ℎ 1 , 1 + , 1 + 2 ,…. form an A.P
Q.4 8 × 10 − 2 is divisible by 6 for all positive values of .
UNIT-5

LIMIT AND DIFFERENTIATION

Written By: Dr. Nasir Rehman


Ms. Mubashara Hafeez
Reviewed By: Dr. Baber Ahmed
125
Contents

Page
Introduction ........................................................................................................... 109
Objectives ........................................................................................................... 109
5.1 Limit ........................................................................................................... 110
5.1.1 Limit of Function ........................................................................................ 110
5.1.2 Test for Existence of a Limit ....................................................................... 110
5.1.3 Properties of Limit ...................................................................................... 111
5.1.4 Limit at Infinity ........................................................................................... 111
5.1.5 Horizontal and Vertical Asymptote ............................................................. 112
5.1.6 Continuity Over an Interval ......................................................................... 112
5.1.7 Average Rate of Change.............................................................................. 113
5.2 The Derivatives ..................................................................................................... 115
5.2.1 Instantaneous Rate of Change ..................................................................... 115
5.2.2 Derivatives .................................................................................................. 115
5.2.3 The Limit Approach to Find the Derivatives .............................................. 115
5.2.4 Rules of Differentiation ............................................................................... 116
5.2.5 Additional Rules of Differentiation ............................................................. 118
5.3 Instantaneous Rate of Change Interpretation ........................................................ 120
5.4 Higher Order Derivatives ...................................................................................... 121
5.3.1 The Second Derivative ................................................................................ 121
5.3.2 Third and Higher Order Derivative ............................................................. 121
5.3.3 Differntiation of Explicit and Implicit Function ......................................... 122
5.3.4 Maxima and Minima ................................................................................... 123
5.5 Applications of Derivatives................................................................................... 124

126
INTRODUCTION
The concept of limit is related to finding the areas of plane region and finding the tangent
lines to the curve. The problem in which we deal with the tangent problem is called
differentiation. In limit and differentiation we deal with physical phenomena which
involve changing quantities the speed of a rocket, the number of bacteria in a culture, the
inflation of a currency, the shock of an earthquake, the voltage of an electric signal and a
relation exist between tangent and rates of change.

OBJECTIVES
At the end of the unit 5, the students will be able to learn about:

1) limit
2) the derivatives
3) instantaneous rate of change interpretation
4) higher order derivatives
5) applications of derivatives

127
5.1 Limit
5.1.1 Limits of Function
Limit is the value that a function or sequence approaches as the input approaches to some
value. This limiting value when it exists is called limit. It is denoted as:

lim ( ) =

5.1.2 Test for Existence of a Limit


If the value of the function approaches the same number as approaches ‘ ’ from right side
(positive direction) or left side (negative direction) then the limit equals i.e.
If lim → − ( ) = and lim → + ( ) = then,

lim ( ) =

Example 1
Determine the lim ( )where ( ) = − 2 +2 +2
→4
Solution
Let’s construct a table of assumed values of x and corresponding values for ( ).

Note that the value = 4 has been approached from both the left and the right sides and
approaching the same value of ( )i.e. −6 hence

lim ( ) = −6 lim ( ) = −6
→4+ →4−

⟹ lim ( ) = −6
→4
Example 2
The production costs per week for producing x widgets is given by ( ) = 500 +
350 − 0.09 2 What is the cost to produce the 301 widget at = 300?

Solution

128
We want to compute the cost to produce 301 widget for this firstly we will compute
(301) as that is the cost of producing 301 widgets and then we will minus it with
(300) which is the cost of producing 300 widgets.

Hence the cost of producing the 301 widget is 295.91.

5.1.3 Properties of Limits


i. If ( ) = where is a real number then,
lim ( ) =

ii. If ( ) = where is a positive integer then,
lim ( ) =

iii. If ( )has a limit as → and is a real number then,

iv. If lim ⟶ ( ) and liml⟶im⟶ (( ))e=xist ltihmen, ( )


lim [ ( ) ± ( )] = lim ( ) ± lim ( )


v. If lim ⟶ ( ⟶) and lim ⟶ ( )exi⟶st then, ⟶
lim [ ( ). ( )] = lim ( ) . lim ( )
vi. If lim ⟶ ( ) ⟶and lim ⟶ ( )exi⟶st then, ⟶

( ) lim ( )
lim = ⟶ lim ( ) ≠ 0
⟶ ( ) lim ( ) ⟶

Self-Assessment Questions:
Find the limit of the following using the properties of limit.
a) lim 9 b) lim 3 c) lim ( 4 + 2 ) d) lim [( + 8) − ( − 5)] →5
→6 →2 →6

5.1.4 Limits at Infinity

129
In limits at infinity we are looking in the behavior of function when is becoming very large
in either the positive or negative sense.

Example lim 2 4 − 2+8


( ) = lim
⟶∞ ⟶∞ −5 4+7
Multiply and divide by 4

2− 1 + 8
2 3

li→m∞ ( ) = l→im∞ −5 + 7
4

2+0+0
lim ( ) =
→∞ −5 + 0

−2
lim ( ) =
→∞ 5

As approaches to infinity the function ( )approaches to − 2 5

5.1.5 Horizontal and Vertical Asymptote


i. Horizontal Asymptote
The line = is a horizontal asymptote of the graph of if and only if lim
( )= lim ( ) =
⟶∞ ⟶−∞

ii. Vertical Asymptote


The line = is a vertical asymptote of the graph of if and only if
lim ( ) = ∞ (−∞) or lim ( ) = ∞ (−∞)
⟶ − ⟶ +

5.1.6 Continuity Over an Interval


130
A function ( ) is said to be continuous at = if

i. lim ⟶ ( )exists
ii. ( ) is defined
iii. lim ⟶ ( ) = ( )

131
Example 3
Given the graph of function ( ) determine if ( ) is continuous at = −2, = 0 and =
3

Solution
i. At = −2

Therefore lim →−2→−(2+) doe( )s= no−t e1x i st. −


lim lim ( ) = 2
→−2

ii. At =0
(0) = 1 lim ( ) = 1
→0
The function is continuous at this point since the function and limit have the same value at
= 0.

iii. At =3
(3) = −1 lim ( ) = 0
→3

The function is not continuous at = 3. Although the limit exists but not equal to the value
of the function at = 3.

5.1.7 Average Rate of Change


Average Rate of Change and Slope

The slope of a straight line can be determined by applying the two-point formula i.e.

Δ
=
= 2Δ− 1
2 − 1
Example 4

132
An object is dropped from a bridge which is 576 feet high. The height of the object can be
determined as a function of time according to the function.

ℎ( ) = 576 − 16 2

Where ℎ( )is height measured in feet and t is time measured in seconds.

Determine the average rate of change in height between = 0 and = 1, between


= 0 and = 2, and between = 0 and = 4.

Solution
The average rate of change means average velocity which is computed as

Here

ℎ( ) = 576 − 16 2

Average rate of change between = 0 and = 1 will be

Average rate of change between = 0 and = 2 will be

Average rate of change between = 0 and = 4 will be

Δℎ ℎ(4) − ℎ(0)
=

133
Δ 4−0
Δℎ (576 − 16(42)) − (576)
=
Δ 4
Δℎ −256
= = −64
Δ 4

134
5.2 The Derivatives
5.2.1 Instantaneous Rate of Change
At a moment, the rate of change is known as instantaneous rate of change. The slope of
tangent line is same as the instantaneous rate of change a point.

5.2.2 Derivatives
The derivative of the function = ( ) is:
( + )− ( )
= lim
→0
Provided this limit exists.

5.2.3 The Limit Approach to Find the Derivatives


To find the derivative by limit approach we will do the following two steps.

I. Determine the difference quotient for using equation:


( + )− ( )
=
II. Find the limit of the difference quotient as → 0 using (
+ )− ( )
= lim
→0
Example 5
Find the derivative of ( ) = 2.
Solution

135
Example 6
The production costs per day for some widget is given by:
( ) = 2500 − 10 − 0.01 2 + 0.0002 3
What is the marginal cost when = 200, = 300 and = 400?

Solution
To find the marginal cost we need to compute the derivative of the given function i.e.

The marginal cost at = 200, = 300 and = 400 is $10, $38 and $78 respectively.

5.2.4 Rules of Differentiation


Differentiation is a process of finding derivatives.

Rule 1: Constant Function


If ( ) = , where is any constant then,

′( ) = 0
Where ′( ) (read as prime ) represents the derivative of the function at .

136
Rule 2: Power Rule
If ( ) = where n is real number then.
′( )= −1
Rule 3: Constant times a Function
If ( ) = . ( ), where c is constant and g is a differentiable function.
′( ) = . ′( )

Rule 4: Sum or Difference of Function


If ( ) = ( ) ± ( ), where u and v are differentiable.
′( ) = ′( ) ± ′( )
Rule 5: Product Rule
If ( ) = ( ). ( ), where u and v are differentiable, then
′ ( ) = ′ ( ). ( ) + ( ). ′ ( )
Rule 6: Quotient Rule
If ( ) = ( )/ ( ), where u and v are differentiable and ( ) ≠ 0,then:
( ). ′ ( ) − ( ) ′ ( )
′( ) =
[ ( )]2
Example 7
Differentiate the following function
( ) = 15 100 − 3 12 + 5 − 46
Solution
To differentiate the given function, we need to remember the first and second rule of
differentiation i.e. constant function and power rule.
′ ( ) = 15 × 100 100−1 − 3 × 12 12−1 + 5 × 1 1−1 − 0
′ ( ) = 1500 99 − 36 11 + 5
Example 8
The position of any object at any time t (in hours) is given by
( ) = 2 3 − 21 2 + 60 − 10

Determine when the object is moving to the right and when the object is moving to the left.

Solution
To determine the direction of the object we need to determine the velocity of the object. If
the velocity is positive this implies the direction of the object is moving off to the right and
if the velocity is negative this implies the direction of the object is moving to the left.

To determine the velocity, we derivate the given function.

′( ) = 2 × 3 3−1 − 21 × 2 2−1 + 60 × 1 1−1 ′


( ) = 6 2 − 42 + 60
′ ( ) = 6( − 2)( − 5)

137
The intervals at which velocity is positive or negative is given by: 1st

Positive at −∞ < < 2 5 < < ∞ i.e. (−∞,2) ∪ (5,∞) 2nd

Negative at 2 < < 5 i.e. (2,5)

Hence the object is moving to the right over the interval (−∞,2) ∪ (5,∞) and the object is
moving to the left over the interval (2,5).

Example 9
Differentiate the given function
( ) = (6 3 − )(10 − 20 )
Solution
To differentiate the given function, we apply the rule 5 that is product rule.
′ ( ) = ′ ( ). ( ) + ( ). ′ ( )
′ = (6 × 3 3−1 − 1 × 1 1−1)(10 − 20 ) + (6 3 − )(−20 × 1 1−1) ′ (
) = (18 2 − 1)(10 − 20 ) + (6 3 − )(−20)
′ ( ) = −480 3 + 180 2 + 40 − 10

Example 10
Differentiate the following function
3 +9
( )=
2−
Solution
To differentiate the given function, we apply the rule 6 that is quotient rule.

′( ) ( ) − ( ) ′( ) ′
( )=
[ ( )]2

(3 × 1 1−1)(2 − ) − (3 + 9)(−1 × 1 1−1) ′


( )=
(2 − )2

3(2 − ) − (3 + 9)(−1)
′( )=
(2 − )2

′( )= 15
(2 − )2

5.2.5 Additional Rules of Differentiation


Rule 7: Power of a Function

138
If ( ) = [ ( )] where u is a differentiable function and n is a real number then
′ ( ) = . [ ( )] −1 . ′ ( )

Rule 8: Base e Exponential Function


If ( ) = ( ) , where u is the differentiable then
′( ) = ′( ) ( )
Rule 9: Natural Logarithm Function
If ( ) = ( ), where u is differentiable then
′( ) ′
( )=
( )
Rule 10: Chain Rule
If = ( ) is a differentiable function and = ( ) is a differentiable function then:

= .
Example 11
Use the chain rule to differentiate the given function:
( ) = √5 − 8
Solution
Using the chain rule, we need to identify the two functions that we need for composition i.e.
( )=√ ( )=5 −8
1
′( ) = ′( ) = 5
2√
Now using the chain rule, we get.
′( ) = ′ ( ) ′( )
′ ( ) = ′ (5 − 8) ′ ( )
1 −1
′ ( ) = (5 − 8) 2 (5)
2
5
′( ) =
2√5 − 8
Example 12
The demand equation for a certain product is = 6 − 1 dollars. Find the level of
2
production that results in maximum revenue.
The revenue function ( )is
1(
)= . = 6−
2(
)=6 −1 2
2
Solution

139
The marginal revenue is given by
′( )=6×1 1−1− 1 × 2 2−1
2
′( ) = 6 −
The graph of R(x) is a parabola that opens downward. It has a horizontal tangent at x for
which ′ ( ) = 0that is for those x at which marginal revenue is zero is at = 6.
Hence the corresponding value of revenue is:

1
(6) = (6)(6) − (6)2 2
(6) = 18$

Thus, the rate of production resulting in maximum revenue is = 6, which results in total
revenue of 18 dollars.

Self-Assessment Questions:
Using the rules of differentiation find the derivatives of the following:

) ( )=6 b) ( ) = 2 + 1 c) ( ) = 6 5 d) ( ) = 9 2 + 5 − 4
e) ( ) = 7 3 f) ( )= 8 3 g) ( ) = ( 4 + 9)2 (5
− 4)
4 +7
h) Using the chain rule find of the function = (2 2 + 1)3

5.3 Instantaneous Rate of Change Interpretation


The derivatives can be interpreted as Instantaneous Rate of Change Interpretation. The
average rate of change of a function = ( ) from = to = is defined by.
( )− ( )
ℎ =

Example 13

140
Suppose a company’s total cost in dollars to produce x units of its product is given by :
( ) = 0.01 2 + 25 + 1500
Find the average rate of change of total cost for
(a) The first 100 units produced from = 0 to = 100
(b) The second 100 units produced.

Solution
(a) The average rate of change of total cost from = 0to = 100 units are:

(100) − (0) (0.01)(100)2 + 25(100) + 1500 − (1500)


=
100 − 0 100
(100) − (0) 4100 − 1500
=
100 100
2600
= = 26
100

The average rate of change of total cost from = 0 to = 100 units is 26 dollars per unit.

(b) The average rate of change of total cost from = 100 to = 200 units are:

(200) − (100) (0.01)(200)2 + 25(200) + 1500 − (4100)


=
200 − 100 100
2800
= = 28
100

The average rate of change of total cost from = 100 to = 200 units is 28 dollars per
unit.

5.4 Higher Order Derivatives


5.4.1 The Second Derivative
The derivative ′ of the function is referred as first order derivative of the function. The
derivative of the function ′ (denoted by ′ ′ ) is referred as second derivative.

5.4.2 Third and Higher Order Derivative


The derivative of the function of second order derivative is referred as third derivative. The
nth-order derivative of denoted by is found by differentiating the derivatives of order
− 1 i.e. at ,
( )= ( −1( ))

141
Example 14
Find three higher order derivatives of the given function:
( ) = 16 3 − 4 2
Solution
The derivatives of are
′ ( ) = 48 2 − 8
′′ ( ) = 96 − 8
′′′ ( ) = 96

5.4.3 Differentiation of Explicit and Implicit Functions


Explicit Function
Function which is given in terms of independent variable is known as explicit function.
For example, = 2 + 3

Here is dependent variable and is given in terms of the independent variable .


Implicit Function

A function which is given in terms of both dependent and independent variables is known as
implicit function.

For example, + 2 +3 +2=0

Sometimes it is not convenient to express a function explicitly such as 2 + 2 = 4 since it


could be written as
= √4 − 2 or = −√4 − 2
Example 15

Find if 2 = 2 + sin
Solution

2 = 2 +

Differentiating both sides with respect to .

( 2) = ( 2) + ( )

Treating as a function of and use the chain rule


2 =2 +( ) ( )

Treat as a product and apply product rule.

142
2 =2 +( ) +

Collect the terms of

2 −( ) =2 +

Now factor out

(2 − ) =2 +

will be
2 +
=
2 −
Example 16
Find if = 5 +

Solution
=5 +

Differentiate both sides with respect to

= (5 ) + ( )

=5 ( )−

will be

=5−

5.4.4 Maxima and Minima Absolute


Maximum, Absolute Minimum

Absolute Maximum
Let be a function with domain D. Then has an absolute maximum value on D at a point c
if :
( ) ≤ ( ) for all in D
Absolute Minimum

143
Let be a function with domain D. Then has an absolute minimum value on D at a point c
if:
( ) ≥ ( ) for all in D

Example 17
A ball is thrown in the air its height at any time t is given by
ℎ( ) = 3 + 14 − 5 2
What is the maximum height?

Solution
Using the derivative, we can find the slope of the given function. :

ℎ = 3 + 14 − 5 2

Now differentiating with respect to t:


ℎ = 0 + 14 − 10
The slope of the function at any time t is:
ℎ = 14 − 10
Now find when the slope is zero i.e.
14 − 10 = 0
10 = 14
14
= = 1.4
10

The slope is zero at = 1.4 and the height at that time is:

ℎ = 3 + 14 × 1.4 − 5(1.4)2 ℎ
= 3 + 19.6 − 9.8
ℎ = 12.8

The maximum height is 12.8m at = 1.4

Example 18
Find the absolute maximum and minimum values of ( ) = 2 on [−2,1].

Solution
The function is differentiable over the entire domain, so only the critical point where
′ ( ) = 2 will be zero is at = 0
Consider the given domain of the function i.e. [−2,1]

at =0
(0) = 0

144
at = −2
(−2) = 4 at
=1
(1) = 1

The function has an absolute maximum value at = −2 and an absolute minimum value at
=0

5.5 Applications of Derivatives


Example 19
An open top box is to be made by cutting small congruent squares from the corners of a 12-
in. by 12-in. sheet of tin and bending up the sides. How large should be the squares cut from
the corners be to make the box hold as much as possible?

Solution
Let the be the side of the box.
The volume of the box is a function of variable i.e.

( ) = (12 − 2 )2
( ) = 144 − 48 2 + 4 3 (i)
Since the sides of the sheet of tin are 12 in. long, ≤ 6 and the domain of is the interval 0
≤ ≤6

The function ( ) will be zero at = 0 and = 6 and a maximum near = 2.


Now we will derivate the equation (i) i.e.

= 144 − 96 + 12 2

= 12(12 − 8 + 2 )
= 12(2 − )(6 − )

At = 2 and = 6 the =0

Only = 2 lies in the interior of the function’s domain and makes the critical point.
The values of at the = 2 and at the domain 0 ≤ ≤ 6 are

At =2
(2) = 128
at =0
(0) = 0
at =6
(6) = 0

145
The maximum volume is 128 3. The cutout squares should be 2in on a side.

Self-Assessment Questions:

a) =3 (4 −2)
6 −2
b) Find the second order derivative of the given functions
i. ( ) = (9 3 + 7 )2 ii. = 5 2
2 −1
c) Find of the given implicit function 6 2 +5 3 +7 4 + 2 = 120
d) Using the rule of differentiation find of the following function
i. 2 2 ii. 2 3 + 5 + 7 = 80 iii. 7 3 + 3 3 + 12 2 = 50

UNIT-6

INTRODUCTION TO STATISTICS

146
Written by: Dr. Zahid Iqbal
Reviewed by: Dr. Muhammad Zakria

147
CONTENTS

Page
Introduction ................................................................................................................. 128
Objectives ................................................................................................................. 128
6.1 Meaning and Definitions of Statistics ......................................................................... 129
6.2 Types of Data and Data sources ................................................................................. 130
6.2.1 Quantitative Data............................................................................................... 130
6.2.2 Qualitative Data................................................................................................. 130
6.3 Types of Statistics ....................................................................................................... 131
6.4 Scope of Statistics ....................................................................................................... 133
6.5 Importance of Statistics ............................................................................................... 134
6.6 Limitations of Statistics............................................................................................... 136
6.7 Summary ................................................................................................................. 138
6.8 Self-Assessments Questions........................................................................................ 138

148
INTRODUCTION
For a layman, ‘Statistics’ means numerical information expressed in quantitative terms. This
information may relate to objects, subjects, activities, phenomena or regions of space.
As a matter of fact, data have no limits as to their reference, coverage and scope. At the
macro level, these are data on gross national product and shares of agriculture,
manufacturing and services in GDP (Gross Domestic Product).

At the micro level, individual firms, howsoever small or large, produce extensive
statistics on their operations. The annual reports of companies contain variety of data on
sales, production, expenditure, inventories, capital employed and other activities. These data
are often field data, collected by employing scientific survey techniques. Unless regularly
updated, such data are the product of a one-time effort and have limited use beyond the
situation that may have called for their collection. A student knows statistics more intimately
as a subject of study like economics, mathematics, chemistry, physics and others. It is a
discipline, which scientifically deals with data and is often described as the science of data.
In dealing with statistics as data, statistics has developed appropriate methods of collecting,
presenting, summarizing, and analyzing data and thus consists of a body of these methods.

OBJECTIVES
After reading this unit, the student will be able to understand the:

1. Meaning of the Statistics,


2. Definition of the Statistics,
3. Nature of the Statistics,
4. Importance of the Statistics and
5. Limitations of statistics.

“A knowledge of statistics is like a knowledge of foreign language of algebra; it may prove


of use at any time under any circumstance”… Bowley.

149
6.1 Meaning and Definitions of Statistics
In the beginning, it may be noted that the word ‘statistics’ is used rather curiously in two
senses plural and singular. In the plural sense, it refers to a set of figures or data. In the
singular sense, statistics refers to the whole body of tools that are used to collect data,
organise and interpret them and, finally, to draw conclusion from them. It should be noted
that both the aspects of statistics are important, if the quantitative data are to serve their
purpose. If statistics, as a subject, is inadequate and consists of poor methodology, we could
not know the right procedure to extract from the data the information they contain.
Similarly, if our data are defective or that they are inadequate or inaccurate, we could not
reach the right conclusion even though our subject is well developed.

A.L. Bowley has defined statistics as: (i) statistics is the science of counting, (ii) Statistics
may rightly be called the science of averages, and (iii) statistics is the science of
measurement of social organism regarded as a whole in all its manifestations. Boddington
defined as: Statistics is the science of estimates and probabilities. Further, W.I. King has
defined Statistics in a wider context, the science of Statistics is the method of judging
collective, natural or social phenomena from the results obtained by the analysis or
enumeration or collection of estimates.

Seligman explored that statistics is a science that deals with the methods of collecting,
classifying, presenting, comparing and interpreting numerical data collected to throw
some light on any sphere of enquiry. Spiegal defines statistics highlighting its role in
decision-making particularly under uncertainty, as follows: statistics is concerned with
scientific method for collecting, organising, summa rising, presenting and analyzing data as
well as drawing valid conclusion and making reasonable decisions on the basis of such
analysis. According to Prof. Horace Secrist, Statistics is the aggregate of facts, affected to a
marked extent by multiplicity of causes, numerically expressed, enumerated or
estimated according to reasonable standards of accuracy, collected in a systematic
manner for a pre-determined purpose, and placed in relation to each other.
From the above definitions, we can highlight the major characteristics of statistics as
follows:

(i) Statistics are the aggregates of facts. It means a single figure is not statistics.
For example, national income of a country for a single year is not statistics but the
same for two or more years is statistics.
(ii) Statistics are affected by a number of factors. For example, sale of a product
depends on a number of factors such as its price, quality, competition, the income of
the consumers, and so on.
(iii) Statistics must be reasonably accurate. Wrong figures, if analysed, will lead to
erroneous conclusion. Hence, it is necessary that conclusion must be based on
accurate figures.
(iv) Statistics must be collected in a systematic manner. If data are collected in a
haphazard manner, they will not be reliable and will lead to misleading conclusion.
(v) Collected in a systematic manner for a pre-determined purpose

150
(vi) Lastly, Statistics should be placed in relation to each other. If one collects data
unrelated to each other, then such data will be confusing and will not lead to any
logical conclusion. Data should be comparable over time and over space.

6.2 Types of Data and Data Sources


Statistical data are the basic raw material of statistics. Data may relate to an activity of our
interest, a phenomenon, or a problem situation under study. They derive as a result of the
process of measuring, counting and/or observing. Statistical data, therefore, refer to those
aspects of a problem situation that can be measured, quantified, counted or classified.
Any object subject phenomenon or activity that generates data through this process is
termed as a variable. In other words, a variable is one that shows a degree of variability
when successive measurements are recorded. In statistics, data are classified into two broad
categories: quantitative data and qualitative data. This classification is based on the kind of
characteristics that are measured.

6.2.1 Quantitative Data are those that can be quantified in definite units of
measurement. These refer to characteristics whose successive measurements yield
quantifiable observations. Depending on the nature of the variable observed for
measurement, quantitative data can be further categorized as continuous and discrete data.

Obviously, a variable may be a continuous variable or a discrete variable.

(i) Continuous data represent the numerical values of a continuous variable. A


continuous variable is the one that can assume any value between any two points on a
line segment, thus representing an interval of values. The values are quite precise and
close to each other, yet distinguishably different. All characteristics such as weight,
length, height, thickness, velocity, temperature, tensile strength, etc., represent
continuous variables. Thus, the data recorded on these and similar other characteristics
are called continuous data. It may be noted that a continuous variable assumes the
finest unit of measurement. Finest in the sense that it enables measurements to the
maximum degree of precision.

(ii) Discrete data are the values assumed by a discrete variable. A discrete variable is the
one whose outcomes are measured in fixed numbers. Such data are essentially count
data. These are derived from a process of counting, such as the number of items
possessing or not possessing a certain characteristic. The number of customers
visiting a departmental store everyday, the incoming flights at an airport and the
defective items in a consignment received for sale, are all examples of discrete data.

6.2.2 Qualitative Data refer to qualitative characteristics of a subject or an object. A


characteristic is qualitative in nature when its observations are defined and noted in terms

of the presence or absence of a certain attribute in discrete numbers. These data are further
classified as nominal and rank data.

151
(i) Nominal data are the outcome of classification into two or more categories of items or
units comprising a sample or a population according to some quality
characteristic. Classification of students according to sex (as males and females), of
workers according to skill (as skilled, semi-skilled, and unskilled), and of
employees according to the level of education (as matriculates, undergraduates, and
post-graduates), all result into nominal data. Given any such basis of classification, it
is always possible to assign each item to a particular class and make a summation of
items belonging to each class. The count data so obtained are called nominal data.

(ii) Rank data, on the other hand, are the result of assigning ranks to specify order in
terms of the integers 1,2,3, ..., n. Ranks may be assigned according to the level of
performance in a test. a contest, a competition, an interview, or a show. The
candidates appearing in an interview, for example, may be assigned ranks in
integers ranging from I to n, depending on their performance in the interview. Ranks
so assigned can be viewed as the continuous values of a variable involving
performance as the quality characteristic.

Data sources could be seen as of two types, viz., secondary and primary. The two can be
defined as under:
(i) Primary data: Those data which do not already exist in any form, and thus have to be
collected for the first time from the primary source(s). By their very nature, these data
require fresh and first-time collection covering the whole population or a sample
drawn from it.

(ii) Secondary data: They already exist in some form: published or unpublished - in an
identifiable secondary source. They are, generally, available from published
source(s), though not necessarily in the form actually required.

6.3 Types of Statistics


There are two major divisions of statistics such as descriptive statistics and inferential
statistics. The term descriptive statistics deals with collecting, summarizing and
simplifying data, which are otherwise quite unwieldy and voluminous. It seeks to achieve
this in a manner that meaningful conclusion can be readily drawn from the data.
Descriptive statistics may thus be seen as comprising methods of bringing out and
highlighting the latent characteristics present in a set of numerical data. It not only
facilitates an understanding of the data and systematic reporting thereof in a manner; and
also makes them amenable to further discussion, analysis, and interpretations.

The first step in any scientific inquiry is to collect data relevant to the problem in hand.
When the inquiry relates to physical and/or biological sciences, data collection is
normally an integral part of the experiment itself. In fact, the very manner in which an
experiment is designed, determines the kind of data it would require and/or generate. The
problem of identifying the nature and the kind of the relevant data is thus automatically
resolved as soon as the design of experiment is finalized. It is possible in the case of
physical sciences. In the case of social sciences, where the required data are often

152
collected through a questionnaire from a number of carefully selected respondents, the
problem is not that simply resolved. For one thing, designing the questionnaire itself is a
critical initial problem. For another, the number of respondents to be accessed for data
collection and the criteria for selecting them has their own implications and importance for
the quality of results obtained. Further, the data have been collected, these are
assembled, organized and presented in the form of appropriate tables to make them
readable. Wherever needed, figures, diagrams, charts and graphs are also used for better
presentation of the data. A useful tabular and graphic presentation of data will require that
the raw data be properly classified in accordance with the objectives of investigation and the
relational analysis to be carried out.

A well thought-out and sharp data classification facilitates easy description of the hidden
data characteristics by means of a variety of summary measures. These include measures of
central tendency, dispersion, skewness and kurtosis, which constitute the essential scope of
descriptive statistics. These form a large part of the subject matter of any basic textbook on
the subject and thus they are being discussed in that order here as well.

Inferential statistics, also known as inductive statistics, goes beyond describing a given
problem situation by means of collecting, summarizing, and meaningfully presenting the
related data. Instead, it consists of methods that are used for drawing inferences, or
making broad generalizations, about a totality of observations on the basis of knowledge
about a part of that totality. The totality of observations about which an inference may be
drawn, or a generalization made, is called a population or a universe. The part of totality,
which is observed for data collection and analysis to gain knowledge about the
population, is called a sample.

The desired information about a given population of our interest; may also be collected even
by observing all the units comprising the population. This total coverage is called census.
Getting the desired value for the population through census is not always feasible and
practical for various reasons. Apart from time and money considerations making the census
operations prohibitive, observing each individual unit of the population with reference
to any data characteristic may at times involve even destructive testing. In such cases,
obviously, the only recourse available is to employ the partial or incomplete
information gathered through a sample for the purpose. This is precisely what inferential
statistics does. Thus, obtaining a particular value from the sample information and using it
for drawing an inference about the entire population underlies the subject matter of
inferential statistics. Consider a situation in which one is required to know the average body
weight of all the college students in a given cosmopolitan city during a certain year. A quick
and easy way to do this is to record the weight of only 500 students, from out of a total
strength of, say, 10000, or an unknown total strength, take the average, and use this average
based on incomplete weight data to represent the average body weight of all the college
students. In a different situation, one may have to repeat this exercise for some future year
and use the quick estimate of average body weight for a comparison. This may be needed,
for example, to decide whether the weight of the college students has undergone a
significant change over the years compared.

Inferential statistics helps to evaluate the risks involved in reaching inferences or


generalizations about an unknown population on the basis of sample information. for
example, an inspection of a sample of five battery cells drawn from a given lot may reveal

153
that all the five cells are in perfectly good condition. This information may be used to
conclude that the entire lot is good enough to buy or not.

Since this inference is based on the examination of a sample of limited number of cells, it is
equally likely that all the cells in the lot are not in order. It is also possible that all the items
that may be included in the sample are unsatisfactory. This may be used to conclude
that the entire lot is of unsatisfactory quality, whereas the fact may indeed be otherwise. It
may, thus, be noticed that there is always a risk of an inference about a population being
incorrect when based on the knowledge of a limited sample. The rescue in such situations
lies in evaluating such risks. For this, statistics provides the necessary methods. These
centres on quantifying in probabilistic term the chances of decisions taken on the basis of
sample information being incorrect. This requires an understanding of the what, why and
how of probability and probability distributions to equip ourselves with methods of drawing
statistical inferences and estimating the degree of reliability of these inferences.

6.4 Scope of Statistics


Apart from the methods comprising the scope of descriptive and inferential branches of
statistics, statistics also consists of methods of dealing with a few other issues of specific
nature. Since these methods are essentially descriptive in nature, they have been
discussed here as part of the descriptive statistics. These are mainly concerned with the
following:

(i) It often becomes necessary to examine how two paired data sets are related.
For example, we may have data on the sales of a product and the expenditure incurred on its
advertisement for a specified number of years. Given that sales and advertisement
expenditure are related to each other, it is useful to examine the nature of relationship
between the two and quantify the degree of that relationship. As this requires use of
appropriate statistical methods, these falls under the purview of what we call regression and
correlation analysis.

(ii) Situations occur quite often when we require averaging (or totalling) of data on prices
and/or quantities expressed in different units of measurement. For example, price of cloth
may be quoted per meter of length and that of wheat per kilogram of weight. Since ordinary
methods of totalling and averaging do not apply to such price/quantity data, special
techniques needed for the purpose are developed under index numbers.

(iii) Many a time, it becomes necessary to examine the past performance of an activity
with a view to determining its future behaviour. For example, when engaged in the
production of a commodity, monthly product sales are an important measure of
evaluating performance. This requires compilation and analysis of relevant sales data over
time. The more complex the activity, the more varied the data requirements. For profit
maximising and future sales planning, forecast of likely sales growth rate is crucial. This
needs careful collection and analysis of past sales data. All such concerns are taken care of
under time series analysis.

(iv) Obtaining the most likely future estimates on any aspect(s) relating to a business or
economic activity has indeed been engaging the minds of all concerned. This is
particularly important when it relates to product sales and demand, which serve the
necessary basis of production scheduling and planning. The regression, correlation and time

154
series analyses together help develop the basic methodology to do the needful. Thus, the
study of methods and techniques of obtaining the likely estimates on
business/economic variables comprises the scope of what we do under business
forecasting.

Keeping in view the importance of inferential statistics, the scope of statistics may finally be
restated as consisting of statistical methods which facilitate decision-- making under
conditions of uncertainty.

6.5 Importance of Statistics


There are three major functions in any business enterprise in which the statistical
methods are useful. These are as follows:

(i) The planning of operations: This may relate to either special projects or to the
recurring activities of a firm over a specified period.

(ii) The setting up of standards: This may relate to the size of employment, volume of
sales, fixation of quality norms for the manufactured product, norms for the daily output and
so forth.

(iii) The function of control: This involves comparison of actual production achieved
against the norm or target set earlier. In case the production has fallen short of the target, it
gives remedial measures so that such a deficiency does not occur again.

A worth noting point is that although these three functions-planning of operations, setting
standards and control-are separate, but in practice they are very much interrelated. Different
authors have highlighted the importance of Statistics in business. For instance, Croxton and
Cowden give numerous uses of Statistics in business such as project planning,
budgetary planning and control, inventory planning and control, quality control, marketing,
production and personnel administration. Within these also they have specified certain
areas where Statistics is very relevant. Another author, Irwing W. Burr, dealing with the
place of statistics in an industrial organisation, specifies a number of areas where statistics is
extremely useful. These are: customer wants and market research, development design and
specification, purchasing, production, inspection, packaging and shipping, sales and
complaints, inventory and maintenance, costs, management control, industrial engineering
and research.

Statistical problems arising in the course of business operations are multitudinous. As such,
one may do no more than highlight some of the more important ones to emphasis the
relevance of statistics to the business world. In the sphere of production, for example,
statistics can be useful in various ways.

Statistical quality control methods are used to ensure the production of quality goods.
Identifying and rejecting defective or substandard goods achieve this. The sale targets can be
fixed on the basis of sale forecasts, which are done by using varying methods of forecasting.
Analysis of sales affected against the targets set earlier would indicate the deficiency in
achievement, which may be on account of several causes: (i) targets were too high and
unrealistic (ii) salesmen's performance has been poor (iii) emergence of increase in

155
competition (iv) poor quality of company's product, and so on. These factors can be further
investigated.

Another sphere in business where statistical methods can be used is personnel


management. Here, one is concerned with the fixation of wage rates, incentive norms and
performance appraisal of individual employee. The concept of productivity is very
relevant here. On the basis of measurement of productivity, the productivity bonus is
awarded to the workers. Comparisons of wages and productivity are undertaken in order to
ensure increases in industrial productivity.
Statistical methods could also be used to ascertain the efficacy of a certain product, say,
medicine. For example, a pharmaceutical company has developed a new medicine in the
treatment of bronchial asthma. Before launching it on commercial basis, it wants to
ascertain the effectiveness of this medicine. It undertakes an experimentation involving the
formation of two comparable groups of asthma patients. One group is given this new
medicine for a specified period and the other one is treated with the usual medicines.
Records are maintained for the two groups for the specified period. This record is then
analysed to ascertain if there is any significant difference in the recovery of the two groups.
If the difference is really significant statistically, the new medicine is commercially
launched.

156
6.6 Limitations of statistics
Statistics has a number of limitations, pertinent among them are as follows:
(i) There are certain phenomena or concepts where statistics cannot be used. This is
because these phenomena or concepts are not amenable to measurement. For
example, beauty, intelligence, courage cannot be quantified. Statistics has no place in
all such cases where quantification is not possible.

(ii) Statistics reveals the average behaviour, the normal or the general trend. An
application of the 'average' concept if applied to an individual or a particular
situation may lead to a wrong conclusion and sometimes may be disastrous. For
example, one may be misguided when told that the average depth of a river from one
bank to the other is four feet, when there may be some points in between where its
depth is far more than four feet. On this understanding, one may enter those points
having greater depth, which may be hazardous.

(iii) Since statistics are collected for a particular purpose, such data may not be relevant or
useful in other situations or cases. For example, secondary data (i.e., data
originally collected by someone else) may not be useful for the other person.
(iv) Statistics are not 100 per cent precise as is Mathematics or Accountancy. Those who
use statistics should be aware of this limitation.

(v) In statistical surveys, sampling is generally used as it is not physically possible to


cover all the units or elements comprising the universe. The results may not be
appropriate as far as the universe is concerned. Moreover, different surveys based on
the same size of sample but different sample units may yield different results.
(vi) At times, association or relationship between two or more variables is studied in
statistics, but such a relationship does not indicate cause and effect' relationship. It
simply shows the similarity or dissimilarity in the movement of the two variables. In
such cases, it is the user who has to interpret the results carefully, pointing out the
type of relationship obtained.

(vii) A major limitation of statistics is that it does not reveal all pertaining to a certain
phenomenon. There is some background information that statistics does not cover.
Similarly, there are some other aspects related to the problem on hand, which are also
not covered. The user of Statistics has to be well informed and should interpret
Statistics keeping in mind all other aspects having relevance on the given problem.

Apart from the limitations of statistics mentioned above, there are misuses of it. Many
people, knowingly or unknowingly, use statistical data in wrong manner. Let us see what the
main misuses of statistics are so that the same could be avoided when one has to use
statistical data. The misuse of Statistics may take several forms some of which are
explained below.

157
(i) Sources of data not given: At times, the source of data is not given. In the absence of
the source, the reader does not know how far the data are reliable. Further, if he wants
to refer to the original source, he is unable to do so.

(ii) Defective data: Another misuse is that sometimes one gives defective data.

This may be done knowingly in order to defend one's position or to prove a


particular point. This apart, the definition used to denote a certain phenomenon may
be defective. For example, in case of data relating to unemployed persons, the
definition may include even those who are employed, though partially. The
question here is how far it is justified to include partially employed persons
amongst unemployed ones.

(iii) Unrepresentative sample: In statistics, several times one has to conduct a survey,
which necessitates to choose a sample from the given population or universe. The
sample may turn out to be unrepresentative of the universe. One may choose a sample
just on the basis of convenience. He may collect the desired information from either
his friends or nearby respondents in his neighbourhood even though such respondents
do not constitute a representative sample.

(iv) Inadequate sample: Earlier, we have seen that a sample that is unrepresentative of the
universe is a major misuse of statistics. This apart, at times one may conduct a survey
based on an extremely inadequate sample. For example, in a city we may find that
there are 1, 00,000 households. When we have to conduct a household survey, we
may take a sample of merely 100 households comprising only 0.1 percent of the
universe. A survey based on such a small sample may not yield right information.

(v) Unfair Comparisons: An important misuse of statistics is making unfair


comparisons from the data collected. For instance, one may construct an index of
production choosing the base year where the production was much less. Then he may
compare the subsequent year's production from this low base.

Such a comparison will undoubtedly give a rosy picture of the production though in
reality it is not so. Another source of unfair comparisons could be when one makes
absolute comparisons instead of relative ones. An absolute comparison of two
figures, say, of production or export, may show a good increase, but in relative terms
it may turnout to be very negligible. Another example of unfair comparison is when
the population in two cities is different, but a comparison of overall death rates and
deaths by a particular disease is attempted. Such a comparison is wrong. Likewise,
when data are not properly classified or when changes in the composition of
population in the two years are not taken into consideration, comparisons of
such data would be unfair as they would lead to misleading conclusion.

(vi) Unwanted conclusion: Another misuse of statistics may be on account of


unwarranted conclusion. This may be as a result of making false assumptions. For
example, while making projections of population in the next five years, one may

158
assume a lower rate of growth though the past two years indicate otherwise.
Sometimes one may not be sure about the changes in business environment in the near
future. In such a case, one may use an assumption that may turn out to be wrong.
Another source of unwarranted conclusion may be the use of wrong average.
Suppose in a series there are extreme values, one is too high while the other is too
low, such as 800 and 50. The use of an arithmetic average in such a case may give a
wrong idea. Instead, harmonic mean would be proper in such a case.

(vii) Confusion of correlation and causation: In statistics, several times one has to
examine the relationship between two variables. A close relationship between the two
variables may not establish a cause-and-effect-relationship in the sense that one
variable is the cause and the other is the effect. It should be taken as something that
measures degree of association rather than try to find out causal relationship.

6.7 Summary
In a summarized manner, ‘Statistics’ means numerical information expressed in
quantitative terms. As a matter of fact, data have no limits as to their reference, coverage
and scope. At the macro level, these are data on gross national product and shares of
agriculture, manufacturing and services in GDP (Gross Domestic Product). At the micro
level, individual firms, howsoever small or large, produce extensive statistics on their
operations. The annual reports of companies contain variety of data on sales, production,
expenditure, inventories, capital employed and other activities. These data are often field
data, collected by employing scientific survey techniques. Unless regularly updated, such
data are the product of a one-time effort and have limited use beyond the situation that may
have called for their collection. A student knows statistics more intimately as a subject of
study like economics, mathematics, chemistry, physics and others. It is a discipline,
which scientifically deals with data, and is often described as the science of data. In dealing
with statistics as data, statistics has developed appropriate methods of collecting, presenting,
summarizing and analysing data and thus consists of a body of these methods.

6.8 Self-Assessments Questions


1. Define Statistics. Explain its types, and importance to trade, commerce and
business.
2. “Statistics is all-pervading”. Elucidate this statement.
3. Write a note on the scope and limitations of Statistics.
4. What are the major limitations of Statistics? Explain with suitable examples.
5. Distinguish between descriptive Statistics and inferential Statistics.

UNIT-7

159
PRESENTATION OF DATA

Written by: Mr. Muhammad Mushtaq


Reviewed by: Prof. Dr. Irshad Ahmad Arshad

160
CONTENTS
Page
Introduction ........................................................................................................... 141
Objectives ........................................................................................................... 141
7.1 Presentation of Data .............................................................................................. 142
7.2 Origion of Statistics............................................................................................... 142
7.3 Types of Data ........................................................................................................ 142
7.4 Presentation of Data .............................................................................................. 143
7.4.1 Classification ........................................................................................................ 143
7.4.2 Tabulation ........................................................................................................... 144
7.4.3 Diagrammatical Presentation ................................................................................ 147
7.4.4 Graphical Representation ...................................................................................... 149

161
INTRODUCTION
In Statistics, presentation of data is very important. In real life problems, we have to
deal with lot of data. Graphs and charts are used to summarize the data and to give
the data an attractive look. This chapter will explain that how large data is
summarized and presented in understandable form by using different statistical tools.

OBJECTIVES
The main objectives are:

1. To introduce the students about the types of data and its presentation.
2. To give introduction of basic graphs, charts and diagrams.

162
7.1 PRESENTATION OF DATA
Whenever we hear the word statistics, we think there will be some information, data,
figures, charts, graphs, diagrams, values or some numeric. Isn’t it? It means statistics relates
to some data or values or numeric. Before discussing the data lets step back to the origin of
statistics.

7.2 Origin of Statistics:


Statistics is a very old word. Nothing is known about the exact origin. However, it is assume
that originally the word “Statistics” have been derived from the Latin word “Status”
or Italian word “Statista” or the German word “Statistik” or the French word “Statistique”,
each of which means, a political state. Shakespeare used a word statist in his drama Hamlet
(1602). In ancient times, statistics primarily was limited for the records of age and sex-wise
manpower and to generate taxes; but gradually it gained broader meanings, because of its
wider applicability to various fields apart from the state activities.

Statistics has developed gradually during the last few centuries. Now it is no longer
restricted to the study of human population or the byproduct of administrative activities of
the state. In the present era of information technology, statistics is regarded as one of the
most import tools for making decisions and its scope has acquired broad spectrum in almost
every sphere of life.

One of the number of meanings and definition of statistics is “the science of systematic
collection, presentation, analysis and interpretation of numerical data to draw conclusions
and to make decisions on the basis of such analysis”. In this sense the word statistics is used
in singular.

Now, then what is data? Before interpreting the data lets understand the concept of
observation. Anything that can be measured or observed is called an observation and the
numbers or measurements that are collected as a result of observations is called data. In
other words, the facts and figures that are collected, analyzed and interpreted are called data.
Data is considered to be useful information.

7.3 Types of Data:


Statistical data are the numerical facts and figures either in counts or measures from relating
to a particular phenomenon under study. The enumerator or investigator is the well-trained
person who collects the statistical data. The respondents (informants) are the persons from
whom the information is collected. Statistical data is of two types:

• Primary data
• Secondary data

163
The data that have been initially collected and have not gone under any statistical
treatment are called primary data while the data which has undergone any sort of
statistical treatment at least once i.e., the data has been collected, tabulated or presented in
some form for a definite purpose.

Activity:
Collect some primary data related to habits of your classmates.

7.4 Presentation of Data:


The number of observations is usually very large in the collected data. Also the raw data,
which have been collected, are generally very large in quantity with the result that it is very
difficult to draw any conclusion from such data until and unless the data are to be condensed
and are fit for further analysis. For this purpose it is necessary to present the data in a
form, which is easy to understand. This is called presentation of data. Presentation of
data includes: i) Classification ii) Tabulation iii) Diagrammatical iv) Graphical

7.4.1 Classification:
It is the process of arranging observations into different classes or categories according to
some common characteristics. The best example of classification is the process of sorting
letters in a post office. The data may be classified or represented by one, two or more
characteristics at a time. If the data is classified according to one characteristic, it is called
one-way classification and if the data is classified according to two characteristics, it is
called two-way classification. As in post office the letters are firstly classified as district-
wise which is an example of one way classification and then they are classified in to tehsil-
wise that is second classification. In this manner the third classification may be mohallah or
town. That is an example of three way classification. When the data is classified according
to many characteristics, it is called many-way classification.

The classification of the data primarily depend s upon the following four basis:

i. Geographical (Spatial)
ii. Chronological (Temporal)
iii. Qualitative
iv. Quantitative

Activity:
Provide some examples of classification based on spatial, temporal, qualitative and
quantitative.
Some characteristics of a good classification are:
• Classification should be unambiguous.
• Classification should be stable.
• Classification should not be rigid.

7.4.2 Tabulation:

164
The process of making tables or arranging the data into rows and columns is called
tabulation.

Construction of tables:
Following are the parts of tables which are involved in the construction of table.

Parts of a table:

Title
Prefatory Notes
Stub Box Head
Column Caption
Row Captions Body of the table
Footnote
Source note

i) Title:
It is the heading at the top of the table. It should be brief and self-explanatory. It
describes the contents of the table.
ii) Column captions and Box-head:
The heading for different columns are called column captions and this part of
column captions is called box-head.
iii) Row captions and Stub:
The heading for different rows are called row captions and this part of row captions is
called stub.
iv) Body of table:
The entries in different cells of columns and rows in a table are called body of the
table.
v) Prefatory notes:
The prefatory note is given after the title of the table. It is used to explain the contents
of the data.
vi) Footnotes:
The footnotes are given at the end of the table. It is used to explain the contents of the
data.

vii) Source note:


Source notes are given at the end of the table, which indicate the compiling agency,
publication, the data and page of distribution.

Frequency Distribution:
A frequency distribution is a compact form of data in a table which displays the
categories of observations according to their magnitudes and frequencies such that the
similar or identical numerical values are grouped together. The number of values falling in a
particular category is called the frequency of that category. It is denoted by t.

Steps for the construction of frequency distribution:

165
i. Calculate the range of the data, where
Range=R=Maximum value in the data-Minimum value in the data
ii. Calculate the number of classes by the following formula:
C = 1 + 3.3 log n
or C = √ (approximately)
iii. Decide about the width of the class by the following:
ℎ = (approximately)

Open-end classes:
By open-end classes in a frequency table, either the lower limit of the 1st class or the upper
limit of the last class is not a fixed number.

Class limits:
Each class is described by two numbers (the smaller number in the class limit is lower class
limit and the upper number in the class limit is called upper class limit). These number are
called class limits.

Class interval:
The class interval is the difference between the upper class boundary and the lower class
boundary of the same class (not the difference between the class limits).
Class frequency:
The number of observations falling in a class is class is called class frequency.

Class mark:
The class mark or the midpoint is the value which divide the class into two equal parts. It is
obtained by adding the lower and upper class limits or class boundaries of a class and
dividing the resulting total by 2.

Class boundaries:
A class boundary is located midway between the upper limit of a class and the lower limit of
the next class. The upper class boundary of a class coincides with the lower class boundary
of the next class.

Cumulative frequency:
It is obtained simply by adding the preceding frequencies including the frequency of that
class.

Relative frequency:
It is obtained by dividing the frequency of a class by the total frequency. It is generally
expressed as a percentage.

Example:
The marks of 30 students of BS class in subject of Statistics are as follows:
51, 57, 64, 66, 71, 56, 58, 67, 80, 82, 71, 72, 70, 64, 66, 43, 30, 33, 38, 40, 46, 49, 55, 59,
60, 66, 70, 88, 70, 72
166
Make a suitable frequency distribution. Also find class boundaries and cumulative
frequency.

Solution:
To construct a frequency distribution , we proceed as follow:
a. Range = R = Maximum value – Minimum value

Here
Maximum Value = 92
Minimum Value = 30 Range
= R = 92 – 30 = 62

b. No. of classes = C =1 + 3.3 log 30


C = 1 + 3.3 log 30 here n = 30 C
= 1 + 3.3 (1.4771)
C = 1 + 4.87443
C = 5.87443
C = 6 (approximately)
c. Class interval = h = R / C = 62 / 6 = 10 (approximately)
Frequency distribution of students-marks data is:

Class Limits Tally f Class boundaries Cumulative frequency


30-39 III 3 29.5-39.5 3
40-49 IIII 4 39.5-49.5 7
50-59 IIII I 6 49.5-59.5 13
60-69 IIII II 7 59.5-69.5 20
70-79 IIII II 7 69.5-79.5 27
80-89 III 3 79.5-89.5 30

7.4.3 Diagrammatical Presentation:


Simple bar diagram (chart):
When the data consists of a single component and have not large variations, then a simple
bar diagram is drawn. The first step in the construction is to arrange the data either in
ascending or descending order if the data do not relate to time. Equispaced vertical or
horizontal bards with moderate uniform width are then drawn. The length of bar is in
proportion to the actual data.

Pie Diagram:
Pie diagram is a circular diagram where the whole circle represent a ‘total’ and the
components of the total are represented by sectors of the pie diagram. Pie diagram is also
called sector diagram. It is a popular diagram and is drawn when the components are to be
shown for comparison. The total angle of the circle is 3600 and the total quantity to be
represented is taken equal to 3600. The angles for each components are calculated and these
angles are made in the circle to show different components.

167
Example: The data on Agricultural Product at current factor cost for Pakistan for the year
1983-84 is given below. Make a pie diagram to represent the data.
Sub-sector Product (million Rs.)
Major crops 46321
Minor crops 14971
Livestock 27096
Fishing 3082
Forestry 457
Source: Punjab Development Statistics, 1984
Solution: The necessary calculations to make the pie diagram are shown below and the
diagram is shown.

Sub-sectors Agriculture Product Angles of a sub-sectors


(million Rs.)
Major crops 46231 46231/91837 * 360 = 181.2
Minor crops 14971 14971/91837 *360 = 58
Livestock 27096 27096/91837 * 360 = 106.2
Fishing 3082 3082/91837 * 360 = 12.1
Forestry 457 457/91837 * 360 = 1.8
Total 91837 360

168
Product (million Rs.)

Major crops
Minor crops
Livestock
Fishing
Forestry

Simple Bar Diagrams


Simple bar diagrams are made to represent geographical, historical, numerical and the
qualitative data. The vertical or horizontal bars are made to represent the data when the
difference between different quantities is not very large. The different quantities may be
arranged in ascending or descending order but the time series data are not arranged. The
height of the bars is in proportion to the size of the quantities. All the bars are of uniform
width. The space between the bars should not be more than the width of the bars. The bars
should be neither very tall nor they should look small statured. Very thick or very thin bars
will not look attractive. Vertical bars are used for time series and quantitative data and
horizontal bars are used for geographical data.

Example:
Draw a simple bar diagram to represent the student enrolment at graduation level in Punjab
University, Lahore for 5 years.

Years 2015 2016 2017 2018 2019


Enrollment (thousands) 65 68 70 75 70

169
Multiple Bar Diagrams
Multiple bar diagrams are made when the data are in the form of different groups and each
group contains two or more pieces of information. Within the group, the differnet quantities
aare represented by adjacent bars. One set of bars is seperated from another by a suitable
space between them. Imports and exports of a country may be shown side by side for
different months. Birth and death rates of a country over a number of years may be shown
side by side for different years.

Example:
The following table gives the details of monthly budgets (in thousands) of two families.
Represent these figures through multiple bar diagram.

Items Family A Family B


Food 6 8
Clothing 1 1
House rent 4 5
Fuel and lighting 1 1
Miscellaneous 3 5
Total: 15 20

Solution:
98

76
54
3
Family A
Family B
2
10

7.4.4 Graphic Representation


Historigram:
A graph (line chart) showing changes in the value of one or more items from one period of
time to the next is known as the graph of a time series or historigram. In order to construct a

170
historigram, time is taken along x-axis and the values of the variable along y- axis. Points
are plotted corresponding to the data and are then joined to get the historigram.

Example:
Draw a graph of the following time series data of prices of a certain commodity:
Years 2010 2011 2012 2013 2014 2015 2016 2017 2018
Prices 32 64 29 77 91 102 110 115 120

Histogram
One of the most important and useful methods of presenting frequency distribution of
continuous data graphically is known as histogram. To construct histogram for a
frequency distribution class-boundaries are taken along x-axis and the frequencies along y-
axis. Rectangles are completed with class-intervals as the width and heights
proportional to the frequencies. The rectangles are adjacent to each other and there is no gap
between the rectangles.

Frequency Polygon:
If the mid points at the top of each rectangle in the histogram are joined by means of straight
lines, the resulting curves is called the frequency polygon. As the polygon is a many-sided
closed figure therefore we add extra classes at both ends of the frequency distribution with
zero frequencies.

Frequency Curve:
In frequency curve the points are not joined together by straight lines. The free-hand
drawing method of drawing curve is used and we get the frequency curve. We can draw
frequency curve on the frequency polygon or we can draw the curve on the separate sheet of
paper.

Cumulative Frequency Polygon or Ogive:

171
Ogive is the curve obtained when the cumulative frequencies of a distribution are
graphed against the upper class-boundaries. This curve is also known as cumulative
frequency polygon. In order to construct an ogive:

i. Compute the cumulative frequencies.


ii. Mark-off the class-interval along x-axis.
iii. Plot cumulative frequencies against the upper class-boundaries of the class-
intervals.
iv. Join the points with a smooth curve

Example:
For the following frequency distribution draw i) Histogram ii) Frequency polygon iii)
Cumulative frequency polygon (Ogive)

X 14 16 18 20 22
f 10 22 30 25 13

Solution:
X f Class boundaries (x-h/2)-(x+h/2) c.f
14 10 13-15 10
16 22 15-17 32
18 30 17-19 62
20 25 19-21 27
22 13 21-23 100

172
Cumulative Frequency Curve or Ogive

173
Self Assessment Questions:
Q. 1: The areas of the various continents/country of the world in millions of square
kilometers are given as:
Continent / country Area (millions of square kilometers)
Africa 30.3
Asia 26.9
Europe 4.9
North America 24.3
Oceania 8.5
South America 17.9
Russia 20.5

Graph the data by i) A bar chart. Ii) A pie chart.


Q.2: The following table gives the details of monthly budgets of two families. Represent
these figures by a suitable diagram.

Items Family A Family B


Food Rs. 6000 Rs. 8000
Clothing Rs. 1000 Rs. 1000
House rent Rs. 4000 Rs. 5000
Fuel and lighting Rs. 1000 Rs. 1000
Miscellaneous Rs. 3000 Rs. 5000
Total Rs. 15000 Rs. 20000

Q. 3: Draw a component bar chart to represent the following data:

Expenditure in thousands of rupees


Year Agriculture Industries Miscellaneous
1999 600 245 222
2000 675 263 230
2001 850 320 255
2002 825 375 300
2003 900 425 350

174
UNIT-8

MEASURES OF CENTRAL TENDENCY

Written by: Dr. Zahid Iqbal


Reviewed by: Dr. Muhammad Zakria

175
CONTENTS
Page

Introduction ........................................................................................................... 156

Objectives .......................................................................................................... 157


8.1 The Objects of Central Tendency ............................................................................. 158

8.2 Essential of a Good Average..................................................................................... 158

8.3 Characteristics ........................................................................................................... 158

8.4. Mean ........................................................................................................... 159

8.4 Mode ........................................................................................................... 159

8.5 Locating the Center of Your Data ............................................................................. 159

8.7 Measures of Central Tendency ................................................................................. 161

8.7.1 Arithmetic Mean ........................................................................................ 161


8.8 Median ........................................................................................................... 165
8.8.1 Merits and Demerits of Median ................................................................... 167
8.9 Mode ........................................................................................................... 168
8.9.1 Limitations of the Mode...................................................................................... 170
8.10 Skewed Distributions and the Mean and Median .................................................. 173
8.11 How do outliers influence the measures of central tendency? ............................... 177

8.12 What is a Quantile? ............................................................................................... 178

8.13 Quartiles ........................................................................................................... 181

8.14 Deciles ........................................................................................................... 182

8.15 Selecting a Suitable Measure of Central Tendency ................................................ 183

8.16 Geometric Mean...................................................................................................... 187

References ........................................................................................................... 188

176
INTRODUCTION
A measure of central tendency is a summary statistic that represents the center point or
typical value of a dataset. These measures indicate where most values in a distribution fall
and are also referred to as the central location of a distribution. You can think of it as the
tendency of data to cluster around a middle value. In statistics, the three most
common measures of central tendency are the mean, median and mode. Each of these
measures calculates the location of the central point using a different method.

A measure of central tendency is a single value that attempts to describe a set of data by
identifying the central position within that set of data. As such, measures of central
tendency are sometimes called measures of central location. They are also classed as
summary statistics. The mean (often called the average) is most likely the measure of central
tendency that you are most familiar with, but there are others, such as the median and the
mode.
Choosing the best measure of central tendency depends on the type of data you have. In this
chapter, we explore these measures of central tendency; show you how to calculate them,
and how to determine which one is the best for your data.
One of the important objectives of statistics is to find out various numerical values which
explain the inherent characteristics of a frequency distribution. The first of such measures is
averages. The averages are the measures which condense a huge unwieldy set of
numerical data into single numerical values which represent the entire distribution. The
inherent inability of the human mind to remember a large body of numerical data
compels us to few constants that will describe the data. Averages provide us the general
picture and given bird’s eye view of the huge mass of unwieldy numerical data. Averages
are the typical values around which other items of the distribution congregate. This value
lies between the centre of two extreme observations of the distribution and give us an idea
about the concentration of the values in the central part of the distribution. They are called
the measures of central tendency.
Averages are also called measures of location since they enable us to locate the position or
place of the distribution in question. Averages are statistical constants which enables us to
comprehend in a single value the significance of the whole group. An average value is a
single value within the range of the data that is used to represent all the values in that series.
Since an average is somewhere within the range of data, it is sometimes called a measure of
central value. An average is the most typical representative item of the group to which it
belongs and which is capable of revealing all important characteristics of that group or
distribution.

177
OBJECTIVES
The objective of the unit is to impart the understanding of the calculations and the
properties of measures of central tendency, including mean, mode, median etc. Upon
successful completion of this chapter, you will be able to:

1. use appropriate summary measures to describe different data sets.


2. compute the mean, median, and mode of a given set of data.
3. identify an outlier given a set of data.
4. identify the mode or modes of a data set for both quantitative and qualitative data.

178
8.1 The Objects of Central Tendency
The most important object of calculating an average or measuring central tendency is to
determine a single figure which may be used to represent a whole series involving
magnitudes of the same variable. Second object is that an average represents the entire data;
it facilitates comparison within one group or between groups of data. Thus, the performance
of the members of a group can be compared with the average performance of different
groups.

Third object is that an average helps in computing various other statistical measures such as
dispersion, skewness, kurtosis etc.

8.2 Essential of a Good Average


An average represents the statistical data and it is used for purposes of comparison, it must
possess the following properties.
1. It must be rigidly defined and not left to the mere estimation of the observer. If the
definition is rigid, the computed value of the average obtained by different persons
shall be similar.

2. The average must be based upon all values given in the distribution. If the item is not
based on all value it might not be representative of the entire group of data.

3. It should be easily understood. The average should possess simple and obvious
properties. It should be too abstract for the common people.

4. It should be capable of being calculated with reasonable care and rapidity.

5. It should be stable and unaffected by sampling fluctuations.

6. It should be capable of further algebraic manipulation.

8.3 Characteristics
1. Mean is computed using all the values in the data set.
2. Mean varies less for samples taken from the same population when compared to the
median or mode.
3. The Mean is unique for a data set. The mean may not be one of the data values in the
distribution.
4. Other statistics such as variance are computed using mean.
5. Mean is affected by the outliers present in the data set. Hence mean is not to be used
for data sets containing outliers.

179
Different methods of measuring “Central Tendency” provide us with different kinds of
averages. The following are the main types of averages that are commonly used:

8.4 Mean
(i) Arithmetic mean
(ii) Weighted mean
(iii) Geometric mean
(iv) Harmonic mean

8.5 Mode
Apart from the mean, median and mode are the two commonly used measures of central
tendency. The median is sometimes referred to as a measure of location as it tells us where
the data are. This article describes about median, mode and also the guidelines for selecting
the appropriate measure of central tendency.

8.6 Locating the Center of your Data


Most books that you’ll read about the mean, median and mode focus on how you
calculate each one. I’m going to take a slightly different approach to start out. My
philosophy is to help you intuitively grasp statistics by focusing on concepts.
Consequently, I’m going to start by illustrating the central point of several datasets
graphically—so you understand the goal. Then, we’ll move on to choosing the best
measure of central tendency for your data and the calculations.
The three distributions below represent different data conditions. In each distribution, look
for the region where the most common values fall. Even though the shapes and type of data
are different, you can find that central location. That’s the area in the distribution where the
most common values are located (Figure 1).

180
Figure 1
The relative position of the various measures of central tendency. (a) Normal distribution (b)
Positively (right) skewed distribution (c) Negatively (left) skewed distribution
As the graphs highlight, you can see where most values tend to occur. That’s the concept.
Measures of central tendency represent this idea with a value. Coming up, you’ll learn that
as the distribution and kind of data changes, so does the best measure of central tendency.
Consequently, you need to know the type of data you have, and graph it, before choosing a
measure of central tendency!

181
8.7 MEASURES OF CENTRAL TENDENCY
MEASURES OF CENTRAL TENDENCY
Three of the many ways to measure central tendency are:

1. Mean the average of the data

2. Median the middle value of the ordered data

3. Mode the value that occurs most often in the data

In most research experimental situations, examination of all members of a population is not


typically conducted due to the cost and time required. Instead, we typically examine a
random sample, i.e., a representative subset of the population.

8.7.1 Arithmetic Mean


The mean is the arithmetic average, and it is probably the measure of central tendency that
you are most familiar. Calculating the mean is very simple. You just add up all of the values
and divide by the number of observations in your dataset.
The calculation of the mean incorporates all values in the data. If you change any value, the
mean changes. However, the mean doesn’t always locate the center of the data
accurately. Observe the graphs above where the mean are displayed in the distributions. In a
symmetric distribution, the mean locates the center accurately.
However, in a skewed distribution, the mean can miss the mark. In the figure 1 above, it is
starting to fall outside the central area. This problem occurs because outliers have a
substantial impact on the mean. Extreme values in an extended tail pull the mean away from
the center. As the distribution becomes more skewed, the mean is drawn further away from
the center. Consequently, it’s best to use the mean as a measure of the central tendency
when you have a symmetric distribution.
Descriptive measures of population are parameters. Descriptive measures of a sample are
statistics. For example, a sample mean is a statistic and a population mean is a
parameter. The sample mean is usually denoted by :

182
sum of the values
Mean = =
the number of values

= (x1+ x2+…+xn) / n =∑ =1

where n is the sample size and xi are the measurements. One may need to use the sample
mean to estimate the population mean since usually only a random sample is drawn and we
don't know the population mean.

Notation!
What if we say we used yi for our measurements instead of xi? Is this a problem? No.
The formula would simply look like this:

=(y1+y2+…+yn)/n= ∑ =1

The formulas are exactly the same. The letters that you select to denote the
measurements are up to you. For instance, many textbooks use x instead of y to denote the
measurements.
The point is to understand how the calculation that is expressed in the formula works. In
this case, the formula is calculating the mean by summing all of the observations and
dividing by the number of observations.
There is some notation that you will come to see as standards, i.e., n will always equal
sample size. We will make a point of letting you know what these are. However, when it
comes to the variables, these labels can (and do) vary.
For example, in one study x may be used to denote weight and y may be used to denote
height, (or the reverse may be used!), but n will always be used to denote sample size in
each case.

When to use the mean: Symmetric distribution, Continuous data


The arithmetic mean is the most common measure of central tendency. It is simply the sum
of the numbers divided by the number of numbers. The symbol "μ" is used for the mean of a
population. The symbol "μ" is used for the mean of a sample. The formula for μ is shown
below:
μ ΣX/N
where ΣX is the sum of all the numbers in the population and N is the number in the
population.
The formula for X is essentially identical:
=∑ /n
=1

183
where ∑ is the sum of all the numbers in the sample and n is the number of numbers
=1
in the sample.
As an example, the mean of the numbers 1, 2, 3, 6, 8 is 20/5 = 4 regardless of whether the
numbers constitute the entire population or just a sample from the population.
Table 4.1 shows the number of touchdown (TD) passes thrown by each of the 31 teams in
the National Football League in the 2017 season. The mean number of touchdown passes
thrown is 20.4516 as shown below.
μ = ΣX/N
= 634/31
= 20.4516
Table 8.1. Number of touchdown passes.
37 33 33 32 29 28 28 23 22 22 22 21 21 21 20 20 19 19 18 18 18 18 16 15 14 14 14 12 12 9 6
Although the arithmetic mean is not the only "mean", it is by far the most commonly used.
Therefore, if the term "mean" is used without specifying whether it is the arithmetic mean,
the geometric mean, or some other mean, it is assumed to refer to the arithmetic mean.

Question 8.1: The following data set is the worth(in billions of dollars) of 10
hypothetical wealthy men. Find the mean worth of these top 10 rich men.
12.6, 13.7, 18.0, 18.0, 18.0, 20.0, 20.0, 41.2, 48.0, 60.0
Solution:
Given data,
12.6, 13.7, 18.0, 18.0, 18.0, 20.0, 20.0, 41.2, 48.0, 60.0
Mean of the data set,

= (12.6+13.7+18+18+18+20+20+41.2+48+60)/10
= 269.5/10 = 26.95
Question 8.2: Compute the mean for the distribution given below:

Value Frequency
x f
20 2
29 4
30 4
39 3
44 2

184
Solution:
The frequency table is redone adding one more column f * x;
Value Frequenc
f*x
x y f
20 2 40
29 4 116
30 4 120
39 3 117
44 2 88
∑f 15 ∑fx = 481

Mean of the distribution:


∑fx/∑f = 481/15
=32.066667≈32.07≈32.1
When not to use the mean
The mean has one main disadvantage: it is particularly susceptible to the influence of
outliers. These are values that are unusual compared to the rest of the data set by being
especially small or large in numerical value. For example, consider the wages of staff at a
factory below:
Staff 1 2 3 4 5 6 7 8 9 10

Salary 15000 18000 16000 14000 15000 15000 12000 17000 90000 95000

The mean salary for these ten staff is Rs 30700. However, inspecting the raw
suggests that this mean value might not be the best way to accurately reflect the typical
salary of a worker, as most workers have salaries in the 12000 to 18000 range. The mean
is being skewed by the two large salaries 0f 90000 and 95000. Therefore, in this situation,
we would like to have a better measure of central tendency. As we will find out later,
taking the median would be a better measure of central tendency in this situation.
Another time when we usually prefer the median over the mean (or mode) is when our
data is skewed (i.e., the frequency distribution for our data is skewed). If we consider the
normal distribution - as this is the most frequently assessed in statistics - when the data is
perfectly normal, the mean, median and mode are identical. Moreover, they all represent
the most typical value in the data set. However, as the data becomes skewed the mean
loses its ability to provide the best central location for the data because the skewed data is
dragging it away from the typical value. However, the median best retains this position
and is not as strongly influenced by the skewed values. This is explained in more detail in
the skewed distribution section later in this guide.
Arithmetic Meanor simply mean is a value obtained by adding together all the items and
by dividing the total by the number of items. It is also called average. It is the most
popular and widely used measure for representing the entire data by one value.

185
Arithmetic mean may be either:

1. Simple arithmetic mean, or


2. Weighted arithmetic mean.

8.7.2 Properties of Arithmetic Mean


1. The sum of deviations of the items from the arithmetic mean is always zero i.e.

( − )=0
=1
2. The Sum of the squared deviations of the items from A.M. is minimum, which is less
than the sum of the squared deviations of the items from any other values.
3. If each item in the series is replaced by the mean, then the sum of these
substitutions will be equal to the sum of the individual items.
To solve different types of problems on average we need to follow the properties of
arithmetic mean.
Here we will learn about all the properties and proof the arithmetic mean showing the step-
by-step explanation.

8.7.3 Merits and Demerits of Arithmetic Mean


Merits of Arithmetic mean.:
1. It is simple to understand and easy to calculate.
2. It is affected by the value of every item in the series.
3. It is rigidly defined.
4. It is capable of further algebraic treatment.
5. It is calculated value and not based on the position in the series.

Demerits of arithmetic mean.


1. It is affected by extreme items i.e., very small and very large items.
2. It can hardly be located by inspection.
3. In some cases arithmetic mean. does not represent the actual item. For example,
average patients admitted in a hospital is 10.7 per day.
4. Arithmetic mean. is not suitable in extremely asymmetrical distributions.

8.8 Median
Median is the value which occupies the middle position when all the observations are
arranged in an ascending/descending order. It divides the frequency distribution exactly into
two halves. Fifty percent of observations in a distribution have scores at or below the
median. Hence median is the 50th percentile. Median is also known as ‘positional
average’.
It is easy to calculate the median. If the number of observations are odd, then (n + 1)/2th
observation (in the ordered set) is the median. When the total number of observations are
even, it is given by the mean of n/2th and (n/2 + 1)th observation.

186
When to use the median: Skewed distribution, Continuous data, Ordinal data The median is
the middle score for a set of data that has been arranged in order of magnitude. The median
is less affected by outliers and skewed data. In order to calculate the median, suppose we
have the data below:

65 55 89 56 35 14 56 55 87 45 92

We first need to rearrange that data into order of magnitude (smallest first):

14 35 45 55 55 56 56 65 87 89 92

Our median mark is the middle mark - in this case, 56 (highlighted in bold). It is the middle
mark because there are 5 scores before it and 5 scores after it. This works fine when you
have an odd number of scores, but what happens when you have an even number of scores?
What if you had only 10 scores? Well, you simply have to take the middle two scores and
average the result. So, if we look at the example below:

65 55 89 56 35 14 56 55 87 45

We again rearrange that data into order of magnitude (smallest first):

14 35 45 55 55 56 56 65 87 89

Only now we have to take the 5th and 6th score in our data set and average them to get a
median of 55.5.
Question 8.3: The number of rooms in 11 hotels in a city is as follows:
380, 220, 555, 678, 756, 823, 432, 367, 546, 402, 347.

Find the median?

Solution:
The data is first arranged starting from the lowest as follows:
220, 347, 367, 380, 402, 432, 546, 555, 678, 756, 823.

As the number of data elements 11 is an odd number, there is only one middle value in
the data array, which is the 6th.

=> The value of data in 6th position = 432.

Hence the mean number of Hotel rooms in the city = 432.

Question 8.4: Find the median of the given data: Value


Frequency
X f

187
20 2

29 4

30 4

39 3

44 2

Solution:

Value Cumulative
x Frequency f * x frequency
f
20 2 40 2

29 4 116 2+4= 6

30 4 120 6 + 4 = 10

39 3 117 10 + 3 = 13

44 2 88 13 + 2 = 15 =∑

∑f = 15 ∑fx = 481

=> ∑f = 15 items,

Median = 15/2 =7.5≈8


The 8th item in the ordered data array will be the median. The 8 item will be included in the
cumulative frequency 10. Hence the median of the distribution is the x value
corresponding to cumulative frequency 10 which reads as 30.
=> Median of the data = 30.

8.8.1 Merits and Demerits of Median


Merits of Median
1. They are easy to determine especially in case of the individual and discrete series.

2. They do not need all the data relating to a series like the mathematical averages viz.
AM… G.M. and H.M.

188
3. They can be directly determined in case of an open end series without locating the
lower limit of the lowest class, and the upper limit of the highest class.

4. They are useful in the computation of the measures of dispersion and skewness.

5. They give an idea about the character of a frequency distribution i.e. whether a series
is symmetric, or asymmetric can be known by measuring their distance from the
Median.

6. They are not affected very much by the extreme values of a series.

7. They can be located both graphically and tabularly.

Demerits of the Median


1. These averages are not easily understood by a common man.
2. The determination of their values in case of continuous series becomes
cumbersome as it involves application of the formula of interpolation.
3. They are not based on all the observations of a series.
4. They need the rearrangement of series in the ascending order if given otherwise.
5. They do not study the entire data. For example, Q1, studies only, first 25%, Q2 only
first 50%, and Q3 only first 75% of the data.
6. They are not capable of further algebraic treatment except in the computation of
quartile deviation and coefficient of skewness.
7. They are affected very much by fluctuatin of sampling.
8. They are influenced much by the number of items rather than their values.
8.9 Mode
Consider this dataset showing the retirement age of 11 people, in whole years:
54, 54, 54, 55, 56, 57, 57, 58, 58, 60, 60
This table shows a simple frequency distribution of the retirement age data.

Age 54 55 56 57 58 59 60

Frequency 3 1 1 2 2 1 2

The most commonly occurring value is 54, therefore the mode of this distribution is 54
years.

Characteristics
Easiest average to determine and it is used when the most typical value is required as the
central value.

189
Found for nominal data set as well.
Mode need not be a unique measure. A distribution can have more than one mode or no
mode at all.
Question: Find the mode of a numerical data set

109 112 109 110 109 107 104 104 104 111 111 109 109 104 104

Solution:
Given data,

109 112 109 110 109 107 104 104 104 111 111 109 109 104 104

Total number of element = 15


Among the 15 data elements the values 104 and 109 both occur five times which are hence
the modes of the data set.

Question : The following table shows the sport activities of 2400 students.

Sport.

190
FREQUENCY
SPORTS
SWIMMING 423
TENNIS 368
GYMNASTICS 125
BASKET BALL 452
BASE BALL 380
ATHLETICS 275

NONE 377

Solution:

From the given table:


For grouped data the class with the highest frequency is called the Modal class. The
category with the longest column in the bar graph represents the mode of data set.

Basketball has the highest frequency of 452. Hence Basketball is the mode of the sport
activities.
Mode is defined as the value that occurs most frequently in the data. Some data sets do not
have a mode because each value occurs only once. On the other hand, some data sets can
have more than one mode. This happens when the data set has two or more values of equal
frequency which is greater than that of any other value. Mode is rarely used as a summary
statistic except to describe a bimodal distribution. In a bimodal distribution, the taller peak is
called the major mode and the shorter one is the minor mode.
When to use the mode: Categorical data, Ordinal data, Count data, Probability
Distributions

8.9.1 Limitations of the Mode


The are some limitations to using the mode. In some distributions, the mode may not reflect
the centre of the distribution very well. When the distribution of retirement age is ordered
from lowest to highest value, it is easy to see that the centre of the distribution is 57 years,
but the mode is lower, at 54 years.

191
54, 54, 54, 55, 56, 57, 57, 58, 58, 60, 60

It is also possible for there to be more than one mode for the same distribution of data, (bi-
modal, or multi-modal). The presence of more than one mode can limit the ability of the
mode in describing the centre or typical value of the distribution because a single value to
describe the centre cannot be identified.
In some cases, particularly where the data are continuous, the distribution may have no
mode at all (i.e. if all values are different).
In cases such as these, it may be better to consider using the median or mean, or group the
data in to appropriate intervals and find the modal class.
The mode is the most frequent score in our data set. On a histogram it represents the highest
bar in a bar chart or histogram. You can, therefore, sometimes consider the mode as being
the most popular option. An example of a mode is presented below:

Normally, the mode is used for categorical data where we wish to know which the most
common category, as illustrated below is:

192
We can see above that the most common form of transport, in this particular data set, is the
bus. However, one of the problems with the mode is that it is not unique, so it leaves us with
problems when we have two or more values that share the highest frequency, such as below:

We are now stuck as to which mode best describes the central tendency of the data. This is
particularly problematic when we have continuous data because we are more likely not to
have any one value that is more frequent than the other. For example, consider
measuring 30 peoples' weight (to the nearest 0.1 kg). How likely is it that we will find two
or more people with exactly the same weight (e.g., 67.4 kg)? The answer, is probably very
unlikely - many people might be close, but with such a small sample (30 people) and a large
range of possible weights, you are unlikely to find two people with exactly the same weight;
that is, to the nearest 0.1 kg. This is why the mode is very rarely used with continuous data.

193
Another problem with the mode is that it will not provide us with a very good measure of
central tendency when the most common mark is far away from the rest of the data in the
data set, as depicted in the diagram below:

In the above diagram the mode has a value of 2. We can clearly see, however, that the mode
is not representative of the data, which is mostly concentrated around the 20 to 30 value
range. To use the mode to describe the central tendency of this data set would be
misleading.

8.10 Skewed Distributions and the Mean and Median


We often test whether our data is normally distributed because this is a common
assumption underlying many statistical tests. An example of a normally distributed set of
data is presented below:

Wh
en you have a normally distributed sample you can legitimately use both the mean and the
median as your measure of central tendency. In fact, in any symmetrical distribution
the mean, median and mode are equal. However, in this situation, the mean is widely
preferred as the best measure of central tendency because it is the measure that includes all
the values in the data set for its calculation, and any change in any of the scores will affect
the value of the mean. This is not the case with the median or mode.

194
However, when our data is skewed, for example, as with the right-skewed data set below:

we find that the mean is being dragged in the direct of the skew. In these situations, the
median is generally considered to be the best representative of the central location of the
data. The more skewed the distribution, the greater the difference between the median and
mean, and the greater emphasis should be placed on using the median as opposed to the
mean. A classic example of the above right-skewed distribution is income (salary), where
higher-earners provide a false representation of the typical income if expressed as a mean
and not a median.
If dealing with a normal distribution and tests of normality show that the data is non-
normal, it is customary to use the median instead of the mean. However, this is more a rule
of thumb than a strict guideline. Sometimes, researchers wish to report the mean of a
skewed distribution if the median and mean are not appreciably different (a subjective
assessment), and if it allows easier comparisons to previous research to be made.
Summary of when to use the mean, median and mode

Please use the following summary table to know what the best measure of central
tendency is with respect to the different types of variable.

Type of Variable Best measure of central tendency

Nominal Mode

Ordinal Median

195
Interval/Ratio (not skewed) Mean

Interval/Ratio (skewed) Median

How does the shape of a distribution influence the Measures of Central Tendency?

8.10.1 Symmetrical distributions:


When a distribution is symmetrical, the mode, median and mean are all in the middle of the
distribution. The following graph shows a larger retirement age dataset with a
distribution which is symmetrical. The mode, median and mean all equal 58 years.

8.10.2 Skewed Distributions:


When a distribution is skewed the mode remains the most commonly occurring value, the
median remains the middle value in the distribution, but the mean is generally ‘pulled’ in the
direction of the tails. In a skewed distribution, the median is often a preferred measure of
central tendency, as the mean is not usually in the middle of the distribution.

A distribution is said to be positively or right skewed when the tail on the right side of the
distribution is longer than the left side. In a positively skewed distribution it is
common for the mean to be ‘pulled’ toward the right tail of the distribution. Although there
are exceptions to this rule, generally, most of the values, including the median value, tend to
be less than the mean value.
The following graph shows a larger retirement age data set with a distribution which is right
skewed. The data has been grouped into classes, as the variable being measured (retirement
age) is continuous. The mode is 54 years, the modal class is 54-56 years, the median is 56
years and the mean is 57.2 years.

196
A distribution is said to be negatively or left skewed when the tail on the left side of the
distribution is longer than the right side. In a negatively skewed distribution, it is
common for the mean to be ‘pulled’ toward the left tail of the distribution. Although there
are exceptions to this rule, generally, most of the values, including the median value, tend to
be greater than the mean value.
The following graph shows a larger retirement age dataset with a distribution which left
skewed. The mode is 65 years, the modal class is 63-65 years, the median is 63 years and
the mean is 61.8 years.

197
8.11 How Do Outliers Influence the Measures of Central Tendency?
Outliers are extreme or atypical data value(s) that are notably different from the rest
of the data.
It is important to detect outliers within a distribution, because they can alter the results of the
data analysis. The mean is more sensitive to the existence of outliers than the median or
mode.
Consider the initial retirement age dataset again, with one difference; the last observation of
60 years has been replaced with a retirement age of 81 years. This value is much higher than
the other values, and could be considered an outlier. However, it has not changed the middle
of the distribution, and therefore the median value is still 57 years.
54, 54, 54, 55, 56, 57, 57, 58, 58, 60, 81
As the all values are included in the calculation of the mean, the outlier will influence the
mean value.

(54+54+54+55+56+57+57+58+58+60+81 = 644), divided by 11 = 58.5 years. In

this distribution the outlier value has increased the mean value.
Despite the existence of outliers in a distribution, the mean can still be an appropriate
measure of central tendency, especially if the rest of the data is normally distributed. If the
outlier is confirmed as a valid extreme value, it should not be removed from the dataset.
Several common regression techniques can help reduce the influence of outliers on the mean
value.

8.12 What is a Quantile?


Quantiles are statistics that describe various subdivisions of a frequency distribution into
equal proportions. The simplest division that can be envisioned is into two equal halves and
the quantile that does this, the median value of the variate, is used also as a measure of
central tendency for the distribution.
When division is into four parts the values of the variate corresponding to 25%, 50% and
75% of the total distribution are called quartiles. The difference between the 1st and 3rd
quartiles is called the inter-quartile range. It embraces the central 50% of the distribution
and gives a measure of the dispersion of the distribution. The 2nd quartile is just the median
under another name.
Other quantiles that may be encountered include quintiles (distribution divided at 20%,
40%, 60% and 80%), deciles (inter-decile range from 1st decile to 9th decile holds 80% of
the distribution) and percentiles.
The word “quantile” comes from the word quantity. In simple terms, a quantile is where a
sample is divided into equal-sized, adjacent, subgroups (that’s why it’s sometimes
called a “fractile“). It can also refer to dividing a probability distribution into areas of equal
probability.

198
Quantiles are points in a distribution that relate to the rank order of values in that
distribution.
For a sample, you can find any quantile by sorting the sample. The middle value of the
sorted sample (middle quantile, 50th percentile) is known as the median. The limits are the
minimum and maximum values. Any other locations between these points can be described
in terms of centiles/percentiles.
Centiles/percentiles are descriptions of quantiles relative to 100; so the 75th percentile
(upper quartile) is 75% or three quarters of the way up an ascending list of sorted values of a
sample. The 25th percentile (lower quartile) is one quarter of the way up this rank order.
Percentile rank is the proportion of values in a distribution that a particular value is greater
than or equal to. For example, if a pupil is taller than or as tall as 79% of his classmates then
the percentile rank of his height is 79, i.e. he is in the 79th percentile of heights in his class.

199
Definition
The median is a quantile; the median is placed in a probability distribution so that exactly
half of the data is lower than the median and half of the data is above the median. The
median cuts a distribution into two equal areas and so it is sometimes called 2-quantile.
Quartiles are also quantiles; they divide the distribution into four equal
parts. Percentiles are quantiles that divide a distribution into 100 equal parts
and deciles are quantiles that divide a distribution into 10 equal parts.
Some authors refer to the median as the 0.5 quantile, which means that the proportion 0.5
(half) will be below the median and 0.5 will be above it. This way of defining quartiles
make sense if you are trying to find a particular quantile in a data set (i.e. the median). Use
the following formula to estimate the ith observation:
ith observation = q (n + 1)
where q is the quantile, the proportion below the ith value that you are looking for n
is the number of items in a data set
Another numerical descriptor for the span of frequency distributions is
the quantile (or fractile). A p-quantile is defined as the x-value of the distribution which
includes p*N observations, with 0<p<1 and N being the number of observations.
An example may clarify this: the 0.1-quantile (0.1-fractile) of the distribution shown below
is 14.6, as it includes 10 % of all observations (starting from the left).

The p- quantile is also called P-percentile, with P=p*100; a 0.12-quantile could be


designated as 12-percentile, as well.
There are a few special quantiles which have their own definition: Quartiles refer to
quarters of the distribution, deciles to the tenth part:

200
1st quartile = 0.25-quantile

2ndquartile = 0.5-quantile = 5. decile = median

3rdquartile = 0.75 quantile

How to Find Quantiles?


Sample question: Find the number in the following set of data where 20 percent of values
fall below it, and 80 percent fall above:
1 3 5 6 9 11 12 13 19 21 22 32 35 36 45 44 55 68 79 80 81 88 90 91 92 100 112 113 114
120 121 132 145 146 149 150 155 180 189 190
Step 1: Order the data from smallest to largest. The data in the question is already in
ascending order.
Step 2: Count how many observations you have in your data set this particular data set has
40 items.
Step 3: Convert any percentage to a decimal for “q”. We are looking for the number where
20 percent of the values fall below it, so convert that to 0.2.

Step 4: Insert your values into the formula: ith observation = q (n + 1)

ith observation = 0.2 (40 + 1) = 8.2 ≈8th value


One way of obtaining resistant statistics is to use the empirical quantiles (percentiles/
fractiles). The quantile (this term was first used by Kendall, 1940) of a distribution is the
number such that a proportion of the values are less than or equal to . For
example, the 0.25 quantile (also referred to as the 25th percentile or
lower quartile) is the value such that 25% of all the values fall below that value. Empirical
quantiles can be most easily constructed by sorting (ranking) the data into
ascending order to obtain a sequence of order statistics .
The 'th quantile is then obtained by taking the rank 'th order

statistic (or an average of neigbouring values if is not integer):

where

is the probability and is the greatest integer not


exceeding . Note that the empirical probability is only

defined at discrete values - quantiles for other values of can be obtained either by
interpolation

201
( ) or by extrapolation (
or ). The use of rather than in the
denominator of

The pth quantile


The pth quantile is that value which demarcates a given proportion of a set of values.
• The median is a single value, the 0.5th quantile, which divides an ordered set
into 2 equal groups.

• A quartiles is one of three values which divide an ordered set into 4 equal sized
groups.

• A decile is one of nine values which divide an ordered set into 10 equal groups.

• A percentile is one of 99 values which demarcate an ordered set into 100 equal
groups.

8.13 Quartiles
The median of a distribution splits the data into two equally-sized groups. In the same way,
the quartiles are the three values that split a data set into four equal parts. Note that the
'middle' quartile is the median.
The upper quartile describes a 'typical' mark for the top half of a class and the lower quartile
is a 'typical' mark for the bottom half of the class.
The quartiles are closely related to the histogram of a data set. Since area equals the
proportion of values in a histogram, the quartiles split the histogram into four
approximately equal areas.
(The relationship is only approximate if the quartiles do not coincide with histogram bin
boundaries.)

202
8.14 Deciles
In a similar way, the deciles of a distribution are the nine values that split the data set into
ten equal parts.
You should not try to calculate deciles from small data sets -- a single class of marks is too
small to get useful values since the extreme deciles are very variable. However the deciles
can be useful descriptions for larger data sets such as national distributions for marks from
standard tests.
The diagram below shows a jittered dot plot of 60 marks from an exam.
Click on the areas between the quartiles and verify that the quartiles split the students into
four groups of 15.
Use the pop-up menu to display deciles for the data, and verify that the 9 deciles split the
students into 10 groups, each containing 6 students.
Deciles for the distribution and for individual students
The term 'deciles' is used in two different contexts. It is confusing that the same word is
used in both ways, so be careful!

When applied to a distribution (a large group of marks), there are nine deciles, each of
which is a mark.

203
A student whose mark is below the first docile is said to be in decile 1. Similarly, a student
whose marks is between the first and second deciles is in decile 2, ... and a student whose
marks is above the ninth decile is in decile 10. When applied to individual students, the term
'decile' is therefore a number between 1 and 10.
For example, the histogram below shows the distribution of marks in a test (out of 60) that
was attempted by 600 students. Each student's mark is represented by a square in the
histogram.

The nine deciles split the students into 10 groups of 60.


The first decile is 17.5 so the weakest tenth of the students in the class had a mark below
this. This decile therefore summarises the performance of the weakest students.
Students with marks below 17.5 are said to be in decile 1. Those with marks between 17.5
and 26.5 are in decile 2, and so on, up to students with marks higher than 54.5 who are in
decile 10.

8.15 Selecting a Suitable Measure of Central Tendency


Mean is generally considered the best measure of central tendency and the most
frequently used one. However, there are some situations where the other measures of central
tendency are preferred.
Median is preferred to mean when
1. There are few extreme scores in the distribution.
2. Some scores have undetermined values.
3. There is an open ended distribution.
4. Data are measured in an ordinal scale.

5. Mode is the preferred measure when data are measured in a nominal scale.
Geometric mean is the preferred measure of central tendency when data are
measured in a logarithmic scale.[8]

Which is Best—the Mean, Median, or Mode?

204
When you have a symmetrical distribution for continuous data, the mean, median, and mode
are equal. In this case, analysts tend to use the mean because it includes all of the data in the
calculations. However, if you have a skewed distribution, the median is often the best
measure of central tendency.
When you have ordinal data, the median or mode is usually the best choice. For
categorical data, you have to use the mode.
In cases where you are deciding between the mean and median as the better measure of
central tendency, you are also determining which types of statistical hypothesis testsare
appropriate for your data—if that is your ultimate goal. I have written an article that
discusses when to use parametric (mean) and nonparametric (median) hypothesis
testsalong with the advantages and disadvantages of each type.
Question: Consider the data set, 5, 19, 19, 20, 21, 23, 23, 23, 24 , 25. Find Mean, Median
and Mode of the Data.
Solution:
The value 5 is an outlier of the data as it is too less than the other values in the
distribution.
Let us calculate the central values for the data set either by including and excluding 5.

Step 1:
The data set is 19, 19, 20, 21, 23, 23, 23, 24 , 25
Mean
x¯=(19+19+20+21+23+23+23+24+25)/9=197/9
x¯=21.89

=> Mean = 21.89


Median = 23 Mode
= 23
Step 2:

For the data including the outlier 5, 19, 19, 20, 21, 23, 23, 23, 24 , 25

Mean
x¯=(5+19+19+20+21+23+23+23+24+25)/10=202/10
x¯=20.2

=> Mean = 20.2

Median = (21+23)/2 = 22

Mode = 23

Step 3:
Comparing the values of mean, median and mode found in step 1 and step 2, the mean is
most affected and mode is least affected by the inclusion of the outlier value 5.

205
EXERCISES
1. What is the mean of the following numbers? 10, 39, 71, 39, 76, 38, 25
a. 42 b. 39 c. 42.5 d. 35.5
2. What number would you divide by to calculate the mean of 3, 4, 5, and 6?
a. 6 b. 3 c. 5 d. 4
3.What measure of central tendency is calculated by adding all the values and dividing
the sum by the number of values? a. Median b. Mean c. Mode d.
Typical value
4. The mean of four numbers is 71.5. If three of the numbers are 58, 76, and 88, what is
the value of the fourth number?
a. 64 b. 60 c. 76 d. 82
5. Determine the mean of the following set of numbers: 40, 61, 95, 79, 9, 50, 80, 63, 109,
42
6. The mean weight of five complete computer stations is 167.2 pounds. The weights of
four of the computer stations are 158.4 pounds, 162.8 pounds, 165 pounds, and 178.2
pounds respectively. What is the weight of the fifth computer station?
7. The mean width of 12 iPads is 5.1 inches. The mean width of 8 Kindles is 4.8 inches.
a. What is the total width of the iPads? b. What is the total width of the Kindles?
c. What is the mean width of the 12 iPads and 8 Kindles?
8. The following data represent the number of pop-up advertisements received by 10
families during the past month. Calculate the mean number of advertisements received by
each family during the month. 43 37 35 30 41 23 33 31 16 21
9. The following table of grouped data represents the weight (in pounds) of 100 computer
towers. Calculate the mean weight for a computer.
Weight (pounds) Number of Computers
3-5 8
5-7 25
7-9 45
9 - 11 18
11 – 13 4
10. A group of customer service surveys were sent out at random. The scores were 90, 50,
70, 80, 70, 60, 20, 30, 80, 90, and 20. Find the mean score.
11. What is the median of the following numbers? 10, 39, 71, 42, 39, 76, 38, 25
a. 42.5 b. 39 c. 42 d. 35.5
12. The front row in a movie theatre has 23 seats. If you were asked to sit in the seat that
occupied the median position, in which seat would you have to sit?
a. 1 b. 11 c. 23 d. 12

13. What is the median score achieved by a student who recorded the following scores on 10
math quizzes? 68, 55, 70, 62, 71, 58, 81, 82, 63, 79
a. 68 b. 71 c. 69 d. 79
14. A set of four numbers that begins with the number 32 is arranged from smallest to
largest. If the median is 35, which of the following could possibly be the set of numbers?
a. 32, 32, 36, 38 b. 32, 35, 38, 41 c. 32, 34, 36, 35 d. 32, 36, 40, 44

206
15. The number of service upgrades sold by each of 30 employees is as follows: 32, 6, 21,
10, 8, 11, 12, 36, 17, 16, 15, 18, 40, 24, 21, 23, 24, 24, 29, 16, 32, 31, 10, 30, 35, 32, 18, 39,
12, 20 What is the median number of service upgrades sold by the 30 employees?
a. 18 b. 21 c. 24 d. 32
16. Whichof the following measures can be determined for quantitative data?
a. Mean b. Median c. Mode d. All of these
17. Which of the following measures can be calculated for qualitative data?
a. Mean b. Median c. Mode d. All of these
18. What is the term used to describe the distribution of a data set with one mode?
a. Multimodal b. Unimodal c. Nonmodal d. Bimodal
19. What is the mode of the following numbers? 12, 11, 14, 10, 8, 13, 11, 9
a. 11 b. 10 c. 14 d. 8
20. Which of the followingmeasures can have more than one value for a set of data?
a. Median b. Mode c. Mean d. None of these
21. What are the modes of the following sets of numbers?
a. 3, 13, 6, 8, 10, 5, 6 b. 12, 0, 15, 15, 13, 19, 16, 13, 16, 16
22. A student recorded her scores on weekly math quizzes that were marked out of a
possible 10 points. Her scores were as follows: 8, 5, 8, 5, 7, 6, 7, 7, 5, 7, 5, 5, 6, 6, 9, 8, 9, 7,
9, 9, 6, 8, 6, 6, 7 What is the mode of her scores on the weekly math quizzes?
23. What is the mode of the following numbers, and what word can be used to describe the
distribution of the data set? 5, 4, 10, 3, 3, 4, 7, 4, 6, 5, 11, 9, 5, 7
24. The temperature in o F on 20 days during the month of June was as follows: 70 o F, 76o
F, 76 o F, 74o F, 70o F, 70 o F, 72o F, 74 o F, 78o F, 80o F, 74 o F, 74o F, 78 o F, 76o F,
78o F, 76 o F, 74o F, 78 o F, 80o F, 76o F What is the mode of the temperatures for the
month of June?

Question 1: Find the mean, median and mode of the given data.
10, 12, 34, 34, 45, 23, 42, 36, 3
Question 3: Find the median of the distribution,
223, 227, 240, 211, 212, 209, 211, 213, 240, 229.
Website developer has been working on programing and updating a Web site for his
company for the past 15 months. The following numbers represent the number of hours
Website developer has worked on this Web site for each of the past 7 months:
24, 25, 31, 50, 53, 66, 78
What is the mean, median and mode number of hours that Website developer worked on this
Web site each month?
Mark operates Technology Metro Cash & Carry, a Web site service that employs 8
people. Find the mean age of his workers if the ages of the employees are as follows:
55, 63, 34, 59, 29, 46, 51, 41

8.16 Geometric Mean

207
The Geometric Mean is calculated by taking the nth root of the product of a set of data. x

= n ∏n x i
i=1

For example, if the set of data was:

1,2,3,4,5
The geometric mean would be calculated:

G.M=5√1 2 3 4 5 = 5√120 = 2.61


Of course, with large n this can be difficult to calculate. Taking advantage of two
properties of the logarithm:

√ 1 2 ……. )= 1 ∑I=1 log(x i )


( n
n

We find that by taking the logarithmic transformation of the geometric mean, we get:

log(a.b) = log (a) + log (b) and log(a)n = n log (a)


Which leads us to the equation for the geometric mean:

=exp {1 ∑ =1 log( )}
When to use the geometric mean
The arithmetic mean is relevant at any time several quantities add together to produce a
total. The arithmetic mean answers the question, "if all the quantities had the same value,
what would that value have to be in order to achieve the same total?"

In the same way, the geometric mean is relevant any time several quantities multiply
together to produce a product. The geometric mean answers the question, "if all the
quantities had the same value, what would that value have to be in order to achieve the same
product?"

For example, suppose you have an investment which returns 10% the first year, 50% the
second year, and 30% the third year. What is its average rate of return? It is not the
arithmetic mean, because what these numbers mean is that on the first year your
investment was multiplied (not added to) by 1.10, on the second year it was multiplied by
1.50, and the third year it was multiplied by 1.30. The relevant quantity is the geometric
mean of these three numbers.

It is known that the geometric mean is always less than or equal to the arithmetic mean.

208
REFERENCES
Bluman, A.G. (2004). Elementary Statistics. A Step by Step Approach. Fifth Edition.
McGraw-Hill Companies Incorporated. London.

Chaudhary, S.M. (2017). Introduction to Statistical Theory Part-1. Eighth Edition. Ilmi
Kitab Khana. Lahore.

Daniel, W.W. (1995). Biostatistics: a foundation for Analysis in Health sciences. Sixth
Edition. John Wiley and sons Incorporated. USA.

Harper, W.M. (1991). Statistics. Sixth Edition. Pitman Publishing, Longman Group,
United Kingdom.

Hoel, P.G. (1976). Elementary Statistics. Four Edition. John Wiley and Sons
Incorporated, NewYork. Pp 151-204

209
UNIT-9

MEASURES OF DISPERSION

Written by: Dr. Muhammad Yameen Danish


Reviewed by: Prof. Dr. Irshad Ahmad Arshad

210
CONTENTS

Page No
Introduction ........................................................................................................... 191
Objectives ........................................................................................................... 192
9.1 Range, ........................................................................................................... 193
9.2 Quartile Deviation, ................................................................................................. 194
9.3 Mean Deviation, ..................................................................................................... 198
9.4 Standard Deviation, ................................................................................................ 201
9.5 Change of Origin and Scale, .................................................................................. 201
9.6 Variance, ........................................................................................................... 206

211
INTRODUCTION
We have studied the measures of central tendency in the previous unit. These
measures alone are not adequate to describe the data sets. Two or more data sets can
have the same measure of central tendency but they can be entirely different.
Consider the following simple examples of three data sets:

I. 34, 34, 34, 34, 34, 34, 34, 34, 34, 34, 34, 34

II. 32, 32, 33, 33, 34, 34, 34, 34, 35, 35, 36, 36

III. 21, 24, 26, 28, 31, 34, 34, 37, 39, 42, 45, 47
All these data sets have the same mean, which is 34; however, it can be observed
that these data sets are substantively different from each other. First data set have no
variation; each observation is equal to the mean (34). In the second data set variation
of the observations from their mean is at most 2 units. The variation among the third
data set is relatively higher than the variation in the second data set as some of the
values are 13 units away from the mean. Thus to describe a data set more
adequately, one needs to know the extent of variability. This is given by the
measures of dispersion. There are two types of measures of dispersion,
namely, absolute and relative. Absolute measures of dispersion measure the
variation among the values of a data set in the units of data. The relative measures of
dispersion measure the variation among the values of a data set relative to their
average and are unit less. The most commonly used measures of dispersion or
variability are given in following table.

Absolute Measures Relative Measures

Range Coefficient of range

Quartile Deviation Coefficient of Quartile Deviation

Mean Deviation Coefficient Quartile Deviation

Standard Deviation Coefficient Standard Deviation

Variance Coefficient of Variance

212
OBJECTIVES
The objective of this unit is to introduce the students with different measures of
dispersion and their application to data in different fields. The selection of the
material reflects the most widely used statistical measures of dispersion in data
presentation and summarization. By the end of this unit, the students are expected to
have basic knowledge of measures of dispersion particularly to understand and apply
the appropriate measure of dispersion for the purpose of presentation and
summarization of statistical data at hand.

213
9.1 Range
The difference between minimum value and maximum value in the data is called
range. The main quality of range is its simple computationand main drawback of
range is its sensitivity to outliers.
Consider the calculation of range for ungrouped data set with values 1 , 2 , … ,
. If (1) and ( ) denote the smallest and largest values respectively, then range
(R) and coefficient of range (CR) are defined as

= ( ) − (1)

and

= ( ) (1) . (
) + (1)

Now consider the calculation of range and coefficient of range for the grouped data
in the form of following frequency distribution.
Mid-point (x) ... ...

Class Boundaries 1 − 1 2 − 2 ... − −

Frequency(f) ... ...


1 2

The range and coefficient of range can be calculated by using the following
formulae

= − 1 = ℎ − ,
and
− −
= 1 = ℎ ,
+ 1 ℎ +
where
= lower class limit(boundary) of the lowest class and

ℎ = upper class limit(boundary) of the highest class.

214
Example # 1: The heights (in centimeters) of second semester students of BS
Statistics are measured nearest to whole number as 56, 71, 62, 65, 59, 67, 64, 68, 70,
63. Determine the range and coefficient range.

Solution: It is simple to find out that (1) = 56 and ( ) = 71 ,

therefore
= ( ) − (1) = 71 − 56 = 15

and
− 71 − 56 15
= ( ) (1) = = = 0.118 = 11.8%
( ) + (1) 71 + 56 127

Example # 2: The marks out of 100 of 64 students of M.Sc. Statistics class in the
first sessional examination of Statistical Inference are given in the form of
following frequency distribution:
Marks 10 – 20 – 30 – 40 – 50 – 60 – 70 – 80 –
Groups 19 29 39 49 59 69 79 89
No. of

Students 1 3 2 16 18 13 9 2

Compute range and coefficient of range for the distribution of marks.


Solution: Since = 10 and ℎ = 89 , therefore, = ℎ − = 89 − 10
= 79

and
ℎ− 89 − 10 79
= = = = 0.7980 = 79.80% ℎ +
89 + 10 99
The range is often used in the fields of industrial quality control and meteorology. It
is some time used to obtain the quick estimate of standard deviation defined
subsequently. It is widely used in the study of prices stocks and returns. It is also
used as the first steps in the construction of frequency distributions and boxplots.

9.2 Quartile Deviation

215
The Quartile Deviation is defined as half the difference between the first and the
third quartiles. It gives an idea about the range within which the central 50% of data
lie. If the quartile deviation is large it means that the middle 50% of the observations
are spaced wide apart. Based on the quartile deviation, the Coefficient of
Quartile Deviation can be defined, which makes it easy to compare the spread of two
or more different distributions.

Consider the calculation of quartile deviation and coefficient of quartile deviation


from ungrouped data set with values 1 , 2 , … , . If 1 and 3 are the
corresponding 1st and 3rd quartiles respectively, then the quartile deviation (Q) and
coefficient of quartile deviation (CQ) is defined as

3 − 1
=
2
and

= 3 1. 3
+ 1

Since the formulae for quartile deviation and coefficient of quartile deviation
depends on data only through 1 and 3 , it makes no difference while we are using
the ungrouped data or the grouped data. Definitely, the calculation of quartiles
for ungrouped data and grouped data will make the difference. The quartile
deviation as a measure of dispersion can be considered as an improved version of
range and best measure of dispersion for open-end classification.
Finally, one other related statistic is the interquartile range (IQR).
Example # 3: The waiting times (in minutes) of 50 students standing in a queue
before the ABL window to be served for admission to Semester Spring-2018 on the
last date were recorded as given below:
0.30, 2.22, 0.71, 3.53, 2.15, 4.18, 0.16, 1.25, 2.46, 8.83,
1.51, 0.92, 2.49, 2.55, 2.35, 0.50, 2.17, 2.35, 0.08, 1.22,
0.31, 1.52, 0.69, 0.24, 0.80, 1.16, 2.98, 3.72 0.58, 6.57,
0.02, 3.93, 0.02, 1.96, 2.56, 2.61, 1.67, 0.23, 8.61, 4.84,
4.67, 4.63, 5.31, 1.11, 0.54, 1.95, 0.20, 0.57, 2.51, 1.98.

216
Compute quartile deviation and coefficient of quartile deviation.

Solution: To calculate quartile deviation we first arrange the data as


0.02, 0.02, 0.08, 0.16, 0.20, 0.23, 0.24, 0.30, 0.31, 0.50,
0.54, 0.57, 0.58, 0.69, 0.71, 0.80, 0.92, 1.11, 1.16, 1.22,
1.25, 1.51, 1.52, 1.67, 1.95, 1.96, 1.98, 2.15, 2.17, 2.22,
2.35, 2.35, 2.46, 2.49, 2.51, 2.55, 2.56, 2.61, 2.98, 3.53,
3.72, 3.93, 4.18, 4.63, 4.67, 4.84, 5.31, 6.57, 8.61, 8.83.

The next step is to calculate lower and upper quartiles as

Lower Quartile = 1 = the +1 th value in an array


4

= the 12.75th value in an array

= 0.57 + 0.75 (0.58 – 0.57) = 0.5775 minutes

and Upper Quartile = = the 3 +1 th value in an array


3 4

= the 38.25th value in an array

= 2.61 + 0.25 (2.98 – 2.61) = 2.7025 minutes


Now the quartile deviation and coefficient of quartile deviation are
3 − 1 2.7025 − 0.5775
= = = 1.0625 minutes
2 2
and
− 2.7025 − 0.5775 2.1250
= 3 1 = = = 0.6479 = 64.79% 3
+ 1 2.7025 + 0.5775 3.2800

Example # 4: For the grouped data in example # 2, compute quartile deviation and
coefficient of quartile deviation.

217
Solution: Fist we make the necessary columns in the data table as
Marks No. of Class Cumulative
Groups Students(f) Boundaries frequencies(F)

10 – 19 1 9.5 – 19.5 1 20

– 29 3 19.5 – 29.5 4 30 –

39 2 29.5 – 39.5 6 40 – 49

16 39.5 – 49.5 22 50 – 59

18 49.5 – 59.5 40 60 – 69

13 59.5 – 69.5 53 70 – 79 9

69.5 – 79.5 62 80 – 89 2

79.5 – 89.5 64

For frequency distributions we calculate quartiles by using the formulae as

ℎ 10 64
= + − = 39.5 + − 6 = 39.5 + 7.5 = 47 marks
1 4 16 4
and
ℎ 3 10 3 × 64
= + − = 59.5 + − 40 = 59.5 + 6.15 = 65.65 marks.
3 4 13 4
Now the quartile deviation andcoefficient of quartile deviation can be calculated as
before

3 − 1 65.65 − 47.00
= = = 9.325 marks
and

3 − 1 65.65 − 47.00 18.65


= = = = 0.1656 = 16.56% 3
+ 1 65.65 + 47.00 126.65

218
9.3 Mean Deviation
The mean deviation (MD) also called mean absolute deviation is defined as the mean
of absolute deviations of the observations from some suitable average. Usually
the mean deviation from mean or mean deviation from median is useful. The mean
deviation from median is preferred in the sense that the sum of absolute deviations
from median is minimum. Consider the calculation of mean deviation and
coefficient of mean deviation (CMD) from ungrouped data set with values
1, 2,… , . The formulae for mean deviation from mean and the
corresponding coefficient are
( ) = ∑ =1| − |
and
( ) ( )
= = .
Similarly the formulae for mean deviation from median and the corresponding coefficient
are
( ) = ∑ =1| − |
and
( ) ( )
= = .
Now consider the calculation of mean deviation and coefficient of mean deviation for the
grouped data in the form of following frequency distribution.
Mid-
... ...
points(x)
Frequency(f) 1 2 ... ...

The formulae for mean deviation from mean and the corresponding coefficient are
( ) = ∑ =1 | − |
∑ =1
and
( ) ( )
= = .
Similarly the formulae for mean deviation from median and the corresponding coefficient
are
∑ =1 | − |
( )=
∑ =1
and

219
( ) ( )
= = .
Example # 5: The weights (in kg) of second semester students of BS Statistics are
measured nearest to one decimal point as 37.7, 40.3, 43.3, 44.5, 46.9, 47.6, 48.6, 51.5, 52.4,
53.8. Determine the mean deviation and coefficient of mean deviation. Solution: First we
compute the mean and median as
10

mean = = ∑ =1 = 466.6 = 46.66 kg


10 10
and
46.9 + 47.6
median = = = 47.25 kg.
2
The next step is to find sum of the absolute deviations as
x − − | − | | − |

37.7 −8.96 −9.55 8.96 9.55

40.3 −6.36 −6.95 6.36 6.95

43.3 −3.36 −3.95 3.36 3.95

44.5 −2.16 −2.75 2.16 2.75

46.9 +0.24 −0.35 0.24 0.35

47.6 +0.94 +0.35 0.94 0.35

48.6 +1.94 +1.35 1.94 1.35

51.5 +4.84 +4.25 4.84 4.25

52.4 +5.74 +5.15 5.74 5.15

53.8 +7.15 +6.55 7.15 6.55

Total 41.68 41.20

Now

∑ =1| − | 41.68
( )= = = 4.17 kg
and
( ) 4.17
= = = 0.0894 = 8.94 %.
Similarly
∑ =1| − | 41.2
( )= = = 4.12

220
and
( ) 4.12
= = = 0.0872.

Example # 6: For the grouped data in example # 2, compute mean deviation and
coefficient of mean deviation.
Solution: First we do the necessary calculations as shown in the following table.
Class
f x fx F | − | | − | | − | | − |
Boundaries
9.5 – 19.5 1 14.5 14.5 1 40.63 40.63 40.56 40.56

19.5 – 29.5 3 24.5 73.5 4 30.63 91.89 30.56 91.68

29.5 – 39.5 2 34.5 69.0 6 20.63 41.26 20.56 41.12

39.5 – 49.5 16 44.5 712 22 10.63 170.08 10.56 168.96

49.5 – 59.5 18 54.5 981 40 0.63 11.34 0.56 10.08

59.5 – 69.5 13 64.5 838.5 53 9.37 121.81 9.44 122.72

69.5 – 79.5 9 74.5 670.5 62 19.37 174.33 19.44 174.96

79.5 – 89.5 2 84.5 169.5 64 29.37 58.74 29.44 58.88

64 3528 710.08 708.96

Next we compute mean and median as


∑8 3528
mean = = =1

∑8=1 = 64 = 55.13 marks


and
ℎ 10 64
median = = + − = 49.5 + − 22
2 18 2
10 × 10
= 49.5 + = 49.5 + 5.56 = 55.06 marks
18
Nowthe mean deviation from mean can be computed as

221
∑ =1 | − | 710.08
( )= = = 11.10 marks
∑ =1 64
and coefficient of mean deviation from men as
( ) 11.10
= = = 0.2013.
55.13
Similarlythe mean deviation from median is

∑ =1 | − | 708.96
( )= = = 11.08 marks
∑ =1 64
andcoefficient of mean deviation from median is
( ) 11.08
= = = 0.2012.
55.06

9.4 Standard Deviation


Standard deviation is the most commonly used measure of dispersion. It is a measure of
spread of data about the mean. It is defined as the square root of sum of squared
deviations of the observations from their mean divided by the number of observations. In
other words, the standard deviation of observations 1 , 2 , … , is defined as

= =1( 2 = ∑ =1 2 − ∑ =1 2.
∑ − )

The corresponding coefficient of standard deviation also called coefficient of variation (CV)
is defined as
= × 100.
The coefficient of variation is often used for comparing the consistency of two or more data
sets beside for comparing the dispersion. For the grouped data the standard deviation is
defined as
∑ ( − )2 ∑ 2 ∑ 2
= =1 = =1 − =1 .
∑ =1 ∑ =1 ∑ =1
9.5 Change of Origin and Scale
Sometimes the computational labor can be reduced by changing the origin and scale of the
data as follows. Suppose a is any constant (called provisional mean) chosen
preferably from the centered of the data, define = − . The standard deviation for the
transformed data is equivalently defined as

222
∑ ( − )2 ∑ − 2 ∑ 2 ∑ 2

= =1 = =1 = =1 − =1 . Furthermore

suppose h is any nonzero constant, define = ( − )/ℎ. The standard


deviation for the transformed grouped data is equivalently defined as

∑ ( − )2 ∑ ( − )2 ∑ 2 ∑ 2
= =1 =ℎ =1 =ℎ =1 − =1 .
∑ =1 ∑ =1 ∑ =1 ∑ =1

9.6 Variance
The square of standard deviation is defined as the variance of data and thus is in squared
units of the data. The above formulae for standard deviation after squaring both sides can be
used to calculate variance directly.

Example #7: For the ungrouped data in example #1, compute standard deviation using
different formulae.

Solution: It is better to construct a table of calculation for such a question as shown below.
( − ) ( − ) −
56 −8.5 72.25 3136 −8 64
59 −5.5 30.25 5041 −5 25
62 −2.5 6.25 3844 −2 4
63 −1.5 2.25 4225 −1 1
64 −0.5 0.25 3481 0 0
65 0.5 0.25 4489 1 1
67 2.5 6.25 4096 3 9
68 3.5 12.25 4624 4 16
70 5.5 30.25 4900 6 36
71 6.5 42.25 3969 7 49
645 0 202.50 41805 5 205

To compute the standard deviation by definitional formula, we need the mean of the data
which is computed first as
∑10 645
= =1 = = 64.5 cm
10 10

Now the standard deviation can be computed as

223
∑ ( − )2 ∑1 0 ( − 64.5)2 202.5
= =1 = =1 = = √20.25 = 4.5 cm.
10 10

Next we use the computing formula to compute the standard deviation as


∑ 2 ∑ 2 ∑10 2 41805
= =1 − =1 = =1 − (64.5)2 = − 4160.25
10 10

= √4180.5 − 416025 = √20.25 = 4.5 cm.


Now we use the shortcut formula given by

∑ 2 ∑ 2 205 5 2
= =1 − =1 = − = √20.50 − 0.25 = √20.25 = 4.5.
10 10
It may be noted that all the formulae provide the same result which must be same except for
rounding error.
Example # 8: For the grouped data in example # 2, calculate standard deviation using
different formulae.
Solution: First we make the necessary columns shown in the following table, where
= ( − )/ℎ = ( − 44.5)/10.
Limits F x fx ( − ) ( − ) f fu f

10 – 19 1 14.5 14.5 −40.63 1650.80 210.25 −3 −3 9 20 –

29 3 24.5 73.5 −30.63 2814.59 1800.75 −2 −6 12 30 – 39

2 34.5 69.0 −20.63 851.19 2380.50 −1 −2 2 40 – 49 16

44.5 712 −10.63 1807.95 31684.00 0 0 0 50 – 59 18 54.5

981 −0.63 7.14 53464.50 1 18 18 60 – 69 13 64.5

838.5 9.37 1141.36 54083.25 2 26 52 70 – 79 9 74.5

670.5 19.37 3376.77 49952.25 3 27 81 80 – 89 2 84.5

169.5 29.37 1725.19 14280.50 4 8 32 Total 64 3528

13375 207856 68 202

First we use the definitional formula given by

224
∑ ( − )2
= =1 = 13375/64 = √208.98 = 14.46 marks.
∑ =1

Next we use the computing formula given by


∑ 2 ∑ 2 207856 3528 2
= =1 − =1 = −
∑ =1 ∑ =1 64 64

= √3247.75 − 3038.77 = √208.98 = 14.46 marks.

Finally we use the shortcut formula given by


∑ 2 ∑ 2 202 68 2
=ℎ =1 − =1 = 10 −
∑ =1 ∑ =1 64 64

= 10√3.16 − 1.13 = 10√2.03 = 10 × 1.424 = 14.24 marks.


This small difference is due to rounding error.

Exercise
Q. 1 Explain different absolute measures of dispersion with their computing formulae.
Q. 2 From the following data calculate (a) mean deviation and coefficient of mean
deviation (b) standard deviation and coefficientof standard deviation.

Classes 1 – 7 8 – 14 15 – 21 22 – 28 29 – 35 36 – 42 43 – 49
Frequency 45 32 34 22 20 12 9

Q. 3 Elaborate different relative measures of dispersion with their computing


formulae.
Q. 4 The following frequency distribution represents the time in seconds needed to
serve the customers by cashiers at Gul Ahmed Discount Store in Rawalpindi.
Compute average service time using mean and average service variation using
Time (in seconds) 20-29 30-39 40-49 50-59 60-69 70-79 80-89 90-99
Frequency 6 16 21 29 25 22 14 10

Q. 5 Differentiate between absolute and relative measures of dispersions with their


corresponding formulae.
Q. 6 The Complex Hospital Islamabad has the following data of weights in pounds of
200 premature babies:

Class 0.5 – 0.9 1.0 – 1.4 1.5 – 1.9 2.0 – 2.4 2.5 – 2.9 3.0 – 3.4 3.5 – 3.9 f
10 19 24 27 29 44 47
Calculate absolute measures of dispersion.

Q. 7 The following data represent the weights in pounds of a sample of packages

225
carried out last month by small airfreight company.
Class 10-10.9 11-11.9 12-12.9 13-13.9 14-14.9 15-15.9 16-16.9 17-17.9 f
1 4 6 8 12 15 11 8
Calculate (a) mean and standard deviation using direct and coding methods (b)
coefficient of variation.

Q. 8 The data given below are a sample from the daily production rate of fiberglass
boats:
17 21 18 27 17 21 20 22
18 23
Calculate coefficients of mean deviation and standard deviation.

Q.9 Student ages in the regular daytime M.B.A. program and the evening program
are described below:

Regular M.B.A 23 29 27 22 24 21 25 26 27 24 31 26
Evening M.B.A 27 34 30 39 28 30 34 35 28 29 34 37

Use a measure of relative variability to suggest which of the two groups will be easier to
teach if homogeneity of the class is a positive factor in learning.

Q.10 There are a number of possible measures of sales performance including


consistency of a sales person. The following data represent the percentage of goal
met by each of three salespersons over the last five years:

Person A: 88 68 89 92 103
Person B: 76 88 90 86 79
Person C: 104 88 118 88 123

Which salesperson is the most consistent among the three investigated?

Q.11 A certain Transportation Commission is concerned about the speed motorists are
deriving on a section of the main highway. Here are the speeds of 45 motorists:

15 32 45 46 42 39 68 47 18 31
48 49 56 52 39 48 69 61 44 42
38 52 55 58 62 58 48 56 58 48
47 52 37 64 29 55 38 29 62 49
69 18 61 55

Compute (a) range and coefficient of range (b) quartile deviation and coefficient
quartile deviation.

226
REFERENCES
1. Bluman, A.G. (2004). Elementary Statistics. A Step by Step Approach. Fifth
Edition. McGraw-Hill Companies Incorporated. London.

2. Chaudhary, S.M. (2017). Introduction to Statistical Theory Part-1. Eighth Edition.


Ilmi Kitab Khana. Lahore.

3. Daniel, W.W. (1995). Biostatistics: a foundation for Analysis in Health sciences.


Sixth Edition. John Wiley and sons Incorporated. USA.

4. Harper, W.M. (1991). Statistics. Sixth Edition. Pitman Publishing, Longman


Group, United Kingdom.

5. Hoel, P.G. (1976). Elementary Statistics. Four Edition. John Wiley and Sons
Incorporated, NewYork. Pp 151-204

6. Kiani, G. H., & Akhtar, M. S. (2012). Basic statistics, Majeed Book Depot.
Khan, A. A., Mirza, S. H., Ahmad, M. I., Baig, I. & Yaqoob, M. (2011). Business
statistics, Qureshi Brothers Publishers.

7. Anton H, Bevens, I., Davis, S. (2012) Calculus,


10th Edition, Wiley Plus Publishers

8. Bartle, R.G Introduction to Red Analysis 4th adition

9. Royden, H.L. (2005) Real Analysis, Third Edition Person Printice Hall

10. Dugopolski, M, (2004) Algebra for college students 3rd Edition Mc Grow Hill New
York USA.

227

You might also like